UPSC Prelims 2024 General Studies (GS) Paper I – Answer Key & Detailed Solutions

This resource presents a provisional answer key and comprehensive solutions for the UPSC Prelims 2024 General Studies (GS) Paper I. Compiled by experienced educators and subject matter experts, the solutions reflect in-depth knowledge and analysis, yet full accuracy can only be assured after the release of the official key by UPSC.

Consider the following statements:

Statement-1: The atmosphere is heated more by incoming solar radiation than by terrestrial radiation. Statement-II: Carbon dioxide and other greenhouse gases in the atmosphere are good absorbers of long wave radiation.

Which one of the following is correct in respect of the above statements?
(a) Both Statement-I and Statement-II are correct and Statement-II explains Statement-I
(b) Both Statement-I and Statement-II are correct, but Statement-II does not explain Statement-I
(c) Statement-I is correct, but Statement-II is incorrect
(d) Statement-I is incorrect, but Statement-II is correct

Explanation

The correct answer is: (d) Statement-I is incorrect, but Statement-II is correct.

  • Statement-I is incorrect: The atmosphere is not heated more by incoming solar radiation than by terrestrial radiation. Instead, it is mainly heated by long wave terrestrial radiation. The Earth’s surface absorbs shortwave solar radiation and re-radiates it as longwave infrared radiation, which is then absorbed by greenhouse gases in the atmosphere, heating it from below.
  • Statement-II is correctCarbon dioxide and other greenhouse gases in the atmosphere are indeed good absorbers of long wave radiation. These gases absorb the infrared radiation emitted by the Earth’s surface and re-radiate it, contributing to the warming of the atmosphere.

Learn more

  • Terrestrial Radiation:
    • The Earth receives solar energy in the form of shortwave radiation.
    • After absorbing this energy, the Earth re-radiates it as longwave infrared radiation.
    • This longwave radiation is absorbed by atmospheric gases, particularly carbon dioxide and other greenhouse gases.
    • This process is known as terrestrial radiation and is the primary way the atmosphere is heated.
  • Greenhouse Effect:
    • Greenhouse gases such as water vapor, carbon dioxide, methane, and others absorb longwave infrared radiation emitted by the Earth’s surface.
    • These gases then re-radiate the energy in all directions, including back towards the Earth’s surface, creating a warming effect known as the greenhouse effect.
    • This effect is crucial for maintaining the Earth’s temperature, making it habitable.
  • Insolation:
    • Insolation refers to the incoming solar radiation that reaches the Earth’s surface.
    • The atmosphere is largely transparent to this shortwave solar radiation, allowing it to pass through and be absorbed by the Earth’s surface.
    • However, the atmosphere absorbs only a small fraction of this shortwave radiation directly.
  • Energy Balance:
    • The Earth’s energy balance involves the absorption of solar radiation and the emission of terrestrial radiation.
    • Approximately 30% of incoming solar radiation is reflected back to space, while the remaining 70% is absorbed by the Earth’s surface and atmosphere.
    • The absorbed energy is eventually re-radiated as longwave infrared radiation, which is then absorbed by greenhouse gases, maintaining the Earth’s energy balance and temperature.
  • Role of Greenhouse Gases:
    • Water vapor and carbon dioxide are the most significant greenhouse gases in terms of their ability to absorb and re-radiate infrared radiation.
    • These gases trap heat within the atmosphere, preventing it from escaping into space, thus warming the planet.
    • Without the natural greenhouse effect, the Earth’s average temperature would be significantly lower, making it less hospitable for life.

By understanding these processes, it becomes clear that the atmosphere is primarily heated by terrestrial radiation rather than direct solar radiation, and greenhouse gases play a crucial role in this heating mechanism.

Reflection in IAS EXPRESS

From Geography Mind Map Notes >>Insolation and Temperature
Environment Mindmap Notes >> Climate Change

[collapse]

Consider the following statements:

Statement-I: Thickness of the troposphere at the equator is much greater as compared to poles.

Statement-II: At the equator, heat is transported to great heights by strong convectional currents.

Which one of the following is correct in respect of the above statements?

(a) Both Statement-I and Statement-II’ are correct and Statement-II explains Statement-I

(b) Both Statement-I and Statement-II are correct, but Statement-II does not explain Statement-I

(c) Statement-I is correct, but Statement-II is incorrect

(d) Statement-I is incorrect, but Statement-II is correct

Prelims Sureshots – Most Probable Topics for UPSC Prelims

A Compilation of the Most Probable Topics for UPSC Prelims, including Schemes, Freedom Fighters, Judgments, Acts, National Parks, Government Agencies, Space Missions, and more. Get a guaranteed 120+ marks!

Explanation

Correct Answer: (a) Both Statement-I and Statement-II are correct and Statement-II explains Statement-I

  • Statement-I: The thickness of the troposphere at the equator is much greater as compared to the poles.
    • This statement is correct. The troposphere is thicker at the equator due to higher temperatures, which cause thermal expansion and stronger convectional currents that push the troposphere to greater heights.
  • Statement-II: At the equatorheat is transported to great heights by strong convectional currents.
    • This statement is correct. The equator receives more solar radiation, leading to higher temperatures and stronger convectional currents that transport heat to higher altitudes.
  • Explanation: Statement-II explains Statement-I because the strong convectional currents at the equator, driven by higher temperatures, result in the expansion of the troposphere, making it thicker at the equator compared to the poles.

Learn more

  • Troposphere: The troposphere is the lowest layer of Earth’s atmosphere, extending from the surface to an average height of about 8 km at the poles and 18 km at the equator.
  • Thermal Expansion: The equator is warmer, causing the air to expand and rise, leading to a thicker troposphere.
  • Convectional Currents: Strong convectional currents at the equator transport heat to great heights, contributing to the increased thickness of the troposphere.
  • Centrifugal Force: The Earth’s rotation causes a centrifugal force that is strongest at the equator, pushing the atmosphere to greater heights.
  • Seasonal Variation: The thickness of the troposphere varies with the seasons, being thicker in the summer and thinner in the winter.
  • Gravitational Pull: The gravitational pull is stronger at the poles, leading to a contraction of the atmosphere and a thinner troposphere.

These factors collectively explain why the troposphere is thicker at the equator than at the poles.

Reflection in IAS EXPRESS

Geography Mindmap Notes >>Atmosphere

[collapse]

Consider the following:

  1. Pyroclastic debris
  2. Ash and dust
  3. Nitrogen compounds
  4. Sulphur compounds

How many of the above are products of volcanic eruptions?

(a) Only one
(b) Only two
(c) Only three
(d) All four

Explanation

Correct Answer: (d) All four

  • Pyroclastic debris: These are fragments of rock ejected during explosive volcanic eruptions. They include various sizes of particles such as ashlapilliblocks, and bombs.
  • Ash and dust: Volcanic ash consists of fine particles of pulverized rock, minerals, and volcanic glass created during explosive eruptions. These particles can be carried by wind over large distances.
  • Nitrogen compounds: Volcanic lightning during eruptions can fix atmospheric nitrogen into bioavailable forms such as nitrate. This process involves the breaking of the dinitrogen bond and subsequent reactions with oxygen.
  • Sulphur compounds: Volcanic eruptions release gases such as sulfur dioxide (SO2) and hydrogen sulfide (H2S). These gases can form sulfuric acid and other sulfur compounds in the atmosphere.

Learn more

  • Pyroclastic debris:
    • Definition: Fragments of rock ejected during explosive volcanic eruptions.
    • Types: Includes ash (particles <2 mm), lapilli (2-64 mm), blocks (angular fragments >64 mm), and bombs (rounded fragments >64 mm).
    • Formation: Created by the violent expansion of gas that shreds magma into small particles.
  • Ash and dust:
    • Definition: Fine particles of pulverized rock, minerals, and volcanic glass.
    • Dispersal: Can be carried by wind over large distances, affecting areas far from the eruption site.
    • Impact: Can cause respiratory issues, damage machinery, and disrupt transportation and communication systems.
  • Nitrogen compounds:
    • Formation: Created by volcanic lightning, which breaks the dinitrogen bond in the atmosphere.
    • Types: Includes nitrate and ammonia, which are essential for biological processes.
    • Role in prebiotic chemistry: Volcanic lightning may have contributed to the synthesis of nitrogen compounds necessary for the emergence of life.
  • Sulphur compounds:
    • Types: Includes sulfur dioxide (SO2) and hydrogen sulfide (H2S).
    • Sources: Emitted from volcanic vents, fumaroles, and during eruptions.
    • Impact: Can form sulfuric acid in the atmosphere, contributing to acid rain and affecting climate.

All four options listed are indeed products of volcanic eruptions, confirming the correct answer is (d) All four.

Reflection in IAS EXPRESS

From Geography Mindmap Notes >> Vulcanicity, Earthquake and Tsunami

[collapse]

Which of the following is/are correct inference/inferences from isothermal maps in the month of January?

1. The isotherms deviate to the north over the ocean and to the south over the continent.
2. The presence of cold ocean currents, Gulf Stream and North Atlantic Drift make the North Atlantic Ocean colder and the isotherms bend towards the north.

Select the answer using the code given below:

(a) 1 only
(b) 2 only
(c) Both 1 and 2
(d) Neither 1 nor 2

Explanation

The correct answer is (a) 1 only.

  • Statement 1: The isotherms deviate to the north over the ocean and to the south over the continent. This is correct. In January, the isotherms bend northward over the oceans due to the warming effect of warm ocean currents like the Gulf Stream and North Atlantic Drift, and they bend southward over the continents due to the cooling effect of the landmasses.
  • Statement 2: The presence of cold ocean currents, Gulf Stream, and North Atlantic Drift make the North Atlantic Ocean colder and the isotherms bend towards the north. This is incorrect. The Gulf Stream and North Atlantic Drift are warm ocean currents, not cold. They warm the Northern Atlantic Ocean, causing the isotherms to bend northward, indicating warmer temperatures.

Learn more

  • Isotherms: Lines on a map connecting points with the same temperature at a given time or over a specified period. They are used in meteorology to depict temperature distribution.
  • January Temperature Distribution:
    • Northern Hemisphere: Isotherms deviate north over the ocean due to warm currents like the Gulf Stream and North Atlantic Drift, and south over the continents due to the cooling effect of landmasses.
    • Southern Hemisphere: Isotherms are more parallel to latitudes due to the ocean’s moderating effect, leading to more gradual temperature variations.
  • Factors Influencing Isotherms:
    • Latitude: Temperature generally decreases from the equator to the poles.
    • Land and Water: Land heats and cools more rapidly than water, causing deviations in isotherms.
    • Ocean Currents: Warm currents (e.g., Gulf Stream) cause northward bends, while cold currents cause southward bends.
    • Topography: Mountains and valleys can affect local temperature distributions, causing deviations in isotherms.
  • Seasonal Variations:
    • January: More pronounced deviations in the northern hemisphere due to larger landmasses and significant temperature differences between land and sea.
    • July: Isotherms generally follow latitudes more closely, with less pronounced deviations compared to January.

Understanding these patterns helps in comprehending global temperature distributions and the climatic influences of various geographical features.

Reflection in IAS EXPRESS

From Geography Mindmap Notes >> Oceans- Relief & Circulation
From Geography Mindmap Notes >> Oceans- Relief & Circulation

[collapse]

Which of the following countries are well known as the two largest cocoa producers in the world?

(a) Algeria and Morocco

(b) Botswana and Namibia

(c) Côte d’Ivoire and Ghana

(d) Madagascar and Mozambique

Explanation

The two largest cocoa producers in the world are Côte d’Ivoire and Ghana (c).

  • (a) Algeria and Morocco: These countries are not known for significant cocoa production. Algeria and Morocco are not listed among the top cocoa-producing nations.
  • (b) Botswana and Namibia: These countries do not have a notable presence in the cocoa production industry. They are not recognized as major cocoa producers.
  • (c) Côte d’Ivoire and Ghana: These countries are the top two cocoa producers in the world, with Côte d’Ivoire leading and Ghana following closely.
  • (d) Madagascar and Mozambique: While Madagascar produces high-quality cocoa, it contributes less than 1% to the global cocoa supply. Mozambique is not a significant cocoa producer.

Learn More

  • Côte d’Ivoire:
    • Production: 2.2 million tonnes annually.
    • Global Share: 40.9% of the world’s cocoa.
    • Economic Impact: Cocoa contributes 15% to the country’s GDP and 40% to its export market.
    • Challenges: Significant deforestation due to cocoa farming.
  • Ghana:
    • Production: 1.1 million tonnes annually.
    • Global Share: 20% of the world’s cocoa.
    • Economic Impact: Cocoa contributes 3.5% to the nation’s GDP and 25% to its export market.
    • Challenges: Biodiversity degradation and deforestation.
  • Indonesia:
    • Production: Approximately 667,000 tonnes annually.
    • Global Share: 13% of the world’s cocoa.
    • Economic Impact: Cocoa contributes significantly to the nation’s GDP.
    • Challenges: Deforestation and soil degradation.
  • Ecuador:
    • Production: Around 337,000 tonnes annually.
    • Global Share: 5% of the world’s cocoa.
    • Economic Impact: Cocoa contributes 0.5% to the GDP and 8% to exports.
    • Specialty: Known for high-grade single-origin chocolate.
  • Brazil:
    • Production: Approximately 274,000 tonnes annually.
    • Global Share: 5% of the world’s cocoa.
    • Economic Impact: Cocoa farming is crucial for the economy but affects biodiversity.
  • Cameroon:
    • Production: Around 300,000 tonnes annually.
    • Global Share: 5% of the world’s cocoa.
    • Economic Impact: Cocoa contributes 1.2% to the national GDP and 70% to exports.
  • Nigeria:
    • Production: Approximately 280,000 tonnes annually.
    • Global Share: 5% of the world’s cocoa.
    • Economic Impact: Cocoa contributes 0.5% to the national GDP and 0.28% to exports.
  • Peru:
    • Production: Around 171,000 tonnes annually.
    • Global Share: 3% of the world’s cocoa.
    • Economic Impact: Significant source of income for farmers.
    • Initiatives: Efforts to produce deforestation-free cocoa.
  • Dominican Republic:
    • Production: Approximately 76,000 tonnes annually.
    • Global Share: Less than 2% of the world’s cocoa.
  • Madagascar:
    • Production: Less than 1% of the world’s cocoa.
    • Specialty: Known for high-quality, fruity-flavored cocoa.
Reflection in IAS EXPRESS

Geography Mindmap Notes >> World Industries Location patterns and problems

[collapse]

With reference to the Himalayan rivers joining the Ganga downstream of Prayagraj from West to East, which one of the following sequences is correct?

(a) Ghaghara – Gomati – Gandak – Kosi

(b) Gomati Ghaghara – Gandak – Kosi

(c) Ghaghara Gomati – Kosi – Gandak

(d) Gomati Ghaghara – Kosi – Gandak

The correct sequence of the Himalayan rivers joining the Ganga downstream of Prayagraj from west to east is:(b) Gomati — Ghaghara — Gandak — Kosi

Learn more

  • Gomati River:
    • Originates from Gomat Taal near Madho Tanda, Pilibhit.
    • It flows through Uttar Pradesh and joins the Ganga near Saidpur, Kaithi, close to Varanasi.
  • Ghaghara River:
    • Originates from the northern slopes of the Himalayas in the Tibetan Plateau.
    • It is the largest tributary of the Ganges by volume and joins the Ganga at Chhapra, Bihar.
  • Gandak River:
    • Originates from the Great Himalaya Range in Nepal.
    • It flows through Bihar and Uttar Pradesh and joins the Ganga near Patna at Sonpur.
  • Kosi River:
    • Known as the Saptakoshi in Nepal due to its seven tributaries.
    • It flows through northern Bihar and joins the Ganga near Kursela in Katihar district.
    • Known for its shifting course and heavy flooding, earning the nickname “Sorrow of Bihar.”

These rivers are significant for their contributions to the Ganges’ flow and their impact on the regions they traverse, including their roles in agriculture, transportation, and cultural significance.

Reflection in IAS EXPRESS

From Prelims Sureshots >> Geography >> India – Minerals, Rivers, Lakes, Glaciers, Mountains, etc.

[collapse]

Consider the following statements:

Statement-I: Rainfall is one of the reasons for weathering of rocks.

Statement-II: Rain water contains carbon dioxide in solution.

Statement-III: Rain water contains atmospheric oxygen.

Which one of the following is correct in respect of the above statements?
(a) Both Statement-II and Statement-III are correct and both of them explain Statement-I
(b) Both Statement-II and Statement-III are correct, but only one of them explains Statement-I
(c) Only one of the Statements II and III is correct and that explains Statement-I
(d) Neither Statement-II nor Statement-III is correct

Correct Answer: (a) Both Statement-II and Statement-III are correct and both of them explain Statement-I

  • Statement-I: Rainfall is one of the reasons for weathering of rocks.
    • This statement is true. Rainfall contributes to the weathering process through both physical and chemical means.
  • Statement-II: Rain water contains carbon dioxide in solution.
    • This statement is true. Rainwater absorbs carbon dioxide from the atmosphere, forming a weak carbonic acid solution, which can chemically weather rocks.
  • Statement-III: Rain water contains atmospheric oxygen.
    • This statement is true. Rainwater contains dissolved oxygen from the atmosphere, which can participate in oxidation processes that contribute to weathering.

Option Analysis:

  • Option (a): Both Statement-II and Statement-III are correct and both of them explain Statement-I.
    • This option is correct. Both carbon dioxide and atmospheric oxygen in rainwater contribute to the chemical weathering of rocks.
  • Option (b): Both Statement-II and Statement-III are correct, but only one of them explains Statement-I.
    • This option is incorrect. Both statements II and III are accurate and relevant to explaining weathering processes.
  • Option (c): Only one of the Statements II and III is correct and that explains Statement-I.
    • This option is incorrect. Both statements are correct and contribute to the weathering process.
  • Option (d): Neither Statement-II nor Statement-III is correct.
    • This option is incorrect. Both statements are accurate descriptions of the components of rainwater and their roles in weathering.

Learn More

  • Weathering: The breaking down or dissolution of rocks and minerals on the Earth’s surface.
    • Types of Weathering:
      • Physical (Mechanical) Weathering:
        • Involves the breakdown of rocks and minerals by physical forces such as temperature changes, freeze-thaw cycles, and biological activity.
        • Example: Freeze-thaw action where water seeps into cracks, freezes, expands, and breaks the rock apart.
      • Chemical Weathering:
        • Involves the breakdown of rocks and minerals through chemical reactions, often involving water, acids, and gases.
        • Carbonation: Rainwater containing dissolved CO2 forms carbonic acid, which reacts with minerals like calcite in limestone, leading to dissolution.
        • Oxidation: Oxygen in rainwater reacts with minerals, particularly iron-bearing ones, to form oxides or rust, leading to weakening and breakdown.
        • Hydrolysis: Water reacts with minerals to form new minerals and soluble salts, altering the rock structure.
    • Factors Influencing Weathering:
      • Climate: Temperature and precipitation are critical; warm, wet climates enhance chemical weathering, while cold climates promote physical weathering.
      • Rock Type: Different minerals weather at different rates; for example, feldspar weathers faster than quartz.
      • Topography: Slope steepness affects the exposure of rocks to weathering agents.
      • Biological Activity: Plants, microbes, and animals contribute to both physical and chemical weathering through root growth, organic acids, and physical disturbances.
    • Importance of Weathering:
      • Soil Formation: Weathering is essential for breaking down rocks into smaller particles that mix with organic material to form soil.
      • Nutrient Cycling: Releases essential minerals and nutrients into the soil, making them available for plant uptake.
      • Landscape Shaping: Influences the formation of various landforms and contributes to the geological cycle.

Consider the following countries:

1. Finland

2. Germany

3. Norway

4. Russia

How many of the above countries have a border with the North Sea?

(a) Only one

(b) Only two

(c) Only three

(d) All four

The correct answer is (b) Only two.

  • Finland: Does not have a border with the North Sea. Finland is bordered by the Baltic Sea, specifically the Gulf of Bothnia and the Gulf of Finland.
  • Germany: Has a border with the North Sea. Germany’s coastline includes the North Sea and the Baltic Sea.
  • Norway: Has a border with the North Sea. Norway’s coastline extends along the North Sea, the Norwegian Sea, and the Barents Sea.
  • Russia: Does not have a border with the North Sea. Russia’s borders include the Baltic Sea, the Black Sea, and the Pacific Ocean, but not the North Sea.

Thus, only Germany and Norway have borders with the North Sea.

Learn more

North Sea

  • Location: The North Sea is a shallow northeastern arm of the Atlantic Ocean, located between the British Isles and the mainland of northwestern Europe.
  • Borders: It is bordered by Great Britain to the southwest and west, the Orkney and Shetland islands to the northwest, Norway to the northeast, Denmark to the east, Germany and the Netherlands to the southeast, and Belgium and France to the south.
  • Connections: It connects to the Atlantic Ocean through the English Channel in the south and the Norwegian Sea in the north. It also connects to the Baltic Sea via the Skagerrak and Kattegat straits.
  • Economic Importance: The North Sea is one of Europe’s most productive fisheries, a prominent shipping zone, and a source of petroleum and natural gas.
  • Geopolitical Significance: Historically, the North Sea has been crucial for maritime trade and military strategy, influencing the rise of the Vikings, the Hanseatic League, and the British Empire.
  • Environmental Issues: The North Sea faces environmental challenges such as overfishing, industrial and agricultural runoff, dredging, and dumping, leading to efforts to prevent degradation and safeguard economic benefits.
Reflection in IAS EXPRESS

[collapse]

Consider the following information:

Waterfall – Region – River

1. Dhuandhar – Malwa – Narmada

2. Hundru – Chota Nagpur – Subarnarekha

3. Gersoppa – Western Ghats – Netravati

In how many of the above rows is the given information correctly matched?

(a) Only one

(b) Only two

(c) All three

(d) None

The correct answer is (b) Only two.

  • Dhuandhar Falls is located in the Malwa region and is formed by the Narmada River. This information is correctly matched.
  • Hundru Falls is located in the Chota Nagpur Plateau and is formed by the Subarnarekha River. This information is also correctly matched.
  • Gersoppa Falls (also known as Jog Falls) is located in the Western Ghats but is formed by the Sharavati River, not the Netravati River. This information is incorrectly matched.

Learn more

  • Dhuandhar Falls:
    • Located in the Jabalpur district of Madhya Pradesh, India.
    • Formed by the Narmada River as it flows through the Marble Rocks and plunges down, creating a misty appearance.
    • The falls are approximately 30 meters high and are a popular tourist destination.
    • Features a cable car service for tourists to view the falls from different angles.
  • Hundru Falls:
    • Located in the Ranchi district of Jharkhand, India.
    • Formed by the Subarnarekha River, falling from a height of 98 meters.
    • Known for its scenic beauty and rock formations created by erosion.
    • Popular as a picnic spot, especially during the dry season when the water flow is less intense.
  • Gersoppa (Jog) Falls:
    • Located in the Uttara Kannada district of Karnataka, India.
    • Formed by the Sharavati River, with a drop of 253 meters, making it one of the highest waterfalls in India.
    • The falls consist of four distinct cascades: Raja, Rani, Roarer, and Rocket.
    • A major tourist attraction, especially during the monsoon season when the water flow is at its peak.
Reflection in IAS EXPRESS

Prelims Sureshots >> Geography >> Major Waterfalls

[collapse]

Consider the following information:

RegionName of the mountain rangeType of mountain
1. Central AsiaVosgesFold mountain
2. EuropeAlpsBlock mountain
3. North AmericaAppalachiansFold mountain
4. South AmericaAndesFold mountain

In how many of the above rows is the given information correctly matched?

(a) Only one
(b) Only two
(c) Only three
(d) All four

Correct Answer: (b) Only two

Explanation:

  • Row 1: Central Asia – Vosges – Fold mountain
    • Incorrect: The Vosges are block mountains, not fold mountains. They were formed by the uplift of bedrock plates due to a graben at the beginning of the Paleogene period.
  • Row 2: Europe – Alps – Block mountain
    • Incorrect: The Alps are fold mountains, not block mountains. They were formed by the collision of the African and Eurasian tectonic plates, causing the folding and faulting of the Earth’s crust.
  • Row 3: North America – Appalachians – Fold mountain
    • Correct: The Appalachians are fold mountains. They were formed by the collision of tectonic plates, which caused the folding and faulting of the Earth’s crust over millions of years.
  • Row 4: South America – Andes – Fold mountain
    • Correct: The Andes are fold mountains. They were formed by the subduction of the Nazca Plate beneath the South American Plate, leading to the folding and faulting of the Earth’s crust.

Learn more:

Types of Mountains

  • Fold Mountains:
    • Formed by the collision of tectonic plates, causing the Earth’s crust to fold and create mountain ranges.
    • Examples include the HimalayasRockiesAndes, and Appalachians.
    • Characterized by long, linear ranges with high peaks and deep valleys.
  • Block Mountains:
    • Created when large areas of the Earth’s crust are broken and displaced vertically.
    • The uplifted blocks are called horsts, and the lowered blocks are called grabens.
    • Examples include the Vosges and the Black Forest in Europe.
    • Characterized by steep, flat-topped blocks separated by valleys or basins.
  • Volcanic Mountains:
    • Formed by volcanic activity, where magma from the Earth’s mantle reaches the surface and solidifies.
    • Examples include Mount Fuji in Japan, Mount Kilimanjaro in Africa, and Mount St. Helens in the USA.
    • Characterized by conical shapes with craters at the summit.
  • Dome Mountains:
    • Formed when magma pushes the crust upwards but does not erupt.
    • The overlying layers of rock are bent into a dome shape.
    • Examples include the Black Hills of South Dakota.
    • Characterized by rounded, dome-like shapes.
  • Plateau Mountains:
    • Formed by erosion of an uplifted plateau.
    • The plateau is worn down by weathering and erosion, leaving behind mountain-like features.
    • Examples include the Colorado Plateau.
    • Characterized by flat-topped elevations with steep sides.
Reflection in IAS EXPRESS

Geography Mindmap Notes >> Landforms Evolution due to Internal Forces

[collapse]

Consider the following airports:

  1. Donyi Polo Airport
  2. Kushinagar International Airport
  3. Vijayawada International Airport

In the recent past, which of the above have been constructed as Greenfield projects?

(a) 1 and 2 only
(b) 2 and 3 only
(c) 1 and 3 only
(d) 1, 2 and 3

The correct answer is (a) 1 and 2 only.

  • Donyi Polo Airport: This airport has been constructed as a Greenfield project. It is a newly built airport in Arunachal Pradesh, India, aimed at improving connectivity in the northeastern region.
  • Kushinagar International Airport: This airport is also a Greenfield project. It was developed to boost tourism and facilitate international travel, particularly for Buddhist pilgrims visiting the historical site of Kushinagar in Uttar Pradesh, India.
  • Vijayawada International Airport: This airport is not a Greenfield project. It was originally a domestic airport and later upgraded to an international airport, thus not fitting the criteria of a Greenfield project.

Learn more

Greenfield Projects

  • Definition: Greenfield projects refer to new developments on previously undeveloped land. These projects are built from scratch, without the need to remodel or demolish existing structures.
  • Purpose: They are often initiated to meet new demands, improve infrastructure, and stimulate economic growth in underdeveloped or strategically important areas.
  • Examples: Airports, industrial parks, and residential complexes are common types of Greenfield projects.
  • Advantages:
    • Customization: Allows for modern, state-of-the-art facilities tailored to current needs.
    • Efficiency: Can incorporate the latest technology and design standards.
    • Economic Growth: Stimulates local economies by creating jobs and improving connectivity.
  • Challenges:
    • Environmental Impact: Potential disruption to local ecosystems and communities.
    • High Costs: Significant investment required for land acquisition and construction.
    • Regulatory Hurdles: Often involves complex approval processes and compliance with various regulations.

Examples of Greenfield Airports in India

  • Donyi Polo Airport: Located in Arunachal Pradesh, it enhances connectivity in the northeastern region.
  • Kushinagar International Airport: Situated in Uttar Pradesh, it serves as a gateway for international tourists, especially Buddhist pilgrims.
Reflection in IAS EXPRESS

Current Affairs >> Newsbits

[collapse]

With reference to “water vapour”, which of the following statements is/are correct?

  1. It is a gas, the amount of which decreases with altitude.
  2. Its percentage is maximum at the poles.

Select the answer using the code given below:

(a) 1 only
(b) 2 only
(c) Both 1 and 2
(d) Neither 1 nor 2

Correct Answer: (a) 1 only

  • Statement 1Water vapour is a gas, the amount of which decreases with altitude.
    • This statement is correct. The quantity of water vapour in the air depends on the rate of evaporation and the temperature of the air, which determines its holding capacity. Both temperature and evaporation decrease with altitude, resulting in a rapid decrease in water vapour with altitude.
  • Statement 2Its percentage is maximum at the poles.
    • This statement is incorrect. Water vapour is present in varying amounts in the atmosphere, but its concentration is generally lower in arid or very cold locations, such as polar regions. The amount of water vapour is higher in warmer regions and decreases from the equator towards the poles.

Learn more

  • Water Vapour:
    • Definition: Water vapour is the gaseous phase of water, produced from the evaporation or boiling of liquid water or from the sublimation of ice.
    • Properties: It is transparent and less dense than most other constituents of air. It plays a crucial role in the hydrologic cycle and atmospheric processes.
    • Role in Climate: Water vapour is a significant greenhouse gas, contributing more to the greenhouse effect than non-condensable gases like carbon dioxide and methane. It acts as a feedback mechanism, amplifying the effects of initial warming forces.
    • Distribution: The concentration of water vapour in the atmosphere varies, typically making up 2-3% of the atmosphere on average, but can be as high as 4% in some regions. It is lower in colder and arid regions.
    • Altitude Effect: The amount of water vapour decreases with altitude due to lower temperatures and reduced evaporation rates at higher altitudes.
    • Hydrologic Cycle: Water vapour is a key component of the hydrologic cycle, contributing to cloud formation, precipitation, and the overall distribution of water on Earth.
    • Climate Impact: As a greenhouse gas, water vapour creates a positive feedback loop for global warming. Increased temperatures lead to higher evaporation rates, which in turn increase the amount of water vapour in the atmosphere, further enhancing warming.
Reflection in IAS EXPRESS

Geography Mindmap Notes >> Atmosphere

[collapse]

Consider the following description:

  1. Annual and daily range of temperatures is low.
  2. Precipitation occurs throughout the year.
  3. Precipitation varies between 50 cm – 250 cm.

What is this type of climate?

(a) Equatorial climate
(b) China type climate
(c) Humid subtropical climate
(d) Marine West coast climate

Correct Answer: (d) Marine West coast climate

  • Annual and daily range of temperatures is low: The Marine West Coast climate is characterized by minimal temperature extremes and a small annual temperature range of about 10-15 °C (50-59 °F).
  • Precipitation occurs throughout the year: This climate type experiences abundant precipitation year-round, with annual accumulations generally ranging from 50 to 250 cm (20 to 98 inches).
  • Precipitation varies between 50 cm – 250 cm: The Marine West Coast climate typically has precipitation totals within this range, although local totals can exceed 500 cm (197 inches) where onshore winds encounter mountain ranges.

Explanation of Options:

  • (a) Equatorial climate:
    • High temperatures year-round: Equatorial climates have consistently high temperatures, averaging around 26-28 °C (79-82 °F).
    • High annual rainfall: These regions receive significant rainfall, often exceeding 200 cm (79 inches) annually, with no dry season.
    • Small temperature range: The annual temperature range is very small, typically around 3 °C (5.4 °F).
  • (b) China type climate:
    • Hot, humid summers and cold, dry winters: This climate features significant temperature variations between summer and winter, with hot, humid summers and cold, dry winters.
    • Moderate to high rainfall: Rainfall ranges from 60 to 150 cm (24 to 59 inches) annually, with a distinct summer peak due to monsoonal influences.
    • Seasonal variations: The climate is influenced by monsoons, leading to distinct wet and dry seasons.
  • (c) Humid subtropical climate:
    • Hot, humid summers and mild winters: This climate is characterized by hot, humid summers and cool to mild winters.
    • Evenly distributed precipitation: Rainfall is generally well-distributed throughout the year, with annual totals ranging from 75 to over 200 cm (30 to 79 inches).
    • Summer peak in rainfall: Many regions experience a summer peak in rainfall due to convectional thunderstorms and tropical cyclones.
  • (d) Marine West coast climate:
    • Mild temperatures year-round: This climate has mild summers and winters, with a small annual temperature range.
    • Year-round precipitation: Precipitation occurs throughout the year, with annual totals ranging from 50 to 250 cm (20 to 98 inches).
    • High humidity and cloud cover: The climate is noted for its high humidity and frequent cloud cover.

Learn more

  • Geographical Distribution: The Marine West Coast climate is found on the western sides of continents between 35° and 60° N and S latitude. Notable regions include the Pacific Northwest in North America, western Europe, and parts of New Zealand and southeastern Australia.
  • Temperature Characteristics: This climate is characterized by mild temperatures with minimal extremes. Mean annual temperatures typically range from 7-13 °C (45-55 °F) in lowland areas.
  • Precipitation Patterns: Precipitation is abundant and evenly distributed throughout the year, with annual totals generally between 50 and 250 cm. Local variations can occur, especially where onshore winds encounter mountain ranges, leading to higher precipitation totals.
  • Climate Influences: The climate is influenced by mid-latitude westerlies and traveling frontal cyclones, which bring frequent and reliable precipitation. The orientation of mountain ranges can significantly affect the distribution and intensity of precipitation.
  • Vegetation and Ecosystems: The Marine West Coast climate supports lush vegetation, including temperate rainforests. The high humidity and frequent rainfall create ideal conditions for diverse plant and animal life.
Reflection in IAS EXPRESS

Geography Notes >> World Climatic Regions

[collapse]

With reference to “Coriolis force”, which of the following statements is/are correct?

  1. It increases with increase in wind velocity.
  2. It is maximum at the poles and is absent at the equator.

Select the answer using the code given below:

(a) 1 only
(b) 2 only
(c) Both 1 and 2
(d) Neither 1 nor 2

Correct Answer: (c) Both 1 and 2

Explanation:

  • Statement 1: It increases with increase in wind velocity.
    • The Coriolis force is directly proportional to the velocity of the moving object. As the wind speed increases, the deflection caused by the Coriolis force also increases. This means that higher wind speeds result in a greater Coriolis effect, leading to more significant deflection of the wind’s path.
  • Statement 2: It is maximum at the poles and is absent at the equator.
    • The Coriolis force is maximum at the poles and absent at the equator. This is because the Coriolis effect is directly proportional to the sine of the latitude. At the poles, the latitude is 90 degrees, making the sine value 1, thus maximizing the Coriolis force. At the equator, the latitude is 0 degrees, making the sine value 0, thus nullifying the Coriolis force.

Learn more

  • Definition: The Coriolis force is an inertial force that acts on objects in motion within a rotating frame of reference, such as the Earth. It causes moving objects to be deflected to the right in the Northern Hemisphere and to the left in the Southern Hemisphere.
  • Cause: The primary cause of the Coriolis effect is the rotation of the Earth. As the Earth spins, different latitudes rotate at different speeds, causing the deflection of moving objects.
  • Impact on Weather: The Coriolis effect significantly influences weather patterns. It affects the direction of trade windscyclones, and ocean currents. In the Northern Hemisphere, cyclones rotate counterclockwise, while in the Southern Hemisphere, they rotate clockwise.
  • Dependence on Latitude: The Coriolis force is zero at the equator and increases with latitude, reaching its maximum at the poles. This variation is due to the change in the tangential velocity of the Earth’s surface with latitude.
  • Impact on Human Activities: The Coriolis effect impacts aviation and ballistics. Pilots and snipers must account for the Coriolis force when charting flight paths or aiming over long distances.
  • Examples: A common example is the deflection of a projectile fired northward from the equator, which lands east of its intended path due to the higher eastward velocity at the equator compared to higher latitudes.
  • Mathematical Expression: The magnitude of the Coriolis force is given by the formula 2𝜈𝜔sin⁡𝜙, where 𝜈 is the velocity of the object, 𝜔 is the angular velocity of the Earth, and ϕ is the latitude.
  • Global Wind Patterns: The Coriolis effect contributes to the formation of Hadley cellsFerrel cells, and Polar cells, which are responsible for the prevailing wind patterns observed on Earth.
Reflection in IAS EXPRESS

Geography Notes >> Pressure & Winds

[collapse]

On June 21 every year, which of the following latitude(s) experience(s) a sunlight of more than 12 hours?

  1. Equator
  2. Tropic of Cancer
  3. Tropic of Capricorn
  4. Arctic Circle

Select the correct answer using the code given below:

(a) 1 only
(b) 2 only
(c) 3 and 4
(d) 2 and 4

Correct Answer: (d) 2 and 4

  • Equator: On June 21, the equator experiences approximately 12 hours of daylight and 12 hours of darkness. This is because the equator is not significantly affected by the tilt of the Earth’s axis during the solstices, resulting in nearly equal day and night lengths throughout the year.
  • Tropic of Cancer: On June 21, the Tropic of Cancer (23.5° North latitude) experiences more than 12 hours of daylight. This is because the sun is directly overhead at noon, making it the longest day of the year for this latitude.
  • Tropic of Capricorn: On June 21, the Tropic of Capricorn (23.5° South latitude) experiences less than 12 hours of daylight. This is because it is winter in the Southern Hemisphere, and the sun is at its lowest point in the sky for this latitude.
  • Arctic Circle: On June 21, the Arctic Circle (66.5° North latitude) experiences 24 hours of daylight. This phenomenon is known as the “midnight sun,” where the sun does not set, resulting in continuous daylight.

Learn more

  • Summer Solstice: The summer solstice occurs when one of Earth’s poles has its maximum tilt toward the Sun. In the Northern Hemisphere, this happens around June 21, marking the longest day of the year with the most daylight hours. The sun is directly overhead at the Tropic of Cancer (23.5° North latitude) at noon.
  • Earth’s Tilt: The Earth’s axis is tilted at an angle of 23.5° relative to its orbital plane around the Sun. This tilt is responsible for the changing seasons and varying lengths of daylight throughout the year.
  • Daylight Variation: The amount of daylight varies significantly with latitude. Near the equator, day and night lengths remain relatively constant throughout the year. As one moves towards the poles, the variation becomes more extreme, with polar regions experiencing continuous daylight or darkness during solstices.
  • Equinoxes: Equinoxes occur twice a year when the Earth’s axis is not tilted toward or away from the Sun, resulting in nearly equal day and night lengths globally. The vernal (spring) equinox occurs around March 20-21, and the autumnal (fall) equinox occurs around September 22-23.
  • Cultural Significance: The summer solstice has been celebrated in various cultures throughout history. In many European traditions, it is known as Midsummer and is associated with festivals and rituals celebrating fertility and the abundance of the growing season.
Reflection in IAS EXPRESS

Geography Notes >> Motions of earth – rotation & revolution

[collapse]

One of the following regions has the world’s largest tropical peatland, which holds about three years worth of global carbon emissions from fossil fuels; and the possible destruction of which can exert detrimental effect on the global climate. Which one of the following denotes that region?

(a) Amazon Basin
(b) Congo Basin
(c) Kikori Basin
(d) Rio de la Plata Basin

Correct Answer: (b) Congo Basin

  • The Congo Basin is home to the world’s largest tropical peatland, which holds about three years’ worth of global carbon emissions from fossil fuels. The peatlands in the central Congo Basin cover approximately 167,600 square kilometers and store between 26 and 32 billion tonnes of carbon, making them a critical carbon sink. Only 8% of this peat carbon lies within nationally protected areas, suggesting vulnerability to future land-use changes.

Explanation of Options:

  • Amazon Basin: While the Amazon Basin contains significant peatlands, they are not as extensive as those in the Congo Basin. The Amazon’s peatlands are important for regional carbon cycling and habitat diversity but do not match the Congo Basin in terms of carbon storage capacity.
  • Congo Basin: The Congo Basin peatlands are the largest tropical peatland complex in the world, covering 36% of the world’s tropical peatland area and storing 28% of the world’s tropical peat carbon. These peatlands are crucial in the fight against climate change due to their massive carbon storage capacity.
  • Kikori Basin: Located in Papua New Guinea, the Kikori Basin has significant peat swamp forests but does not compare to the Congo Basin in terms of size or carbon storage. The Kikori Basin is known for its biodiversity and ecological importance but is not the largest tropical peatland.
  • Rio de la Plata Basin: This basin, located in South America, is primarily known for its extensive river system and does not contain significant peatland areas. It is not relevant to the context of tropical peatlands and their carbon storage capabilities.

Learn More

  • Importance of Peatlands:
    • Carbon Storage: Peatlands store a significant amount of carbon, preventing it from being released into the atmosphere and contributing to climate change. The Congo Basin peatlands alone store between 26 and 32 billion tonnes of carbon.
    • Climate Regulation: Peatlands help regulate the climate by acting as carbon sinks. They store carbon that plants have absorbed from the atmosphere, providing a net-cooling effect.
    • Biodiversity: Peatlands support diverse ecosystems, including many plant and animal species. The Congo Basin peatlands are home to endangered species such as the western lowland gorilla and the African forest elephant.
  • Threats to Peatlands:
    • Land-Use Changes: Activities such as logging, mining, and agriculture can lead to the drainage and degradation of peatlands, releasing stored carbon into the atmosphere and exacerbating climate change.
    • Oil Exploration: In the Congo Basin, oil exploration poses a significant threat to peatlands. The DRC government has auctioned large areas of peatland for oil development, which could lead to substantial carbon emissions if the peatlands are disturbed.
  • Conservation Efforts:
    • Protected Areas: Only a small percentage of peatlands are currently protected. Increasing the number of protected areas is crucial to preserving these vital ecosystems.
    • Sustainable Management: Engaging with governments, local communities, and industries to find a balance between development and conservation is essential for the long-term protection of peatlands.
Reflection in IAS EXPRESS

Current Affairs >> Newsbits (July 2023)

[collapse]

With reference to perfluoroalkyl and polyfluoroalkyl substances (PFAS) that are used in making many consumer products, consider the following statements:

  1. PFAS are found to be widespread in drinking water, food and food packaging materials.
  2. PFAS are not easily degraded in the environment.
  3. Persistent exposure to PFAS can lead to bioaccumulation in animal bodies.

Which of the statements given above are correct?

(a) 1 and 2 only
(b) 2 and 3 only
(c) 1 and 3 only
(d) 1, 2 and 3

Correct Answer: (d) 1, 2 and 3

Explanation:

  • Statement 1: PFAS are found to be widespread in drinking water, food, and food packaging materials.
    • Correct. PFAS are widely used in various consumer products, including food packaging materials, and have been detected in drinking water and food. Their widespread use and persistence in the environment lead to contamination of these sources.
  • Statement 2: PFAS are not easily degraded in the environment.
    • Correct. PFAS are known for their high persistence in the environment due to the strong carbon-fluorine bonds, making them resistant to degradation. This characteristic has earned them the nickname “forever chemicals”.
  • Statement 3: Persistent exposure to PFAS can lead to bioaccumulation in animal bodies.
    • Correct. PFAS can bioaccumulate in the bodies of animals and humans over time, leading to potential health risks. This bioaccumulation occurs because these substances are not easily broken down and can persist in the body for years.

Learn more

  • Definition and Uses:
    • PFAS (Per- and polyfluoroalkyl substances) are a large group of synthetic chemicals used since the 1940s in various industrial and consumer products for their water, grease, and stain-resistant properties.
    • Common applications include non-stick cookware, water-repellent clothing, stain-resistant fabrics, firefighting foams, and food packaging materials.
  • Environmental Persistence:
    • PFAS are highly persistent in the environment due to the strong carbon-fluorine bonds, which do not break down easily. This persistence leads to long-term environmental contamination.
  • Routes of Exposure:
    • Humans can be exposed to PFAS through multiple routes, including drinking contaminated water, consuming food packaged in PFAS-containing materials, inhaling dust, and using consumer products that contain PFAS.
  • Health Risks:
    • Exposure to certain PFAS has been linked to various health issues, including cancer, hormone disruption, liver and kidney toxicity, immune system harm, and reproductive and developmental toxicity.
  • Bioaccumulation:
    • PFAS can accumulate in the bodies of humans and animals over time. This bioaccumulation is concerning because it can lead to higher concentrations of these chemicals in the body, potentially causing adverse health effects.
  • Regulatory Actions:
    • Various regulatory bodies and governments are taking steps to limit the use of PFAS. Some states and countries have banned or restricted the use of PFAS in certain products, and there are ongoing efforts to find safer alternatives.
  • Research and Monitoring:
    • Ongoing research aims to better understand the environmental and health impacts of PFAS, develop methods to detect and measure PFAS in various media, and find effective ways to remove PFAS from contaminated environments.
  • Alternatives:
    • Alternatives to PFAS-containing products are being developed and are available in the market. These alternatives aim to provide similar benefits without the associated environmental and health risks.
Reflection in IAS EXPRESS

Current Affairs >> Indepth Articles >> Forever Chemicals (PFAS): How They’re Harming Your Health Without You Knowing
Current Affairs >> Indepth Articles >> Forever Chemicals (PFAS): How They’re Harming Your Health Without You Knowing

[collapse]

Consider the following:

  1. Carabid beetles
  2. Centipedes
  3. Flies
  4. Termites
  5. Wasps

Parasitoid species are found in how many of the above kind of organisms?

(a) Only two
(b) Only three
(c) Only four
(d) All five

The correct answer is: (b) Only three (1, 3 & 5)

  • Carabid beetles:
    • Carabid beetles, a diverse family of ground beetles, exhibit a range of behaviors, including predation and parasitism.
    • While most carabid beetles are known for their predatory habits, some species do exhibit parasitoid behavior.
  • Centipedes:
    • Centipedes are predatory arthropods that are primarily carnivorous.
    • They do not have any known parasitoid species.
  • Flies:
    • Many species of flies, especially those in the families Tachinidae and Sarcophagidae, have parasitoid species.
    • These flies lay their eggs in or on other insects, and the larvae develop inside the host, eventually killing it.
  • Termites:
    • Termites are primarily decomposers and are not typically associated with parasitoid behavior.
    • However, certain species of flies and wasps can act as parasitoids to termites.
  • Wasps:
    • Wasps, especially those in the families Ichneumonidae and Braconidae, are well-known for their parasitoid behavior.
    • They lay their eggs in or on a host insect, and the developing larvae consume the host from the inside.

Learn more

Parasitoid Species

  • Definition: A parasitoid is an organism that spends a significant portion of its life attached to or within a single host organism, ultimately leading to the host’s death. This is different from parasites, which usually do not kill their hosts.
  • Life Cycle:
    • The life cycle of parasitoids typically involves laying eggs on or inside a host.
    • The larvae hatch and feed on the host’s tissues, eventually leading to the host’s demise.
    • Adult parasitoids emerge from the host to continue the cycle.
  • Ecological Role:
    • Parasitoids play a crucial role in biological control by naturally regulating the populations of their hosts, many of which are agricultural pests.
    • They contribute to the balance of ecosystems by keeping pest populations in check.
  • Examples:
    • Tachinid flies: These flies are parasitoids of caterpillars, beetles, and other insects.
    • Braconid wasps: They parasitize caterpillars, aphids, and beetle larvae.
    • Ichneumon wasps: Known for their long ovipositors, they target caterpillars, beetles, and other insects.
  • Adaptations:
    • Parasitoids have evolved various adaptations to locate and exploit their hosts, including acute sensory organs to detect host cues, venoms to paralyze hosts, and behavioral strategies to avoid host defenses.
  • Importance in Agriculture:
    • By reducing pest populations, parasitoids help minimize the need for chemical pesticides, promoting sustainable agricultural practices.
    • They are often introduced as part of integrated pest management (IPM) programs to control invasive pest species.
Reflection in IAS EXPRESS

Prelims Sureshots >> Environment >> Organisms Facts
Prelims Sureshots >> Environment >> Organisms Facts
Prelims Sureshots >> Environment >> Organisms Facts

[collapse]

Consider the following plants:

  1. Groundnut
  2. Horse-gram
  3. Soybean

How many of the above belong to the pea family?

(a) Only one
(b) Only two
(c) All three
(d) None

Correct Answer: (c) All three

Explanation:

  • Groundnut (Peanut):
    • Scientific NameArachis hypogaea
    • Family: Fabaceae (Leguminosae)
    • Details: Groundnut is a legume of the pea family, grown for its edible seeds and known for its nitrogen-fixing ability, which enriches the soil.
  • Horse-gram:
    • Scientific NameMacrotyloma uniflorum
    • Family: Fabaceae (Leguminosae)
    • Details: Horse-gram is a legume native to tropical southern Asia, known for its nutritional and medicinal properties, and is part of the Fabaceae family.
  • Soybean:
    • Scientific NameGlycine max
    • Family: Fabaceae (Leguminosae)
    • Details: Soybean is a species of legume native to East Asia, widely grown for its edible bean and numerous uses, and belongs to the Fabaceae family.

Learn more:

Fabaceae Family

  • Overview:
    • Also known as Leguminosae or pea family.
    • Includes trees, shrubs, and herbaceous plants.
    • Third-largest family of flowering plants with about 20,000 species.
  • Characteristics:
    • Leaves: Usually pinnately compound, sometimes trifoliate or palmate.
    • Flowers: Typically have five petals and sepals, often with vexillary aestivation.
    • Fruit: Typically a legume or pod that splits open to release seeds.
  • Economic Importance:
    • Nitrogen Fixation: Plants in this family have root nodules with nitrogen-fixing bacteria, enriching the soil.
    • Food Source: Includes important crops like peas, beans, lentils, peanuts, and soybeans, which are rich in protein.
    • Industrial Uses: Oils from soybeans and groundnuts are used in cooking and various industrial applications.
    • Medicinal Uses: Some plants have medicinal properties and are used in traditional medicine.
  • Ecological Role:
    • Soil Enrichment: Improves soil fertility through nitrogen fixation.
    • Biodiversity: Supports a wide range of insects and animals, contributing to ecosystem health.

Consider the following statements:

Statement-I: The Indian Flying Fox is placed under the “vermin” category in the Wild Life (Protection) Act, 1972.

Statement-II: The Indian Flying Fox feeds on the blood of other animals.

Which one of the following is correct in respect of the above statements?

(a) Both Statement-I and Statement-II are correct and Statement-II explains Statement-I
(b) Both Statement-I and Statement-II are correct, but Statement-II does not explain Statement-I
(c) Statement-I is correct, but Statement-II is incorrect
(d) Statement-I is incorrect, but Statement-II is correct

Correct Answer: (c) Statement-I is correct, but Statement-II is incorrect

  • Statement-I is correct: The Indian Flying Fox is indeed placed under the “vermin” category in the Wild Life (Protection) Act, 1972. This classification is due to the damage they cause to fruit orchards and their potential role in spreading diseases like the Nipah virus.
  • Statement-II is incorrect: The Indian Flying Fox does not feed on the blood of other animals. Instead, it has a frugivorous diet, primarily consuming fruits, nectar, and occasionally insects.

Explanation:

  • Option (a): Both Statement-I and Statement-II are correct and Statement-II explains Statement-I.
    • This option is incorrect because Statement-II is incorrect. The Indian Flying Fox does not feed on blood.
  • Option (b): Both Statement-I and Statement-II are correct, but Statement-II does not explain Statement-I.
    • This option is incorrect because Statement-II is incorrect. The Indian Flying Fox is not a blood-feeding species.
  • Option (c): Statement-I is correct, but Statement-II is incorrect.
    • This option is correct. Statement-I is correct as the Indian Flying Fox is classified as vermin, and Statement-II is incorrect as it does not feed on blood.
  • Option (d): Statement-I is incorrect, but Statement-II is correct.
    • This option is incorrect because Statement-I is correct and Statement-II is incorrect.

Learn more

  • Indian Flying Fox (Pteropus giganteus):
    • Classification: It is classified as vermin under the Wild Life (Protection) Act, 1972 due to its impact on fruit orchards and potential disease transmission.
    • Diet: The Indian Flying Fox is frugivorous, feeding on a variety of fruits, nectar, and occasionally insects. It plays a crucial role in pollination and seed dispersal, contributing to ecosystem health.
    • Habitat: Found in tropical regions of South Central Asia, including India, Pakistan, Nepal, and the Maldives. They roost in large colonies in trees near water bodies.
    • Behavior: These bats are nocturnal and highly social, forming large roosts. They exhibit vigilant behavior during the day to watch for dangers.
    • Conservation Status: Despite being considered pests, they are vital for maintaining biodiversity. Their conservation is essential for ecosystem balance.
Reflection in IAS EXPRESS

Prelims Sureshots >> Environment >> Scheduled Animals of WPA 1972

[collapse]

The organisms “Cicada, Froghopper and Pond skater” are:

(a) Birds
(b) Fish
(c) Insects
(d) Reptiles

Correct Answer: (c) Insects

Explanation:

  • Cicada:
    • Cicadas belong to the order Hemiptera and are part of the family Cicadidae.
    • They are known for their sound-producing abilities and have a life cycle that includes egg, nymph, and adult stages.
  • Froghopper:
    • Froghoppers are part of the superfamily Cercopoidea within the order Hemiptera.
    • They are known for their jumping abilities and the foam shelters produced by their nymphs, commonly called spittlebugs.
  • Pond Skater:
    • Pond skaters, also known as water striders, belong to the family Gerridae within the order Hemiptera.
    • They are recognized for their ability to walk on water due to their hydrophobic legs.

Other Options:

  • Birds:
    • Birds are endothermic animals with feathers, wings, and beaks.
    • They have pneumatic bones and a high metabolic rate to support flight.
    • Examples include penguins, hawks, and songbirds.
  • Fish:
    • Fish are cold-blooded vertebrates with gills, fins, and scales.
    • They have a swim bladder for buoyancy and a two-chambered heart.
    • Examples include salmon, trout, and sharks.
  • Reptiles:
    • Reptiles are cold-blooded animals with scales and internal fertilization.
    • They have a three-chambered heart (except crocodiles, which have four chambers) and lay eggs.
    • Examples include snakes, lizards, and turtles.

Learn More

  • Cicada:
    • Cicadas are known for their periodical emergence every 13 or 17 years in some species.
    • They have prominent compound eyes and membranous wings.
    • Their songs are produced by vibrating tymbals near the base of the abdomen.
  • Froghopper:
    • Froghoppers can jump up to 70 cm vertically, an impressive feat relative to their body size.
    • They produce foam nests to protect their nymphs, which are commonly seen in meadows during spring.
    • They have stout spines on their hind tibiae, distinguishing them from leafhoppers.
  • Pond Skater:
    • Pond skaters have hydrophobic legs that allow them to stand on water due to surface tension.
    • They have a slender body and three pairs of legs adapted for different functions: grabbing prey, paddling, and steering.
    • They are found in freshwater habitats worldwide and play a role in controlling insect populations.
Reflection in IAS EXPRESS

Prelims Sureshots >> Environment >> Organism Facts

[collapse]

Consider the following statements:

Statement-I: Many chewing gums found in the market are considered a source of environmental pollution.

Statement-II: Many chewing gums contain plastic as gum base.

Which one of the following is correct in respect of the above statements?

(a) Both Statement-I and Statement-II are correct and Statement-II explains Statement-I
(b) Both Statement-I and Statement-II are correct, but Statement-II does not explain Statement-I
(c) Statement-I is correct, but Statement-II is incorrect
(d) Statement-I is incorrect, but Statement-II is correct

Correct Answer: (a) Both Statement-I and Statement-II are correct and Statement-II explains Statement-I

Explanation:

  • Option (a): Both Statement-I and Statement-II are correct and Statement-II explains Statement-I
    • Statement-I: Many chewing gums found in the market are considered a source of environmental pollution. This is correct because chewing gum contributes significantly to plastic pollution, with an estimated 100,000 tonnes of plastic pollution annually due to improper disposal of gum.
    • Statement-II: Many chewing gums contain plastic as gum base. This is also correct as most modern chewing gums are made from synthetic polymers, which are essentially plastics.
    • Explanation: Statement-II explains Statement-I because the presence of plastic in the gum base makes chewing gum non-biodegradable, leading to long-lasting environmental pollution when improperly disposed of.
  • Option (b): Both Statement-I and Statement-II are correct, but Statement-II does not explain Statement-I
    • This option is incorrect because Statement-II directly explains why chewing gums are a source of environmental pollution.
  • Option (c): Statement-I is correct, but Statement-II is incorrect
    • This option is incorrect because Statement-II is correct; many chewing gums do contain plastic as a gum base.
  • Option (d): Statement-I is incorrect, but Statement-II is correct
    • This option is incorrect because Statement-I is correct; chewing gums are indeed a source of environmental pollution.

Learn More

  • Environmental Impact of Chewing Gum:
    • Plastic Pollution: Chewing gum contributes to plastic pollution as it is made from synthetic polymers that do not biodegrade. An estimated 100,000 tonnes of plastic pollution is generated annually from chewing gum.
    • Microplastics: As gum breaks down, it forms microplastics that persist in the environment, contaminating soil and waterways, and entering the food chain, causing ecological damage.
    • Economic Costs: The cleanup of gum litter is costly. For example, the UK spends millions annually to remove gum from streets.
  • Composition of Chewing Gum:
    • Gum Base: The gum base is a non-nutritive, non-digestible, water-insoluble substance that provides the chewiness of gum. It often contains synthetic polymers like polyethylene and polyvinyl acetate, which are types of plastic.
    • Ingredients: Besides the gum base, chewing gum contains sweeteners, flavors, and softeners. The exact composition is often a trade secret.
  • Health and Environmental Concerns:
    • Health Issues: Chewing gum can cause gastrointestinal issues, tooth decay, and muscle dysfunction if consumed excessively.
    • Wildlife Harm: Microplastics from gum can be ingested by wildlife, causing internal injuries and digestive problems, and potentially leading to death.
  • Solutions and Alternatives:
    • Biodegradable Gum: Researchers are working on developing biodegradable gum that retains the pleasure of chewing while being environmentally friendly.
    • Regulation and Awareness: Increased regulation and public awareness about the environmental impact of chewing gum can help reduce gum littering.

Consider the following pairs:

CountryAnimal found in its natural habitat
1. BrazilIndri
2. IndonesiaElk
3. MadagascarBonobo

How many of the pairs given above are correctly matched?

(a) Only one
(b) Only two
(c) All three
(d) None

Correct Answer: (d) None

Explanation:

  • Pair 1: Brazil – Indri
    • The Indri is native to Madagascar, not Brazil. It is a diurnal tree-dweller and one of the largest living lemurs, found in the lowland and montane forests along the eastern coast of Madagascar.
  • Pair 2: Indonesia – Elk
    • The Elk is found in North America and Central and East Asia, not Indonesia. They inhabit diverse habitats such as coniferous rain forests, prairies, and mountainous areas.
  • Pair 3: Madagascar – Bonobo
    • The Bonobo is found only in the Democratic Republic of the Congo, specifically in the lowland rainforests along the south bank of the Congo River, not Madagascar.

Learn more

  • Indri:
    • Habitat: Found in the lowland and montane forests along the eastern coast of Madagascar.
    • Diet: Herbivorous, primarily folivorous, feeding on leaves, seeds, fruits, and flowers.
    • Behavior: Monogamous, lives in small family groups, and communicates through vocalizations.
    • Conservation Status: Critically endangered due to habitat destruction and hunting.
  • Elk:
    • Habitat: Found in North America and Central and East Asia, thriving in diverse habitats such as coniferous rain forests, prairies, and mountainous areas.
    • Diet: Grazes on grasses and sedges, and browses on higher-growing plants, leaves, twigs, and bark.
    • Behavior: Males have large antlers and engage in ritualized mating behaviors; they migrate seasonally.
    • Conservation Status: Listed as least concern by the IUCN.
  • Bonobo:
    • Habitat: Found only in the lowland rainforests along the south bank of the Congo River in the Democratic Republic of the Congo.
    • Diet: Primarily frugivorous, eating fruits, vegetation, and occasionally invertebrates.
    • Behavior: Known for their peaceful and egalitarian society, led by females, and use sexual behaviors to maintain social bonds.
    • Conservation Status: Endangered due to habitat destruction and poaching.

Consider the following statements regarding World Toilet Organization:

  1. It is one of the agencies of the United Nations.
  2. World Toilet Summit, World Toilet Day and World Toilet College are the initiatives of this organization, to inspire action to tackle the global sanitation crisis.
  3. The main focus of its function is to grant funds to the least developed countries and developing countries to achieve the end of open defecation.

Which of the statements given above is/are correct?

(a) 2 only
(b) 3 only
(c) 1 and 2
(d) 2 and 3

Correct Answer: (a) 2 only

Explanation:

  • Statement 1Incorrect. The World Toilet Organization (WTO) is not an agency of the United Nations. It is an independent global non-profit organization founded in 2001 to improve toilet and sanitation conditions worldwide.
  • Statement 2Correct. The WTO has several initiatives, including the World Toilet SummitWorld Toilet Day, and World Toilet College, aimed at inspiring action to tackle the global sanitation crisis.
  • Statement 3Incorrect. The main focus of the WTO is not to grant funds to the least developed and developing countries. Instead, it focuses on advocacy, education, and practical solutions to improve sanitation conditions globally.

Learn more

  • World Toilet Organization (WTO):
    • Founded: 19 November 2001 by Jack Sim in Singapore.
    • Mission: To improve toilet and sanitation conditions worldwide.
    • Vision: A world with a clean, safe toilet for everyone, everywhere at all times.
    • Key Initiatives:
      • World Toilet Day: Established to draw attention to the global sanitation crisis.
      • World Toilet Summit: An annual event to discuss sanitation issues and solutions.
      • World Toilet College: Provides training on best practices in toilet design, cleanliness, and sanitation technologies.
    • Projects:
      • Rainbow School Toilet Initiative: Launched in 2015 to improve sanitation in rural schools.
      • Floating Community Toilet Project: Provides sanitation systems to floating communities in Cambodia.
    • Impact: The WTO has trained over 5,000 people and installed sanitation systems benefiting thousands of students and community members.

Consider the following statements:

  1. Lions do not have a particular breeding season.
  2. Unlike most other big cats, cheetahs do not roar.
  3. Unlike male lions, male leopards do not proclaim their territory by scent marking.

Which of the statements given above are correct?

(a) 1 and 2 only
(b) 2 and 3 only
(c) 1 and 3 only
(d) 1, 2 and 3

Correct Answer: (a) 1 and 2 only

Explanation:

  • Statement 1: Lions do not have a particular breeding season.
    • This statement is correct. Lions do not mate at a specific time of year, and the females are polyestrous, meaning they can come into heat multiple times a year.
  • Statement 2: Unlike most other big cats, cheetahs do not roar.
    • This statement is correct. Cheetahs cannot roar because they have a one-piece hyoid bone, unlike lions, tigers, leopards, and jaguars, which have a two-piece hyoid bone that allows them to roar.
  • Statement 3: Unlike male lions, male leopards do not proclaim their territory by scent marking.
    • This statement is incorrect. Male leopards do proclaim their territory by scent marking, using methods such as spraying urine, clawing trees, and rubbing their cheeks against objects.

Learn more

  • Lion Reproduction and Life Cycle:
    • Breeding Frequency: In the wild, lions typically breed once every two years, while in captivity, they may breed annually.
    • Mating Behavior: Females are receptive to mating for three to four days within a variable reproductive cycle. During this period, a pair may mate every 20-30 minutes, resulting in up to 50 copulations per 24 hours.
    • Gestation and Cubs: The gestation period is about 108-110 days, with litter sizes ranging from one to six cubs. Cubs are born blind and helpless, and they start following their mothers at about three months of age.
    • Maturity and Pride Dynamics: Lions reach sexual maturity at three to four years of age. Male cubs are expelled from the pride at about three years old and become nomads until they can take over another pride.
  • Cheetah Vocalization:
    • Inability to Roar: Cheetahs cannot roar due to their one-piece hyoid bone structure. Instead, they can purr, meow, chirp, and make other sounds.
    • Communication: Cheetahs use vocalizations and scent marking to communicate with each other and avoid predators. They often visit scent-marking sites at different times to minimize encounters with dominant predators like leopards.
  • Leopard Scent Marking:
    • Territorial Behavior: Leopards are solitary animals that use scent marking to communicate and establish their territory. They mark their territory by spraying urine, clawing trees, and rubbing their cheeks against objects.
    • Communication: Scent marking helps leopards avoid conflicts with other leopards and predators. They also use vocalizations like sawing, coughing, and rasping to communicate their presence and reproductive status.

Which one of the following is the correct description of “100 Million Farmers”?

(a) It is a platform for accelerating the transition towards food and water systems that are net-zero (carbon), nature-positive and that aims to increase farmer resilience.

(b) It is an international alliance and a network of individuals and farming organizations interested in supporting and strengthening the development of the organic animal husbandry.

(c) It is a digital platform fully integrated with service providers and built on blockchain that lets buyers, sellers and third parties trade fertilizers quickly and securely.

(d) It is a platform with the mission of encouraging the farmers to form Farmer Product Organizations or Agribusiness Consortiums, thus facilitating the access to global open markets to sell their products.

Correct Answer:(a) It is a platform for accelerating the transition towards food and water systems that are net-zero (carbon), nature-positive and that aims to increase farmer resilience.

Explanation:

  • Option (a) is correct. The “100 Million Farmers” initiative is a multistakeholder platform aimed at transitioning to net-zero, nature-positive food systems by 2030. It supports local solutions that incentivize farmers and empower consumers to place climate, nature, and resilience at the core of the food economy.
  • Option (b) is incorrect. This description matches the IFOAM International Animal Husbandry Alliance (IAHA), which is a network focused on organic animal husbandry.
  • Option (c) is incorrect. This description pertains to the Fertilizer Exchange platform, which is a blockchain-enabled trading platform for the fertilizer industry.
  • Option (d) is incorrect. This description aligns with the Small Farmers’ Agribusiness Consortium (SFAC), which promotes the formation of Farmer Producer Organizations (FPOs) to facilitate access to markets.

Learn more

  • 100 Million Farmers Initiative:
    • Objective: To transition to net-zero, nature-positive food systems by 2030.
    • Stakeholders: Involves public and private leaders, farmers, and consumers.
    • Approach: Operates through regional and national coalitions that develop public-private projects to promote sustainable agriculture.
    • Goals:
      • Climate and Nature: Position food and farmers as central pillars in the global climate and nature agenda.
      • Resilience: Increase farmer resilience by adopting regenerative and climate-smart practices.
      • Collaboration: Facilitate pre-competitive collaboration among value chain companies to aggregate demand for environmental outcomes and improved farm resilience.
    • Implementation: Supports local solutions and incentivizes farmers to adopt sustainable practices, aiming to accelerate the shift towards sustainable consumption and production.
  • IFOAM International Animal Husbandry Alliance (IAHA):
    • Focus: Development of organic animal husbandry.
    • Activities: Organizes conferences, workshops, and provides policy recommendations.
  • Fertilizer Exchange:
    • Platform: A blockchain-enabled trading platform for the fertilizer industry.
    • Purpose: To facilitate secure and reliable trading of fertilizers.
  • Small Farmers’ Agribusiness Consortium (SFAC):
    • Mission: To increase incomes of small and marginal farmers through the formation of Farmer Producer Organizations (FPOs).
    • Activities: Provides financial support, market linkages, and implements the National Agriculture Market Electronic Trading (e-Nam) platform.

Consider the following:

  1. Battery storage
  2. Biomass generators
  3. Fuel cells
  4. Rooftop solar photovoltaic units

How many of the above are considered “Distributed Energy Resources”?

(a) Only one
(b) Only two
(c) Only three
(d) All four

Correct Answer: (d) All four

Explanation:

  • Battery storage: Considered a Distributed Energy Resource (DER). Batteries store electrical energy and can be used to provide power during peak demand or when renewable sources are not generating electricity. They are often used in conjunction with other DERs like solar panels and wind turbines.
  • Biomass generators: Also considered a DER. Biomass generators use organic materials to produce electricity and can be located close to the point of use, making them a part of the distributed energy system.
  • Fuel cells: These are considered DERs as well. Fuel cells generate electricity through a chemical process and can be used in various applications, including residential, commercial, and industrial settings.
  • Rooftop solar photovoltaic units: These are a classic example of DERs. They generate electricity from sunlight and are typically installed on rooftops, providing power directly to the building on which they are installed.

Learn more

  • DefinitionDistributed Energy Resources (DERs) are small-scale units of power generation or storage that are located close to the point of use. They can operate independently or in conjunction with the larger power grid.
  • Types of DERs:
    • Solar Panels: Convert sunlight into electricity and are often installed on rooftops or in small solar farms.
    • Wind Turbines: Small wind turbines can generate electricity for local use, reducing reliance on centralized power plants.
    • Biomass Generators: Use organic materials to produce electricity, often from waste products like agricultural residues or wood chips.
    • Fuel Cells: Generate electricity through a chemical reaction, often using hydrogen as a fuel source. They are efficient and produce low emissions.
    • Battery Storage: Stores electricity for use during peak demand times or when renewable sources are not generating power.
  • Benefits:
    • Energy Independence: DERs allow consumers to generate their own electricity, reducing dependence on the centralized grid.
    • Grid Resilience: By distributing power generation, DERs can make the grid more resilient to outages and disruptions.
    • Environmental Impact: Many DERs use renewable energy sources, which can reduce greenhouse gas emissions and reliance on fossil fuels.
    • Cost Savings: Consumers can save on energy costs by generating their own electricity and potentially selling excess power back to the grid.
  • Challenges:
    • Integration: Incorporating DERs into the existing grid can be complex and requires advanced management systems.
    • Initial Costs: The upfront cost of installing DERs can be high, although this is often offset by long-term savings and incentives.
    • Regulatory Hurdles: Different regions have varying regulations and policies that can impact the deployment and operation of DERs.

Which one of the following shows a unique relationship with an insect that has coevolved with it and that is the only insect that can pollinate this tree?

(a) Fig
(b) Mahua
(c) Sandalwood
(d) Silk cotton

The correct answer is (a) Fig.

  • Fig: The fig tree has a unique relationship with the fig wasp, which is the only insect that can pollinate it. This relationship is a classic example of coevolution, where both species have evolved together to become mutually dependent. The fig wasp lays its eggs inside the fig fruit, and in the process, it pollinates the fig flowers, ensuring the reproduction of both the fig tree and the wasp.
  • Mahua: The mahua tree is pollinated by various insects, including bees and moths, but it does not have a unique relationship with a single insect species for pollination.
  • Sandalwood: Sandalwood trees are primarily pollinated by wind and insects, but there is no specific insect that has a unique coevolved relationship with sandalwood for pollination.
  • Silk cotton: The silk cotton tree is pollinated by a variety of insects and birds, and it does not rely on a single insect species for its pollination.

Learn more

  • Coevolution: This is the process where two or more species influence each other’s evolutionary pathway. In the case of the fig tree and the fig wasp, both species have developed traits that benefit each other, leading to a highly specialized and interdependent relationship.
  • Mutualism: The relationship between the fig tree and the fig wasp is an example of mutualism, where both species benefit. The fig tree gets pollinated, and the fig wasp gets a place to lay its eggs and a food source for its larvae.
  • Pollination: This is the transfer of pollen from the male part of a flower to the female part, enabling fertilization. Insects, birds, wind, and water can all act as pollinators.
  • Specialized Pollination: Some plants have evolved to be pollinated by a single species of insect, which can lead to a highly efficient but risky pollination strategy. If the insect species declines, the plant’s reproduction can be severely affected.
  • Biodiversity and Ecosystem Health: The coevolution of plants and their pollinators is crucial for maintaining biodiversity and ecosystem health. The loss of a pollinator species can have cascading effects on the ecosystem.

Consider the following:

  1. Butterflies
  2. Fish
  3. Frogs

How many of the above have poisonous species among them?

(a) Only one
(b) Only two
(c) All three
(d) None

The correct answer is (c) All three.

Explanation:

  • Butterflies:
    • Many butterflies are poisonous. They ingest toxic chemicals from their host plants as caterpillars and retain these toxins into adulthood. For example, the monarch butterfly is known for its toxicity, which it acquires from milkweed plants.
  • Fish:
    • There are numerous poisonous fish species. For instance, species of puffer fish contain tetrodotoxin, a potent toxin that can be fatal if ingested. Other examples include the spotted trunkfish and certain moray eels.
  • Frogs:
    • Many frogs are poisonous, such as the poison dart frogs. These frogs secrete toxins through their skin, which can be harmful or even lethal if ingested. Additionally, some frogs like Bruno’s casque-headed frog and Greening’s frog are also venomous, meaning they can inject toxins through spines on their heads.

Learn more

  • Butterflies:
    • Butterflies often acquire their toxicity from the plants they consume as caterpillars.
    • Aposematism is a common defense mechanism where bright colors warn predators of their toxicity.
    • Some non-toxic butterflies mimic the appearance of toxic species to avoid predation.
  • Fish:
    • Poisonous fish contain toxins that are harmful when ingested. Examples include puffer fish and moray eels.
    • Venomous fish deliver toxins through bites, stings, or spines. Examples include the reef stonefish and lionfish.
    • Venomous fish are more numerous than venomous snakes and are found in various habitats, primarily tropical waters.
  • Frogs:
    • Poisonous frogs secrete toxins through their skin as a defense mechanism. Poison dart frogs are a well-known example.
    • Venomous frogs like Bruno’s casque-headed frog and Greening’s frog can inject toxins through spines on their heads.
    • The venom of these frogs can cause severe pain and other symptoms in predators and humans.

Consider the following:

  1. Cashew
  2. Papaya
  3. Red sanders

How many of the above trees are actually native to India?

(a) Only one
(b) Only two
(c) All three
(d) None

Correct Answer: (a) Only one

Explanation:

  • Cashew: The cashew tree (Anacardium occidentale) is native to Brazil. It was introduced to India by Portuguese colonists in the late 16th century.
  • Papaya: The papaya tree (Carica papaya) is native to southern Mexico and Central America. It was later spread to other tropical regions, including India.
  • Red Sanders: The Red Sanders tree (Pterocarpus santalinus), also known as Red Sandalwood, is native to India, specifically the southern parts of the Eastern Ghats in Andhra Pradesh.

Thus, only one of the trees listed is native to India.

Learn more

  • Cashew Tree:
    • Scientific NameAnacardium occidentale
    • Origin: Native to northeastern Brazil.
    • Introduction to India: Brought by Portuguese colonists in the late 16th century.
    • Uses: Cashew nuts are used in cooking and as snacks; cashew apples are used in beverages and jams. The tree also provides wood and resin for various industrial uses.
    • Cultivation: Grows in tropical climates with well-distributed rainfall. Major producers include India, Vietnam, and Brazil.
  • Papaya Tree:
    • Scientific NameCarica papaya
    • Origin: Native to southern Mexico and Central America.
    • Introduction to India: Spread to India through historical trade routes.
    • Uses: Papaya fruit is consumed fresh, in juices, and in various culinary dishes. It is rich in vitamins and enzymes beneficial for digestion.
    • Cultivation: Grows in tropical and subtropical climates. Major producers include India, Brazil, and Mexico.
  • Red Sanders Tree:
    • Scientific NamePterocarpus santalinus
    • Origin: Native to the southern Eastern Ghats of India.
    • Uses: The wood is highly valued for its rich red color and is used in furniture, musical instruments, and traditional medicine. The tree also produces a red dye.
    • Conservation Status: Listed as endangered due to over-exploitation and habitat loss. Protected under various national and international regulations.

With reference to radioisotope thermoelectric generators (RTGs), consider the following statements:

  1. RTGs are miniature fission reactors.
  2. RTGs are used for powering the onboard systems of spacecrafts.
  3. RTGs can use Plutonium-238, which is a by-product of weapons development.

Which of the statements given above are correct?

(a) 1 and 2 only
(b) 2 and 3 only
(c) 1 and 3 only
(d) 1, 2 and 3

Correct Answer: (b) 2 and 3 only

Explanation:

  • Statement 1: RTGs are miniature fission reactors.
    • This statement is incorrect. RTGs are not fission reactors. They generate power through the natural radioactive decay of isotopes like plutonium-238, not through a fission chain reaction.
  • Statement 2: RTGs are used for powering the onboard systems of spacecrafts.
    • This statement is correct. RTGs have been used extensively to power the onboard systems of various spacecraft, including missions like Cassini, Voyager, and New Horizons.
  • Statement 3: RTGs can use Plutonium-238, which is a by-product of weapons development.
    • This statement is correct. Plutonium-238 is produced by irradiating neptunium-237, which can be recovered from research reactor fuel or special targets, and has been used in RTGs for space missions.

Learn more

  • Definition and Functionality:
    • Radioisotope Thermoelectric Generators (RTGs) are devices that convert heat released by the decay of radioactive materials into electricity using thermocouples. They are highly reliable and have no moving parts, making them ideal for long-duration missions in harsh environments.
  • Applications:
    • RTGs are primarily used in space missions where solar power is impractical. They have powered numerous missions, including the Apollo lunar landings, Viking Mars landers, Voyager probes, and the Cassini mission to Saturn.
  • Isotopes Used:
    • The most common isotope used in RTGs is plutonium-238 due to its high decay heat and relatively low gamma radiation, which requires minimal shielding. Americium-241 is another isotope used, particularly by the European Space Agency.
  • Safety and Design:
    • RTGs are designed with multiple safety features, including heat-resistant ceramic fuel forms, iridium capsules, and graphite blocks to contain the fuel and prevent its release in case of accidents. These features ensure that RTGs can withstand severe physical conditions.
  • Historical Context:
    • The first RTG was launched in 1961 aboard the Navy TRANSIT 4A Navigational Satellite. Since then, RTGs have been a crucial power source for many NASA missions, providing reliable power for decades.
  • Production Challenges:
    • The production of plutonium-238 is complex and was halted in the late 1980s. However, production resumed in 2015 at Oak Ridge National Laboratory to meet the needs of future NASA missions.

Consider the following statements:

Statement-I: Giant stars live much longer than dwarf stars.

Statement-II: Compared to dwarf stars, giant stars have a greater rate of nuclear reactions.

Which one of the following is correct in respect of the above statements?

(a) Both Statement-I and Statement-II are correct and Statement-II explains Statement-I
(b) Both Statement-I and Statement-II are correct, but Statement-II does not explain Statement-I
(c) Statement-I is correct, but Statement-II is incorrect
(d) Statement-I is incorrect, but Statement-II is correct

Correct Answer: (d) Statement-I is incorrect, but Statement-II is correct

Explanation:

  • Statement-I: Giant stars live much longer than dwarf stars.
    • This statement is incorrectGiant stars have shorter lifespans compared to dwarf stars. Giant stars, due to their massive size and higher core temperatures, burn through their nuclear fuel much faster, resulting in shorter lifetimes. For example, a star with 10 times the mass of the Sun may only live for about 20 million years, whereas a red dwarf, which is much smaller, can last for 80 to 100 billion years, far exceeding the current age of the universe.
  • Statement-II: Compared to dwarf stars, giant stars have a greater rate of nuclear reactions.
    • This statement is correctGiant stars have hotter and denser cores, leading to a greater rate of nuclear reactions. This increased rate of nuclear fusion causes them to burn through their fuel more quickly than dwarf stars. For instance, a giant star can be thousands of times more luminous than the Sun due to its rapid nuclear reactions.

Analysis of Options:

  • (a) Both Statement-I and Statement-II are correct and Statement-II explains Statement-I
    • This option is incorrect because Statement-I is incorrect.
  • (b) Both Statement-I and Statement-II are correct, but Statement-II does not explain Statement-I
    • This option is incorrect because Statement-I is incorrect.
  • (c) Statement-I is correct, but Statement-II is incorrect
    • This option is incorrect because Statement-I is incorrect and Statement-II is correct.
  • (d) Statement-I is incorrect, but Statement-II is correct
    • This option is correct because Statement-I is incorrect and Statement-II is correct.

Learn more

  • Lifespan of Stars:
    • Massive stars have shorter lifespans due to their rapid consumption of nuclear fuel. For example, stars with masses 10 times that of the Sun may only live for a few million years.
    • Dwarf stars, especially red dwarfs, have longer lifespans. They burn their fuel slowly and can last for tens to hundreds of billions of years, far exceeding the current age of the universe.
  • Nuclear Reactions in Stars:
    • Giant stars have higher core temperatures and greater rates of nuclear reactions. This results in higher luminosity and faster fuel consumption.
    • Dwarf stars have lower core temperatures and slower rates of nuclear reactions, leading to longer lifespans.
  • Types of Stars:
    • Giant stars include red giantsblue giants, and supergiants. They are characterized by their large radii and high luminosities.
    • Dwarf stars include red dwarfsyellow dwarfs (like the Sun), and white dwarfs. They are smaller and less luminous compared to giant stars.
  • Stellar Evolution:
    • Massive stars end their lives in supernova explosions, leaving behind neutron stars or black holes.
    • Smaller stars like the Sun will become red giants and eventually shed their outer layers, leaving behind a white dwarf.

Which one of the following is synthesized in the human body that dilates blood vessels and increases blood flow?

(a) Nitric oxide
(b) Nitrous oxide
(c) Nitrogen dioxide
(d) Nitrogen pentoxide

Correct Answer: (a) Nitric oxide

Explanation:

  • Nitric oxide (NO) is synthesized in the human body and acts as a potent vasodilator. It is produced by endothelial cells lining the blood vessels and plays a crucial role in regulating vascular tone and blood flow by relaxing the smooth muscle cells in the vessel walls, leading to vasodilation and increased blood flow.
  • Nitrous oxide (N2O), commonly known as laughing gas, is an anesthetic used in medical settings. It does not have a significant role in vasodilation or blood flow regulation in the human body.
  • Nitrogen dioxide (NO2) is a toxic gas that can cause respiratory issues. It is not involved in the physiological process of vasodilation or blood flow regulation.
  • Nitrogen pentoxide (N2O5) is a chemical compound used in industrial processes and is not synthesized in the human body. It does not play any role in vasodilation or blood flow regulation.

Learn more

  • Vasodilation is the process of widening blood vessels due to the relaxation of smooth muscle cells in the vessel walls. This process increases blood flow and decreases blood pressure.
  • Mechanism of Action:
    • Nitric oxide (NO) is produced from the amino acid L-arginine by the enzyme endothelial nitric oxide synthase (eNOS).
    • NO diffuses into the smooth muscle cells of blood vessels and activates soluble guanylate cyclase (sGC).
    • This activation increases the production of cyclic guanosine monophosphate (cGMP), which leads to the relaxation of smooth muscle cells and vasodilation.
  • Sources of NO:
    • NO is produced endogenously by endothelial cells.
    • It can also be introduced into the body through certain foods and supplements, such as those high in nitrates (e.g., beetroot).
  • Physiological Roles:
    • Regulates blood pressure: By inducing vasodilation, NO helps maintain proper blood pressure levels.
    • Enhances blood flow: Increased blood flow ensures efficient delivery of oxygen and nutrients to tissues and organs.
    • Supports cardiovascular health: Adequate NO levels are crucial for preventing hypertension and other cardiovascular diseases.
  • Factors Influencing NO Production:
    • Exercise: Physical activity stimulates NO production, improving endothelial function and vascular health.
    • Diet: Consuming nitrate-rich vegetables can boost NO levels in the body.
    • Age and health conditions: Reduced NO production is associated with aging and certain health conditions like diabetes and heart disease.
  • Clinical Applications:
    • Medications: Drugs like nitroglycerin and sildenafil (Viagra) work by enhancing NO pathways to treat conditions like angina and erectile dysfunction.
    • Therapeutic interventions: NO donors and vasodilators are used to manage hypertension and other cardiovascular conditions.

Consider the following activities:

  1. Identification of narcotics on passengers at airports or in aircraft
  2. Monitoring of precipitation
  3. Tracking the migration of animals

In how many of the above activities can the radars be used?

(a) Only one
(b) Only two
(c) All three
(d) None

Correct Answer: (b) Only two

  • Identification of narcotics on passengers at airports or in aircraft: Radars are not typically used for this purpose. Instead, specialized technologies such as X-ray scanners, chemical trace-detection techniques, and behavioral analysis are employed for identifying narcotics on passengers.
  • Monitoring of precipitation: Radars are extensively used for monitoring precipitation. Weather radars, including Doppler weather radars, are designed to locate precipitation, calculate its motion, and estimate its type (rain, snow, hail, etc.). These radars provide high spatial and temporal resolution data, which is crucial for hydrological and meteorological applications.
  • Tracking the migration of animals: Radars are used to track the migration of animals, including birds, bats, and insects. Advanced radar systems can distinguish between different biological targets and provide valuable data on the timing, spatial distribution, and behavioral patterns of migrating animals.

Learn more

  • Radar Technology: Radar stands for Radio Detection and Ranging. It works by emitting a pulse of radio waves that bounce off objects and return to the radar system, allowing it to detect and locate objects.
  • Weather Radars: These radars are used to monitor and predict weather conditions. They can detect precipitation, its intensity, and movement. Doppler weather radars can also measure the velocity of precipitation particles, providing insights into storm dynamics and potential severe weather events.
  • Animal Migration Tracking: Radars have been used since the 1940s to study the aerial movements of animals. Modern radar systems can track large-scale movements of birds, bats, and insects, providing data on migration patterns, altitude preferences, and responses to environmental conditions. This information is crucial for conservation efforts and understanding ecological dynamics.
  • Airport Surveillance Radars: These radars are used to detect and display the presence and position of aircraft in the airspace around airports. They ensure safe and orderly flow of air traffic by providing real-time data to air traffic controllers.
  • Limitations of Radar: While radars are versatile, they have limitations. For instance, primary radars cannot distinguish between different types of objects, and secondary radars require transponders on aircraft to provide additional information. Radars also face challenges in detecting small or low-reflectivity objects.

Consider the following aircraft:

  1. Rafael
  2. MiG-29
  3. Tejas MK-1

How many of the above are considered fifth generation fighter aircraft?

(a) Only one
(b) Only two
(c) All three
(d) None

Correct Answer: (d) None

  • Rafale: The Dassault Rafale is a 4.5-generation fighter aircraft. It incorporates advanced avionics, AESA radar, and multirole capabilities but lacks the full suite of fifth-generation features such as stealth and supercruise performance.
  • MiG-29: The Mikoyan MiG-29 is a fourth-generation fighter aircraft. It was designed in the 1970s and entered service in the 1980s. While it has been upgraded over the years, it does not meet the criteria for fifth-generation fighters.
  • Tejas MK-1: The HAL Tejas is classified as a 4.5-generation fighter aircraft. It features advanced avionics and multirole capabilities but does not possess the stealth and other advanced features of fifth-generation fighters.

Learn more

  • Fifth-generation fighter aircraft are the most advanced fighters in operation as of the 21st century. They typically include:
    • Stealth: Designed to be less visible to radar and other detection methods.
    • Low-probability-of-intercept radar (LPIR): Advanced radar systems that are difficult for enemies to detect.
    • Agile airframes with supercruise performance: Ability to cruise at supersonic speeds without afterburners.
    • Advanced avionics: Integrated systems for enhanced situational awareness and battlefield management.
    • Highly integrated computer systems: Capable of networking with other elements within the battlespace for command, control, and communications (C3) capabilities.
  • Examples of fifth-generation fighters include:
    • Lockheed Martin F-22 Raptor: Entered service with the USAF in 2005, known for its stealth, speed, and agility.
    • Lockheed Martin F-35 Lightning II: Entered service with the USMC in 2015, featuring advanced stealth and sensor fusion capabilities.
    • Chengdu J-20: Entered service with the PLAAF in 2017, incorporating stealth and advanced avionics.
    • Sukhoi Su-57: Entered service with the Russian Air Force in 2020, known for its supermaneuverability and stealth features.
  • Fourth and 4.5-generation fighters:
    • Fourth-generation fighters: Designed in the 1970s and 1980s, focusing on maneuverability and multirole capabilities. Examples include the F-15 Eagle and MiG-29.
    • 4.5-generation fighters: Upgraded versions of fourth-generation fighters with advanced avionics, AESA radar, and some stealth features. Examples include the Dassault Rafale, HAL Tejas MK-1, and Eurofighter Typhoon.

In which of the following are hydrogels used?

  1. Controlled drug delivery in patients
  2. Mobile air-conditioning systems
  3. Preparation of industrial lubricants

Select the correct answer using the code given below:

(a) 1 only
(b) 1 and 2 only
(c) 2 and 3 only
(d) 1, 2 and 3

Correct Answer: (d) 1, 2 and 3

Explanation:

  • Controlled drug delivery in patients:
    • Hydrogels are extensively used in controlled drug delivery systems due to their tunable properties, controllable degradation, and ability to protect labile drugs. They provide spatiotemporal control over drug release and are highly biocompatible, making them suitable for delivering a wide range of therapeutic agents, including small molecules, proteins, and nucleic acids.
  • Mobile air-conditioning systems:
    • Hydrogels have been proposed and investigated for use in passive thermal management systems. They can absorb moisture from the air and provide cooling through evaporative processes, similar to how human skin cools through sweating. This property makes them suitable for applications in cooling electronics and potentially in mobile air-conditioning systems.
  • Preparation of industrial lubricants:
    • Hydrogels are also used in the preparation of industrial lubricants. They can be fabricated with various organic and inorganic gelators to provide lubrication, reduce friction, and wear in mechanical systems. This makes them valuable in industrial applications where efficient lubrication is critical.

Learn more

  • Hydrogels are three-dimensional polymer networks capable of holding large amounts of water due to their hydrophilic structure. They can be synthesized from natural, synthetic, or semi-synthetic polymers and can be cross-linked physically or chemically.
  • Controlled drug delivery:
    • Hydrogels offer spatial and temporal control over drug release, protecting drugs from degradation and allowing for precise delivery to target sites.
    • They are used in various medical applications, including cancer treatment, wound healing, and tissue engineering.
  • Thermal management:
    • Hydrogels can absorb and release moisture, providing a passive cooling mechanism. This property is utilized in cooling high-power electronics and potentially in air-conditioning systems to reduce energy consumption.
  • Industrial lubricants:
    • Hydrogels reduce friction and wear in mechanical systems, making them suitable for use in industrial lubricants. They can be tailored to provide specific lubrication properties, enhancing the efficiency and lifespan of mechanical components.

Which one of the following is the exhaust pipe emission from Fuel Cell Electric Vehicles, powered by hydrogen?

(a) Hydrogen peroxide
(b) Hydronium
(c) Oxygen
(d) Water vapour

Correct Answer: (d) Water vapour

Explanation

  • (a) Hydrogen peroxide: This is not the correct emission from Fuel Cell Electric Vehicles (FCEVs). Hydrogen peroxide is not produced in the electrochemical reaction within the fuel cell.
  • (b) Hydronium: This is also incorrect. Hydronium ions are not emitted from the exhaust of FCEVs.
  • (c) Oxygen: While oxygen is involved in the reaction within the fuel cell, it is not emitted from the exhaust. Instead, it combines with hydrogen to form water.
  • (d) Water vapour: This is the correct answer. The only exhaust emission from FCEVs is water vapor, which is produced as a byproduct of the electrochemical reaction between hydrogen and oxygen within the fuel cell.

Learn More

Hydrogen Fuel Cell Electric Vehicles (FCEVs):

  • Power Source: FCEVs are powered by hydrogen, the most abundant element in the universe. Hydrogen is stored in high-pressure tanks within the vehicle.
  • How They Work:
    • Fuel Cell Stack: Hydrogen gas is fed into the fuel cell stack, where it undergoes an electrochemical reaction with oxygen from the air.
    • Electrochemical Reaction: In the fuel cell, hydrogen molecules split into protons and electrons. The electrons travel through an external circuit, generating electricity to power the electric motor. The protons move through the electrolyte membrane and combine with oxygen and electrons at the cathode to form water.
  • Emissions: The only byproduct of this reaction is water vapor, making FCEVs zero-emission vehicles.
  • Efficiency and Performance:
    • Energy Efficiency: FCEVs are more efficient than conventional internal combustion engine vehicles.
    • Refueling Time: They can be refueled in about 3-5 minutes, similar to conventional gasoline vehicles.
    • Driving Range: FCEVs typically have a driving range of over 300 miles on a full tank of hydrogen.
  • Environmental Impact: FCEVs produce no harmful tailpipe emissions, contributing to reduced air pollution and greenhouse gas emissions.
  • Challenges:
    • Infrastructure: The development of hydrogen refueling infrastructure is still in its early stages, limiting the widespread adoption of FCEVs.
    • Cost: Hydrogen production and storage are currently more expensive compared to other fuels, though costs are expected to decrease with technological advancements and increased production.

Recently, the term “pumped-storage hydropower” is actually and appropriately discussed in the context of which one of the following?

(a) Irrigation of terraced crop fields
(b) Lift irrigation of cereal crops
(c) Long duration energy storage
(d) Rainwater harvesting system

Correct Answer: (c) Long duration energy storage

Explanation:

  • Pumped-storage hydropower (PSH) is primarily discussed in the context of long duration energy storage. It stores energy by pumping water from a lower reservoir to a higher one when there is excess electricity, and releases it to generate power during high demand periods.

Analysis of other Options:

  • (a) Irrigation of terraced crop fields: PSH is not typically used for irrigation purposes. It is designed for energy storage and grid management, not for agricultural irrigation.
  • (b) Lift irrigation of cereal crops: Similar to the previous point, PSH is not used for lift irrigation. Its primary function is to store and generate electricity, not to pump water for crop irrigation.
  • (d) Rainwater harvesting system: While rainwater harvesting involves storing water, it is not the same as PSH. Rainwater harvesting is for water conservation and usage, whereas PSH is for energy storage and grid stability.

Learn more

  • DefinitionPumped-storage hydropower (PSH) is a type of hydroelectric energy storage that uses two water reservoirs at different elevations to store and generate electricity.
  • Working Principle:
    • Energy Storage: Water is pumped from a lower reservoir to an upper reservoir using surplus electricity during low demand periods.
    • Energy Generation: During high demand, water is released back to the lower reservoir through turbines, generating electricity.
  • Efficiency: The round-trip energy efficiency of PSH ranges from 70% to 80%, with some sources claiming up to 87%.
  • Global Usage: PSH accounts for over 90% of grid-scale energy storage capacity worldwide, with significant installations in the United States, China, and Japan.
  • Types:
    • Open-Loop PSH: Connected to natural water bodies.
    • Closed-Loop PSH: Independent of natural water bodies, relying on man-made reservoirs.
  • Historical Context: The first PSH systems were developed in the 1890s in Italy and Switzerland. The technology has evolved significantly since then, with modern systems incorporating advanced reversible turbines.
  • Applications:
    • Grid Stability: Balances supply and demand, especially with the integration of intermittent renewable energy sources like wind and solar.
    • Peak Shaving: Provides electricity during peak demand periods, reducing the need for additional power plants.
  • Environmental Impact: While PSH is a clean energy technology, it can have ecological impacts, particularly on local water bodies and ecosystems. Careful site selection and management are crucial.
  • Future Prospects: As renewable energy sources become more prevalent, the role of PSH in providing long-duration energy storage and grid stability is expected to grow.

“Membrane Bioreactors” are often discussed in the context of:

(a) Assisted reproductive technologies
(b) Drug delivery nanotechnologies
(c) Vaccine production technologies
(d) Wastewater treatment technologies

Correct Answer: (d) Wastewater treatment technologies.

Explanation:

  • Membrane Bioreactors (MBRs) are primarily used in wastewater treatment technologies. They combine biological treatment and membrane filtration to treat municipal and industrial wastewater, providing high-quality effluent and reducing plant footprint.

Other Options:

  • (a) Assisted reproductive technologies: MBRs are not related to assisted reproductive technologies. These technologies typically involve medical procedures such as in vitro fertilization (IVF) and do not use membrane bioreactors.
  • (b) Drug delivery nanotechnologies: While nanotechnology is used in drug delivery systems, MBRs are not part of this field. Drug delivery systems focus on targeted delivery of pharmaceuticals using nanoparticles, which is unrelated to the wastewater treatment processes of MBRs.
  • (c) Vaccine production technologies: Although bioreactors are used in vaccine production, MBRs specifically refer to wastewater treatment processes. Vaccine production involves different types of bioreactors designed for cell culture and virus amplification, not the membrane filtration used in MBRs.

Learn more

  • DefinitionMembrane Bioreactors (MBRs) integrate biological degradation of waste with membrane filtration to treat wastewater efficiently.
  • Components:
    • Bioreactor: Supports a biologically active environment where microorganisms degrade pollutants.
    • Membrane Filtration Unit: Separates solids from liquids, typically using microfiltration (MF) or ultrafiltration (UF) membranes.
  • Configurations:
    • Submerged MBR: Membranes are submerged in the bioreactor.
    • Sidestream MBR: Membranes are located outside the bioreactor.
  • Advantages:
    • High-quality effluent: Produces clear, pathogen-free water.
    • Compact design: Requires less space compared to conventional systems.
    • Independent control: Allows separate control of solids retention time (SRT) and hydraulic retention time (HRT).
  • Challenges:
    • Membrane fouling: A significant issue that reduces performance and increases maintenance costs.
    • High energy costs: Due to the need for aeration and membrane cleaning.
  • Applications:
    • Municipal wastewater treatment: Commonly used in urban areas to treat sewage.
    • Industrial wastewater treatment: Effective for treating wastewater from various industries, including food processing and pharmaceuticals.
  • Innovations:
    • Nanomaterials: Incorporation of nanomaterials in membranes to enhance performance and reduce fouling.
    • Aerobic granulation: Combining aerobic granules with MBRs for improved filtration and nutrient removal.

With reference to the Indian economy, “Collateral Borrowing and Lending Obligations” are the instruments of:

(a) Bond market
(b) Forex market
(c) Money market
(d) Stock market

Correct Answer: (c) Money market

  • Collateral Borrowing and Lending Obligations (CBLO) are instruments of the money market. They are used for short-term borrowing and lending, typically ranging from one day to one year, and are operated by the Clearing Corporation of India Ltd (CCIL).

Analysis of Options:

  • Bond market: This market deals with debt securities, including government and corporate bonds, which are used to raise long-term capital. Bonds provide fixed returns and are considered safer investments compared to equities.
  • Forex market: The foreign exchange market is a global decentralized market for trading currencies. It involves the buying, selling, and exchanging of currencies at current or determined prices and is the largest financial market in the world.
  • Money market: This market deals with short-term borrowing and lending, typically for periods up to one year. Instruments in this market include Treasury bills, commercial paper, and CBLO. The money market provides liquidity to financial institutions and helps in managing short-term funding needs.
  • Stock market: This market involves the buying and selling of shares of publicly traded companies. It includes various financial instruments such as stocks, derivatives, bonds, and mutual funds. The stock market is known for its volatility and potential for high returns.

Learn more

  • DefinitionCollateral Borrowing and Lending Obligations (CBLO) are short-term money market instruments that allow financial entities to borrow and lend funds against the collateral of eligible securities, primarily government securities.
  • Participants: Entities that participate in the CBLO market include nationalized banks, private banks, foreign banks, cooperative banks, insurance companies, mutual funds, primary dealers, non-banking financial companies (NBFCs), and corporate entities.
  • Functioning: The CBLO market operates through the Indian Financial Network (INFINET) and the Negotiated Dealing System (NDS). Borrowers provide collateral in the form of government securities, and the CCIL matches borrowing and lending orders submitted by members.
  • Regulation: The Reserve Bank of India (RBI) promotes collateralized borrowing and lending operations. CBLO transactions are subject to certain regulatory requirements, including the maintenance of Cash Reserve Ratio (CRR) and Statutory Liquidity Ratio (SLR) by scheduled commercial banks.
  • Benefits: CBLO provides a secure and efficient mechanism for managing short-term liquidity needs. It offers flexibility in terms of borrowing and lending periods and helps in maintaining financial stability by providing a reliable source of short-term funding.

The total fertility rate in an economy is defined as:

(a) the number of children born per 1000 people in the population in a year.

(b) the number of children born to a couple in their lifetime in a given population.

(c) the birth rate minus death rate.

(d) the average number of live births a woman would have by the end of her child-bearing age.

The correct answer is: (d) the average number of live births a woman would have by the end of her child-bearing age.

Explanation

  • (a) the number of children born per 1000 people in the population in a year:
    • This describes the birth rate, not the total fertility rate. The birth rate is the number of live births per 1,000 people in a given period.
  • (b) the number of children born to a couple in their lifetime in a given population:
    • This is not the definition of the total fertility rate. It might refer to the mean number of children ever born per woman or average family size.
  • (c) the birth rate minus death rate:
    • This describes the rate of natural increase or population index, which is the difference between the birth rate and the death rate.
  • (d) the average number of live births a woman would have by the end of her child-bearing age:
    • This is the correct definition of the total fertility rate (TFR). It represents the average number of children a woman would have if she experienced the current age-specific fertility rates throughout her childbearing years.

Learn more:

  • Total Fertility Rate (TFR):
    • Definition: The TFR is the average number of children a woman would have if she lived through her childbearing years and experienced the current age-specific fertility rates.
    • Calculation: It is calculated by summing the age-specific fertility rates over five-year intervals.
    • Global Variations: As of 2023, TFR varies widely, from 0.72 in South Korea to 6.73 in Niger.
    • Economic Correlation: TFR tends to be inversely correlated with economic development. Developed countries generally have lower fertility rates due to factors like wealth, education, and urbanization, while least developed countries have higher rates due to labor needs, lack of contraceptives, and lower female education and employment.
    • Historical Trends: The global TFR has declined rapidly since the 1960s. Projections indicate a transition from long-term growth to decline between 2050 and 2070, with the UN predicting a global TFR of 1.8 by 2100.
    • Importance: TFR is a crucial demographic indicator for understanding population growth and its implications on economic and social development.

Consider the following statements:

  1. In India, Non-Banking Financial Companies can access the Liquidity Adjustment Facility window of the Reserve Bank of India.
  2. In India, Foreign Institutional Investors can hold the Government Securities (G-Secs).
  3. In India, Stock Exchanges can offer separate trading platforms for debts.

Which of the statements given above is/are correct?

(a) 1 and 2 only
(b) 3 only
(c) 1, 2 and 3
(d) 2 and 3 only

Correct Answer: (d) 2 and 3 only

  • Statement 1Incorrect. In India, Non-Banking Financial Companies (NBFCs) cannot directly access the Liquidity Adjustment Facility (LAF) window of the Reserve Bank of India (RBI). Unlike banks, NBFCs do not have large amounts of government securities to offer as collateral, which limits their ability to borrow from the LAF window.
  • Statement 2Correct. In India, Foreign Institutional Investors (FIIs) are allowed to hold Government Securities (G-Secs). FIIs can invest in dated Government Securities and Commercial papers of Indian establishments.
  • Statement 3CorrectStock Exchanges in India can offer separate trading platforms for debts. For instance, the National Stock Exchange (NSE) has unveiled India’s first debt trading platform.

Learn more

  • Non-Banking Financial Companies (NBFCs):
    • NBFCs are financial institutions that provide banking services without meeting the legal definition of a bank.
    • They cannot directly access the RBI’s LAF window due to limited government securities for collateral.
    • NBFCs are required to maintain a Statutory Liquidity Ratio (SLR) of 15% of their outstanding public deposits.
  • Foreign Institutional Investors (FIIs):
    • FIIs include entities like insurance companies, pension funds, and sovereign wealth funds.
    • They can invest in various Indian securities, including government securities, corporate bonds, and commercial papers.
    • FIIs help diversify the investment pool and bring in foreign capital, which is crucial for funding projects like India’s green bonds.
  • Debt Trading Platforms:
    • Stock exchanges like the NSE and BSE offer separate trading platforms for debt instruments.
    • These platforms facilitate the trading of various debt securities, including government securities, corporate bonds, and other fixed-income instruments.
    • The introduction of these platforms aims to enhance liquidity and transparency in the debt market.

In India, which of the following can trade in Corporate Bonds and Government Securities?

  1. Insurance Companies
  2. Pension Funds
  3. Retail Investors

Select the correct answer using the code given below:

(a) 1 and 2 only
(b) 2 and 3 only
(c) 1 and 3 only
(d) 1, 2 and 3

Correct Answer: (d) 1, 2 and 3

Explanation:

  • Insurance Companies: Insurance companies in India are active participants in the bond market. They invest in both corporate bonds and government securities to meet their investment requirements and regulatory obligations. They are significant players in the government bond market, often holding a substantial portion of government securities.
  • Pension Funds: Pension funds also invest in corporate bonds and government securities. They are allowed to invest in these instruments to ensure stable and long-term returns for their beneficiaries. The Pension Fund Regulatory and Development Authority (PFRDA) provides guidelines for such investments.
  • Retail Investors: Retail investors can trade in government securities and corporate bonds. The Reserve Bank of India (RBI) has made it easier for retail investors to participate in the bond market through platforms like RBI Retail Direct, which allows them to invest in government securities. They can also invest in corporate bonds through brokers and online platforms.

Learn more

  • Government Securities (G-Secs):
    • Definition: Tradeable instruments issued by the Central or State Governments to acknowledge their debt obligations.
    • Types: Include Treasury Bills (T-bills), which are short-term instruments, and Government Bonds, which are long-term instruments.
    • Risk: Considered risk-free due to the sovereign guarantee.
    • Investment: Institutional investors like banks, insurance companies, and pension funds are major holders, but retail investors can also participate through platforms like RBI Retail Direct.
  • Corporate Bonds:
    • Definition: Debt instruments issued by companies to raise funds for various purposes like expansion, project financing, and working capital.
    • Risk: Carry credit risk, which is the risk of default by the issuer. This risk is mitigated by investing in top-rated bonds.
    • Investment: Institutional investors like mutual funds, insurance companies, and pension funds are major players. Retail investors can also invest through brokers and online platforms.
  • Investment Guidelines:
    • Insurance Companies: Must invest a certain percentage of their assets in government securities and other approved investments as per regulatory requirements.
    • Pension Funds: Follow guidelines set by PFRDA, allowing investments in both government and corporate bonds to ensure stable returns.
    • Retail Investors: Can invest in government securities through RBI Retail Direct and in corporate bonds through brokers and online platforms. The minimum investment amount has been reduced to make it more accessible.

Consider the following:

  1. Exchange-Traded Funds (ETF)
  2. Motor vehicles
  3. Currency swap

Which of the above is/are considered financial instruments?

(a) 1 only
(b) 2 and 3 only
(c) 1, 2 and 3
(d) 1 and 3 only

Correct Answer: (d) 1 and 3 only

Explanation:

  • Option 1: Exchange-Traded Funds (ETF)
    • ETFs are considered financial instruments because they are investment funds that hold a collection of assets such as stocks, bonds, or commodities and are traded on stock exchanges. They provide diversification and can be bought and sold like stocks.
  • Option 2: Motor vehicles
    • Motor vehicles are not considered financial instruments. They are tangible assets that depreciate over time and do not represent a financial agreement or monetary value in the context of financial markets.
  • Option 3: Currency swap
    • Currency swaps are considered financial instruments. They are derivatives used by banks, multinational corporations, and institutional investors to exchange principal and interest payments in different currencies, helping to manage foreign exchange risk and borrowing costs.

Learn more

Exchange-Traded Funds (ETF)

  • Definition: An ETF is a type of investment fund that holds a collection of assets such as stocks, bonds, or commodities and is traded on stock exchanges.
  • Types: Equity ETFs, Bond ETFs, Commodity ETFs, Sectoral/Thematic ETFs, International ETFs.
  • Advantages: Diversification, lower fees, ease of trading, transparency.
  • Disadvantages: Market risk, trading costs, potential tracking errors.
  • Market Size: In the US, $5.4 trillion in equity ETFs and $1.4 trillion in fixed-income ETFs.

Motor Vehicles

  • Definition: Tangible assets that depreciate over time.
  • Accounting: Considered depreciating assets, not financial instruments.
  • Value: Depreciates due to factors like usage, design, repair costs, and market conditions.

Currency Swap

  • Definition: A financial instrument where two parties exchange principal and interest payments in different currencies.
  • Purpose: To manage foreign exchange risk and reduce borrowing costs.
  • Types: Floating vs. Floating, Fixed vs. Floating, Fixed vs. Fixed, Non-deliverable swaps.
  • Advantages: Lower borrowing costs, hedging against exchange rate fluctuations, customized terms.
  • Usage: Widely used by banks, multinational corporations, and institutional investors.

With reference to the sectors of the Indian economy, consider the following pairs:

Economic activitySector
1. Storage of agricultural produceSecondary
2. Dairy farmPrimary
3. Mineral explorationTertiary
4. Weaving clothSecondary

How many of the pairs given above are correctly matched?
(a) Only one
(b) Only two
(c) Only three
(d) All four

The correct answer is (b) Only two.

Explanation:

  • Pair 1: Storage of agricultural produce – Secondary
    • Incorrect. Storage of agricultural produce falls under the tertiary sector as it involves services related to the storage and preservation of goods, which support the primary and secondary sectors.
  • Pair 2: Dairy farm – Primary
    • Correct. Dairy farming is part of the primary sector as it involves the extraction and collection of natural resources, specifically milk from animals.
  • Pair 3: Mineral exploration – Tertiary
    • Incorrect. Mineral exploration is part of the primary sector as it involves the extraction of natural resources from the earth.
  • Pair 4: Weaving cloth – Secondary
    • Correct. Weaving cloth is part of the secondary sector as it involves the transformation of raw materials (yarn) into finished goods (cloth).

Learn more

  • Primary Sector:
    • Involves the extraction and harvesting of natural resources.
    • Examples include agriculture, mining, fishing, forestry, and dairy farming.
    • Contributes significantly to the GDP in many developing countries.
    • Faces issues like underemployment and disguised employment.
  • Secondary Sector:
    • Involves the transformation of raw materials into finished goods.
    • Includes manufacturing, construction, and industries like weaving cloth.
    • Adds value to products and is crucial for economic development.

Consider the following materials:

  1. Agricultural residues
  2. Corn grain
  3. Wastewater treatment sludge
  4. Wood mill waste

Which of the above can be used as feedstock for producing Sustainable Aviation Fuel?
(a) 1 and 2 only
(b) 3 and 4 only
(c) 1, 2, 3 and 4
(d) 1, 3 and 4 only

The correct answer is (c) 1, 2, 3 and 4.

  • Agricultural residues: These include materials like straw and corn stover, which are recognized as viable feedstocks for producing Sustainable Aviation Fuel (SAF).
  • Corn grain: Corn grain is a commonly used feedstock for SAF production, particularly through the alcohol-to-jet (AtJ) pathway.
  • Wastewater treatment sludge: This material can be used as a feedstock for SAF, as demonstrated by projects converting sewage sludge into biofuels.
  • Wood mill waste: Wood mill waste, including residues from wood processing, is also a recognized feedstock for SAF production.

Learn more

Sustainable Aviation Fuel (SAF)

  • Definition: SAF is a biofuel used to power aircraft, offering similar properties to conventional jet fuel but with a significantly smaller carbon footprint.
  • Environmental Impact: SAF can reduce life cycle greenhouse gas (GHG) emissions by up to 94% compared to conventional jet fuel, depending on the feedstock and production technology used.
  • Feedstocks: SAF can be produced from a variety of renewable and waste resources, including:
    • Agricultural residues: Such as straw and corn stover, which are by-products of crop production.
    • Corn grain: Used in the alcohol-to-jet (AtJ) pathway, converting sugary and starchy biomass into ethanol, which is then processed into jet fuel.
    • Wastewater treatment sludge: Sewage sludge can be converted into SAF through processes like anaerobic digestion and hydrothermal liquefaction.
    • Wood mill waste: Includes residues from wood processing, which can be converted into biofuels through methods like gasification and Fischer-Tropsch synthesis.
  • Economic and Environmental Benefits:
    • Economic Opportunities: SAF production can create new revenue streams for farmers and support job creation in feedstock production, biorefinery construction, and aviation.
    • Environmental Services: Growing biomass crops for SAF can improve soil quality, reduce erosion, and enhance water quality. Using waste materials for SAF reduces pollution and methane emissions.
  • Challenges and Future Prospects:
    • Feedstock Availability: Ensuring a sustainable and sufficient supply of feedstocks is crucial for scaling up SAF production.
    • Technological Advancements: Continued research and development are needed to improve production pathways and reduce costs.
    • Policy Support: Government policies and incentives play a vital role in promoting SAF adoption and scaling up production.

With reference to physical capital in Indian economy, consider the following pairs:

ItemsCategory
1. Farmer’s ploughWorking capital
2. ComputerFixed capital
3. Yarn used by the weaverFixed capital
4. PetrolWorking capital

How many of the above pairs are correctly matched?

(a) Only one

(b) Only two

(c) Only three

(d) All four

Correct Answer: (b) Only two

Explanation:

  • Pair 1: Farmer’s plough – Working capital
    • Incorrect. A farmer’s plough is an example of fixed capital. It is a tool that can be used over many years in the production process and is not consumed in a single production cycle.
  • Pair 2: Computer – Fixed capital
    • Correct. A computer is considered fixed capital as it is a long-term asset used in the production process over multiple years.
  • Pair 3: Yarn used by the weaver – Fixed capital
    • Incorrect. Yarn used by a weaver is an example of working capital. It is a raw material that is consumed in the production process.
  • Pair 4: Petrol – Working capital
    • Correct. Petrol is considered working capital as it is consumed during the production process and needs to be replenished regularly.

Learn more:

Physical Capital in the Indian Economy

  • Definition: Physical capital refers to the variety of inputs required at every stage during production. It includes both fixed capital and working capital.
  • Fixed Capital:
    • Nature: Long-term assets used in production over many years.
    • Examples: Machinery, buildings, tools, computers, and vehicles.
    • Characteristics: Not easily converted to cash, used repeatedly in production, and depreciates over time.
    • Importance: Essential for establishing and maintaining the production capacity of a business.
  • Working Capital:
    • Nature: Short-term assets used in day-to-day operations.
    • Examples: Raw materials, cash, inventory, and fuel.
    • Characteristics: Highly liquid, consumed in the production process, and needs regular replenishment.
    • Importance: Crucial for maintaining the smooth operation of a business and meeting short-term financial obligations.
  • Role in Production:
    • Fixed Capital: Provides the necessary infrastructure and equipment for production.
    • Working Capital: Ensures the availability of necessary materials and liquidity to keep the production process running smoothly.
  • Economic Impact:
    • Fixed Capital: Investments in fixed capital can lead to increased production capacity and long-term growth.
    • Working Capital: Adequate working capital is essential for operational efficiency and financial stability.

Which one of the following words/phrases is most appropriately used to denote “an interoperable network of 3D virtual worlds that can be accessed simultaneously by millions of users, who can exert property rights over virtual items”?

(a) Big data analytics

(b) Cryptography

(c) Metaverse

(d) Virtual matrix

The correct answer is (c) Metaverse.

  • Metaverse: The term “metaverse” refers to an interoperable network of 3D virtual worlds that can be accessed simultaneously by millions of users, who can exert property rights over virtual items.

Explanation of other options

  • (a) Big data analytics:
    • Definition: Big data analytics involves methods, tools, and applications used to collect, process, and derive insights from varied, high-volume, high-velocity data sets.
    • Usage: It is primarily used for analyzing large data sets to uncover hidden patterns, correlations, and market trends, not for creating or managing virtual worlds.
  • (b) Cryptography:
    • Definition: Cryptography is the process of hiding or coding information so that only the intended recipient can read it.
    • Usage: It is used for securing communications and data, ensuring confidentiality, integrity, authentication, and non-repudiation, but not for creating virtual worlds.
  • (d) Virtual matrix:
    • Definition: This term is not commonly used in the context of 3D virtual worlds or networks and does not fit the description provided.
    • Usage: It is not a recognized term in the context of interoperable networks of 3D virtual worlds.

Learn more:

Metaverse:

  • Definition: The metaverse is a vast, interconnected network of 3D virtual worlds where users can interact, own virtual items, and experience a sense of presence.
  • Key Features:
    • Virtual Worlds: Computer-generated environments that users can explore and interact with.
    • Three-Dimensional: The metaverse is inherently 3D, providing a more immersive experience than the traditional internet.
    • Real-Time Rendered: Virtual worlds respond to user inputs in real-time.
    • Massively Scaled: Capable of supporting millions of users simultaneously.
    • Interoperable Network: Users can carry personal information and virtual items across different virtual worlds.
    • Persistence: The metaverse is always on and accessible.
    • Synchronous: Users can share experiences in real-time.
    • Unlimited Users and Individual Presence: Supports millions of users, each with their own avatars and perspectives.
  • Technologies Involved:
    • Virtual Reality (VR): Provides a fully immersive experience through VR headsets and haptic feedback devices.
    • Augmented Reality (AR): Overlays digital information on the real world, enhancing user interaction with their environment.
    • Blockchain and NFTs: Used for securing transactions and ownership of virtual items within the metaverse.
    • Cryptocurrencies: Often used as the medium of exchange within virtual economies.
  • Applications:
    • Gaming: Platforms like Roblox, Minecraft, and Fortnite are early examples of metaverse-like experiences.
    • Social Interaction: Users can meet, socialize, and collaborate in virtual spaces.
    • Commerce: Virtual stores and marketplaces allow users to buy and sell virtual goods.
    • Education and Training: Virtual classrooms and training environments provide immersive learning experiences.
    • Work and Collaboration: Virtual offices and meeting spaces enable remote work and collaboration.

With reference to the rule/rules imposed by the Reserve Bank of India while treating foreign banks, consider the following statements:

  1. There is no minimum capital requirement for wholly owned banking subsidiaries in India.
  2. For wholly owned banking subsidiaries in India, at least 50% of the board members should be Indian nationals.

Which of the statements given above is/are correct?

(a) 1 only
(b) 2 only
(c) Both 1 and 2
(d) Neither 1 nor 2

Correct Answer: (b) 2 only

  • Statement 1Incorrect. There is a minimum capital requirement for wholly owned banking subsidiaries (WOS) in India. The initial minimum paid-up voting equity capital for a WOS is ₹5 billion.
  • Statement 2Correct. For wholly owned banking subsidiaries in India, at least 50% of the board members should be Indian nationals.

Learn more

  • Minimum Capital Requirement:
    • The Reserve Bank of India (RBI) mandates that the initial minimum paid-up voting equity capital for a wholly owned subsidiary (WOS) of a foreign bank in India is ₹5 billion.
    • This capital must be brought in upfront and funded by free foreign exchange remittance from the parent bank.
  • Board Member Nationality Requirements:
    • At least 50% of the directors of a WOS must be Indian nationals.
    • Additionally, not less than 50% of the directors should be non-executive directors, and a minimum of one-third of the directors should be independent of the management of the subsidiary in India, its parent, or associates.
  • Corporate Governance:
    • The WOS must comply with the corporate governance norms set by the RBI, which include having a majority of independent directors on the board.
    • The directors must conform to the ‘Fit and Proper’ criteria as laid down in RBI’s guidelines.
  • Regulatory Compliance:
    • The WOS will be subject to the licensing requirements and conditions consistent with those for new private sector banks.
    • It will be governed by the provisions of the Companies Act, 1956, Banking Regulation Act, 1949, Reserve Bank of India Act, 1934, and other relevant statutes and directives issued by the RBI.
  • Branch Expansion:
    • The branch expansion guidelines applicable to domestic scheduled commercial banks will generally apply to WOSs of foreign banks.
    • However, they will require the RBI’s prior approval for opening branches at certain locations sensitive from the perspective of national security.
  • Priority Sector Lending:
    • WOSs are required to meet the priority sector lending targets similar to those applicable to domestic scheduled commercial banks, with an adequate transition period for existing foreign bank branches converting into WOS.

With reference to Corporate Social Responsibility (CSR) rules in India, consider the following statements:

  1. CSR rules specify that expenditures that benefit the company directly or its employees will not be considered as CSR activities.
  2. CSR rules do not specify minimum spending on CSR activities.

Which of the statements given above is/are correct?

(a) 1 only
(b) 2 only
(c) Both 1 and 2
(d) Neither 1 nor 2

Correct Answer: (a) 1 only

  • Statement 1: CSR rules specify that expenditures that benefit the company directly or its employees will not be considered as CSR activities. This is correct. According to the Companies (CSR Policy) Rules, 2014, any activity benefitting employees of the company is not considered as eligible CSR activity. The spirit behind CSR activities is to benefit the public at large and not to be discriminatory to any class of beneficiaries.
  • Statement 2: CSR rules do not specify minimum spending on CSR activities. This is incorrect. The Companies Act, 2013 mandates that companies meeting certain criteria must spend at least 2% of their average net profits made during the three immediately preceding financial years on CSR activities.

Learn more

  • Corporate Social Responsibility (CSR): CSR refers to the responsibility of companies to contribute positively to society. It involves companies taking actions that further social good beyond the interests of the firm and that which is required by law.
  • Legal Framework: The Companies Act, 2013, particularly Section 135, and the Companies (CSR Policy) Rules, 2014, provide the legal framework for CSR in India. These rules mandate that companies meeting certain financial thresholds must spend a minimum of 2% of their average net profits from the last three years on CSR activities.
  • Eligibility Criteria: Companies with a net worth of ₹500 crore or more, turnover of ₹1,000 crore or more, or a net profit of ₹5 crore or more during any financial year are required to comply with CSR provisions.
  • Exclusions: CSR activities do not include:
    • Activities undertaken in the normal course of business.
    • Activities that benefit only the employees of the company.
    • Contributions to political parties.
    • Activities undertaken outside India, except for training of Indian sports personnel.
  • Implementation: Companies can undertake CSR activities through:
    • Themselves.
    • Registered trusts, societies, or Section 8 companies.
    • Collaborations with other companies.
  • Reporting and Compliance: Companies must report their CSR activities in their annual reports and on their websites. Non-compliance can result in penalties, including fines and imprisonment for responsible officers.
  • Impact Assessment: Companies with significant CSR expenditures must conduct impact assessments of their CSR projects to ensure effectiveness and transparency.

Consider the following statements :

Statement-I :
If the United States of America (USA) were to default on its debt, holders of US Treasury Bonds will not be able to exercise their claims to receive payment.

Statement-II :
The USA Government debt is not backed by any hard assets, but only by the faith of the Government.

Which one of the following is correct in respect of the above statements ?
(a) Both Statement-I and Statement-II are correct and Statement-II explains Statement-I
(b) Both Statement-I and Statement-II are correct, but Statement-II does not explain Statement-I
(c) Statement-I is correct, but Statement-II is incorrect
(d) Statement-I is incorrect, but Statement-II is correct

Correct Answer: (b) Both Statement-I and Statement-II are correct, but Statement-II does not explain Statement-I

Explanation:

  • Option (a): Both Statement-I and Statement-II are correct and Statement-II explains Statement-I
    • Incorrect: While both statements are correct, Statement-II does not explain Statement-I. Statement-I is about the inability of bondholders to exercise their claims in the event of a default, while Statement-II discusses the nature of the backing of US government debt.
  • Option (b): Both Statement-I and Statement-II are correct, but Statement-II does not explain Statement-I
    • Correct: Statement-I is correct because if the USA defaults, holders of US Treasury Bonds will not be able to exercise their claims to receive payment. Statement-II is also correct as the US government debt is backed by the faith and credit of the government, not by hard assets. However, Statement-II does not explain Statement-I.
  • Option (c): Statement-I is correct, but Statement-II is incorrect
    • Incorrect: Statement-II is correct as the US government debt is indeed backed by the faith of the government and not by hard assets.
  • Option (d): Statement-I is incorrect, but Statement-II is correct
    • Incorrect: Statement-I is correct because in the event of a default, bondholders will not be able to exercise their claims.

Learn More

  • US Debt Default:
    • Definition: A debt default occurs when a country fails to meet its debt obligations on schedule, which for the US would mean not making payments to US Treasury bondholders.
    • Consequences: A default could lead to a significant downturn in financial markets, a surge in global interest rates, and a potential recession in the US.
    • Historical Context: The US has never defaulted in the traditional sense, but there have been close calls, such as in 1979 due to a technical error.
  • US Government Debt:
    • Nature: US government debt is backed by the full faith and credit of the government, not by hard assets.
    • Debt Instruments: The US issues Treasury bonds, bills, and notes to finance its spending, which are considered safe investments due to their perceived stability.
    • Debt-to-GDP Ratio: The US debt-to-GDP ratio is a key indicator of the country’s ability to repay its debt. As of recent data, it is around 100%, which is a cause for concern among economists.
  • Risks of Default:
    • Economic Impact: A default could shave around 4% from US GDP, cause stock prices to fall by a third, and result in significant job losses.
    • Legal and Financial Ramifications: The US government would face legal challenges and increased borrowing costs, and its credit rating could be downgraded.
  • Debt Ceiling:
    • Definition: The debt ceiling is the maximum amount the US government can borrow to meet its legal obligations.
    • Historical Adjustments: The debt ceiling has been raised or suspended numerous times to prevent default and ensure the government can meet its financial obligations.

Consider the following statements:

Statement-I:
Syndicated lending spreads the risk of borrower default across multiple lenders.

Statement-II:
The syndicated loan can be a fixed amount/lump sum of funds, but cannot be a credit line.

Which one of the following is correct in respect of the above statements?

(a) Both Statement-I and Statement-II are correct and Statement-II explains Statement-I

(b) Both Statement-I and Statement-II are correct, but Statement-II does not explain Statement-I

(c) Statement-I is correct, but Statement-II is incorrect

(d) Statement-I is incorrect, but Statement-II is correct

Correct Answer: (c) Statement-I is correct, but Statement-II is incorrect

Explanation:

  • Statement-I: Syndicated lending spreads the risk of borrower default across multiple lenders. This is correct because syndicated loans involve multiple lenders pooling their resources to provide a large loan to a single borrower, thereby distributing the risk among the lenders.
  • Statement-II: The syndicated loan can be a fixed amount/lump sum of funds, but cannot be a credit line. This is incorrect because syndicated loans can indeed be structured as a credit line, such as a revolving credit facility, in addition to being a fixed amount or lump sum.

Dealing with all options:

  • Option (a): Both Statement-I and Statement-II are correct and Statement-II explains Statement-I. This is incorrect because Statement-II is not correct.
  • Option (b): Both Statement-I and Statement-II are correct, but Statement-II does not explain Statement-I. This is incorrect because Statement-II is not correct.
  • Option (c): Statement-I is correct, but Statement-II is incorrect. This is correct because Statement-I accurately describes the risk distribution in syndicated lending, while Statement-II incorrectly states that syndicated loans cannot be credit lines.
  • Option (d): Statement-I is incorrect, but Statement-II is correct. This is incorrect because Statement-I is correct and Statement-II is incorrect.

Learn more

  • Definition: A syndicated loan is a loan provided by a group of lenders (syndicate) to a single borrower, typically for large-scale projects or substantial financial needs.
  • Participants: The syndicate includes multiple lenders, with one acting as the lead arranger or agent, responsible for structuring the loan and managing the syndicate.
  • Types of Loans: Syndicated loans can be structured as term loansrevolving credit facilities, or a combination of both. Term loans are fixed amounts, while revolving credit facilities allow borrowers to draw, repay, and reborrow funds.
  • Risk Distribution: The primary purpose of syndicated lending is to distribute the risk of borrower default across multiple lenders, reducing the exposure for any single lender.
  • Advantages for Borrowers: Borrowers benefit from access to larger pools of capital, flexible repayment terms, and the expertise of multiple lenders.
  • Advantages for Lenders: Lenders benefit from risk diversification, participation in larger transactions, and potential for higher returns.
  • Administrative Role: The lead arranger or agent handles administrative tasks, such as disbursing funds, monitoring compliance, and coordinating communication among lenders.
  • Documentation: Syndicated loans involve complex documentation, including loan agreements, intercreditor agreements, and security documents, which require careful negotiation and legal oversight.

Consider the following statements in respect of the digital rupee:

  1. It is a sovereign currency issued by the Reserve Bank of India (RBI) in alignment with its monetary policy.
  2. It appears as a liability on the RBI’s balance sheet.
  3. It is insured against inflation by its very design.
  4. It is freely convertible against commercial bank money and cash.

Which of the statements given above are correct?
(a) 1 and 2 only
(b) 1 and 3 only
(c) 2 and 4 only
(d) 1, 2 and 4

Correct Answer: (d) 1, 2 and 4

Explanation:

  • Statement 1: It is a sovereign currency issued by the Reserve Bank of India (RBI) in alignment with its monetary policy.
    • This statement is correct. The digital rupee is a sovereign currency issued by the RBI in accordance with its monetary policy.
  • Statement 2: It appears as a liability on the RBI’s balance sheet.
    • This statement is correct. The digital rupee appears as a liability on the RBI’s balance sheet.
  • Statement 3: It is insured against inflation by its very design.
    • This statement is incorrect. The digital rupee is not insured against inflation by its design. It functions similarly to physical cash, which is not inherently protected against inflation.
  • Statement 4: It is freely convertible against commercial bank money and cash.
    • This statement is correct. The digital rupee is freely convertible against commercial bank money and cash.

Learn more

  • Definition and Issuance:
    • The digital rupee is a Central Bank Digital Currency (CBDC) issued by the Reserve Bank of India (RBI). It is a digital form of the Indian rupee and is designed to complement existing forms of money.
  • Legal Tender:
    • The digital rupee is considered legal tender and can be used for all types of transactions, similar to physical currency.
  • Distribution and Usage:
    • The digital rupee is distributed through intermediaries, such as banks, and can be used for Person to Person (P2P) and Person to Merchant (P2M) transactions. Payments can be made using QR codes displayed at merchant locations.
  • Pilot Program:
    • The RBI has launched a pilot program to test the digital rupee in select locations with participating banks and users. The pilot aims to assess the robustness of the digital rupee’s creation, distribution, and retail usage.
  • Conversion and Storage:
    • The digital rupee can be converted into physical cash and vice versa. It is stored in a digital wallet provided by participating banks and can be accessed via mobile devices.
  • Benefits:
    • The digital rupee offers benefits such as reduced transaction costsfaster settlements, and enhanced financial inclusion. It also aims to provide a secure and efficient means of payment.
  • Comparison with Cryptocurrency:
    • Unlike cryptocurrencies, the digital rupee is regulated by the RBI and has a fixed value equivalent to the physical rupee. It is not subject to the same volatility as cryptocurrencies.
  • Privacy and Security:
    • The digital rupee transactions are designed to be secure and transparent. However, there are concerns about privacy as transactions can be monitored by the central bank.

With reference to ancient India, Gautama Buddha was generally known by which of the following epithets?

  1. Nayaputta
  2. Shakyamuni
  3. Tathagata

Select the correct answer using the code given below:
(a) 1 only
(b) 2 and 3 only
(c) 1, 2 and 3
(d) None of the above are epithets of Gautama Buddha

The correct answer is (b) 2 and 3 only.

  • Shakyamuni: This epithet means “Sage of the Shakyas” and is commonly used to refer to Gautama Buddha, highlighting his origin from the Shakya clan.
  • Tathagata: This term is used by Gautama Buddha to refer to himself or other Buddhas, meaning “one who has thus gone” or “one who has thus come”.
  • Nayaputta: This is not an epithet of Gautama Buddha. It is an epithet used for Mahavira, the 24th Tirthankara of Jainism.

Learn more

Gautama Buddha

  • Birth and Early Life: Born as Siddhartha Gautama in Lumbini, near Kapilavastu, on the present-day India-Nepal border, he was a prince of the Shakya clan.
  • Enlightenment: After renouncing his princely life, he sought enlightenment through various ascetic practices and ultimately achieved it under the Bodhi tree by realizing the Four Noble Truths.
  • Teachings: He preached the Eightfold Path, which offers a middle way between self-indulgence and self-mortification, leading to nirvana.
  • Death: He died after eating a meal that may have contained spoiled pork, escaping the cycle of rebirth, and his relics were enshrined in stupas.

Epithets of Gautama Buddha

  • Shakyamuni: Emphasizes his origin from the Shakya clan.
  • Tathagata: Used by Buddha to refer to himself, signifying one who has transcended the human condition.
  • Other Epithets: Includes Bhagavato (Blessed One), Sammasambuddho (Perfectly self-awakened), and many others that highlight his qualities and achievements.

Mahavira

  • Names and Epithets: Known by several names such as Nayaputta, Muni, Samana, and Nigantha.
  • Teachings: Promoted doctrines like anekantavada, emphasizing multiple perspectives and the nature of reality.

Consider the following information:

Archaeological SiteStateDescription
1. ChandraketugarhOdishaTrading Port town
2. InamgaonMaharashtraChalcolithic site
3. MangaduKeralaMegalithic site
4. SalihundamAndhra PradeshRock-cut cave shrines

In which of the above rows is the given information correctly matched?

(a) 1 and 2
(b) 2 and 3
(c) 3 and 4
(d) 1 and 4

Correct Answer: (b) 2 and 3

  • Option 1: Chandraketugarh | Odisha | Trading Port town
    • Incorrect: Chandraketugarh is located in West Bengal, not Odisha. It is an ancient site known for being a trading port town and an important urban center in early historic times.
  • Option 2: Inamgaon | Maharashtra | Chalcolithic site
    • Correct: Inamgaon is an archaeological site located in Maharashtra. It is a significant Chalcolithic site known for its extensive excavations revealing details about the Malwa and Jorwe cultures.
  • Option 3: Mangadu | Kerala | Megalithic site
    • Correct: Mangadu is located in Kerala and is known for its Megalithic site with numerous megaliths, including urn burials and stone circles.
  • Option 4: Salihundam | Andhra Pradesh | Rock-cut cave shrines
    • Incorrect: Salihundam is located in Andhra Pradesh but is known for its Buddhist stupas and monastic complexes, not rock-cut cave shrines.

Learn more

  • Chandraketugarh:
    • Located in West Bengal, about 35 km northeast of Kolkata.
    • Identified with the ancient kingdom of Gangaridai.
    • Archaeological findings date back to the 3rd century BCE.
    • Known for its fortified city, trade relations, and terracotta art.
  • Inamgaon:
    • Situated in Maharashtra on the banks of the river Ghod.
    • Chalcolithic site with evidence of Malwa and Jorwe cultures.
    • Excavations revealed house plans, pottery, and burial practices.
    • Provides insights into the subsistence and habitation patterns of Chalcolithic farmers.
  • Mangadu:
    • Located in Kerala, known for its Megalithic monuments.
    • Features include urn burials, stone circles, and iron implements.
    • Dates back to the Iron Age, indicating early human settlement and agricultural practices.
  • Salihundam:
    • Situated in Andhra Pradesh on the banks of the Vamsadhara River.
    • Known for its Buddhist stupas, chaityas, and viharas.
    • Reflects the spread of Buddhism from the 2nd century CE.
    • Important for understanding the evolution of Buddhist architecture.

Who of the following rulers of medieval India gave permission to the Portuguese to build a fort at Bhatkal?

(a) Krishnadevaraya

(b) Narasimha Saluva

(c) Muhammad Shah III

(d) Yusuf Adil Shah

Correct Answer: (a) Krishnadevaraya

  • Krishnadevaraya: The Portuguese Governor Afonso de Albuquerque sought and received permission from Krishnadevaraya to build a fort at Bhatkal. Krishnadevaraya was a prominent ruler of the Vijayanagara Empire and maintained friendly relations with the Portuguese, facilitating their strategic and commercial interests in the region.
  • Narasimha Saluva: There is no historical evidence indicating that Narasimha Saluva granted permission to the Portuguese to build a fort at Bhatkal. He was a ruler of the Vijayanagara Empire but did not engage in such diplomatic relations with the Portuguese.
  • Muhammad Shah IIIMuhammad Shah III was the Sultan of the Bahmani Sultanate and did not have the authority over Bhatkal to grant such permissions. His reign was marked by conflicts with other regional powers, but not by granting permissions to the Portuguese for fort construction.
  • Yusuf Adil ShahYusuf Adil Shah was the founder of the Adil Shahi dynasty of Bijapur. While he had interactions with the Portuguese, there is no record of him granting permission to build a fort at Bhatkal. His engagements with the Portuguese were more conflictual, particularly over the control of Goa.

Learn more

  • Krishnadevaraya:
    • Reign: 1509-1529, considered the greatest ruler of the Vijayanagara Empire.
    • Diplomacy: Maintained friendly relations with the Portuguese, allowing them to build forts and trading posts.
    • Cultural Contributions: Authored the Telugu work “Amuktamalyada” and patronized literature and arts.
    • Military Achievements: Expanded the empire and secured its borders against the Bahmani Sultanate and other adversaries.
  • Narasimha Saluva:
    • Reign: Founder of the Saluva dynasty within the Vijayanagara Empire.
    • Military Campaigns: Known for consolidating power and defending the empire against internal and external threats.
    • Diplomatic Relations: Did not engage in significant diplomatic relations with the Portuguese.
  • Muhammad Shah III:
    • Reign: Sultan of the Bahmani Sultanate from 1463 to 1482.
    • Military Campaigns: Known for his invasions of Orissa and conflicts with regional powers.
    • Administrative Reforms: Implemented standard measurements and land valuations.
  • Yusuf Adil Shah:
    • Reign: Founder of the Adil Shahi dynasty of Bijapur, ruled from 1489 to 1510.
    • Diplomatic Relations: Engaged in both conflict and cooperation with the Portuguese, particularly over Goa.
    • Cultural Contributions: Promoted arts and invited scholars to his court.

With reference to revenue collection by Cornwallis, consider the following statements:

  1. Under the Ryotwari Settlement of revenue collection, the peasants were exempted from revenue payment in case of bad harvests or natural calamities.
  2. Under the Permanent Settlement in Bengal, if the Zamindar failed to pay his revenues to the state, on or before the fixed date, he would be removed from his Zamindari.

Which of the statements given above is/are correct?
(a) 1 only
(b) 2 only
(c) Both 1 and 2
(d) Neither 1 nor 2

Correct Answer: (b) 2 only

  • Statement 1Under the Ryotwari Settlement of revenue collection, the peasants were exempted from revenue payment in case of bad harvests or natural calamities.
    • This statement is incorrect. Under the Ryotwari System, peasants were not exempted from paying revenue even in the event of crop failure or natural calamities. They had to pay taxes in cash, which often led to exploitation and financial distress for the peasants.
  • Statement 2Under the Permanent Settlement in Bengal, if the Zamindar failed to pay his revenues to the state, on or before the fixed date, he would be removed from his Zamindari.
    • This statement is correct. Under the Permanent Settlement, if a Zamindar failed to pay the fixed revenue to the state by the specified date, his land would be confiscated and auctioned off by the British authorities.

Learn more

Permanent Settlement

  • Introduced by: Lord Cornwallis in 1793.
  • Regions: Initially Bengal, Bihar, and Odisha; later extended to other regions.
  • Key Features:
    • Zamindars were recognized as the owners of the land with hereditary rights.
    • They had to pay a fixed revenue to the British government.
    • If they failed to pay, their land would be auctioned.
    • The system aimed to create a loyal class of landlords who would support British rule.
    • The revenue demand was fixed permanently, which meant it could not be increased in the future.

Ryotwari System

  • Introduced by: Sir Thomas Munro in the early 19th century.
  • Regions: Practiced in Madras, Bombay, Assam, and Coorg.
  • Key Features:
    • Peasants (Ryots) were recognized as the owners of the land.
    • They paid revenue directly to the government.
    • The revenue rates were high and could be increased.
    • Peasants had to pay taxes in cash, leading to exploitation by moneylenders.
    • There were no middlemen like Zamindars in this system.

Comparison with Mahalwari System

  • Introduced by: Holt Mackenzie in 1822.
  • Regions: North-Western Provinces, Punjab, and parts of Central India.
  • Key Features:
    • Land was owned collectively by the village (Mahal).
    • Revenue was collected by the village headman on behalf of the entire village.
    • The revenue demand was periodically revised, not fixed permanently.
    • Aimed to combine elements of both Zamindari and Ryotwari systems.

Consider the following statements:

  1. There are no parables in Upanishads.
  2. Upanishads were composed earlier than the Puranas.

Which of the statements given above is/are correct?

(a) 1 only
(b) 2 only
(c) Both 1 and 2
(d) Neither 1 nor 2

Correct Answer: (b) 2 only

Explanation:

  • Statement 1: There are no parables in Upanishads.
    • This statement is incorrect. The Upanishads contain several parables. For example, the Chandogya Upanishad includes a parable involving a dialogue between Udhalaka and his son Svetaketu, which illustrates profound philosophical concepts through a simple story about a Nyagrodha (peepal) tree.
  • Statement 2: Upanishads were composed earlier than the Puranas.
    • This statement is correct. The Upanishads were composed between 800 BCE and the start of the Common Era. In contrast, the Puranas were likely composed between the 3rd and 10th centuries CE.

Learn more

  • Upanishads:
    • Definition: The Upanishads are ancient Hindu texts that form the philosophical basis of Hinduism, often referred to as Vedanta, meaning the end of the Vedas.
    • Content: They explore concepts such as Brahman (the ultimate reality) and Atman (the individual soul), and their interconnectedness.
    • Composition: The principal Upanishads were composed between 800 BCE and 500 BCE, with some later ones extending to the start of the Common Era.
    • Influence: They have significantly influenced Hinduism and other Indian religions like Buddhism and Jainism, and have also impacted Western philosophical thought.
  • Puranas:
    • Definition: The Puranas are a genre of ancient Indian literature encompassing mythological stories, traditions, and legends, often with a focus on genealogies and cosmology.
    • Content: They include narratives about the creation of the universe, genealogies of gods, heroes, and sages, and descriptions of Hindu cosmology and geography.
    • Composition: The Puranas were composed over a long period, from the 3rd to the 10th century CE, with some texts undergoing significant redactions and interpolations over time.
    • Significance: They serve as important texts for understanding the cultural and religious history of India and are considered encyclopedic in nature.

Consider the following statements:

  1. India is a member of the International Grains Council.
  2. A country needs to be a member of the International Grains Council for exporting or importing rice and wheat.

Which of the statements given above is/are correct?

(a) 1 only

(b) 2 only

(c) Both 1 and 2

(d) Neither 1 nor 2

Correct Answer: (a) 1 only

  • Statement 1India is a member of the International Grains Council. This statement is correct. India is indeed a member of the International Grains Council (IGC).
  • Statement 2A country needs to be a member of the International Grains Council for exporting or importing rice and wheat. This statement is incorrect. There is no requirement for a country to be a member of the IGC to export or import rice and wheat.

Learn more

  • International Grains Council (IGC):
    • Establishment: The IGC was established in 1949 as the International Wheat Council and was renamed in 1995 under the Grains Trade Convention.
    • Objectives:
      • Promote international cooperation in the grains trade.
      • Enhance market transparency through information-sharing, analysis, and consultation on market and policy developments.
      • Monitor market conditions for grains, rice, and oilseeds on a daily basis.
    • Functions:
      • Oversee the implementation of the Grains Trade Convention.
      • Discuss current and prospective grain market developments.
      • Monitor changes in national grain policies and their market implications.
    • Membership:
      • Comprises major grain-producing and consuming countries.
      • Members are designated as importers or exporters based on their average trade in grains, rice, and oilseeds.
    • Key Activities:
      • Grains and Oilseeds Index (GOI): Provides daily updates on international grain and oilseed prices.
      • Grain Market Report: Monthly publication reviewing the situation and outlook for various grains and oilseeds.
      • Annual IGC Grains Conference: A public forum for discussion among international buyers, sellers, industry representatives, and policymakers.
    • Role in Food Security:
      • Provides forecasts and statistics to help countries plan for food security and stabilize prices in volatile markets.
      • Coordinates international responses during food crises or emergencies.
    • Collaborations:
      • Engages with other international bodies like the World Trade Organization (WTO) and the Food and Agriculture Organization (FAO).
      • Part of the Secretariat of the Agricultural Market Information System (AMIS), promoting food market transparency and policy coordination.

Which one of the following was the latest inclusion in the Intangible Cultural Heritage List of UNESCO?

(a) Chhau dance

(b) Durga puja

(c) Garba dance

(d) Kumbh mela

The correct answer is (c) Garba dance.

  • Garba dance was the latest inclusion in the Intangible Cultural Heritage List of UNESCO in 2023.

Explanation

  • Chhau dance:
    • Inscribed in 2010 on the Representative List of the Intangible Cultural Heritage of Humanity.
    • It is a traditional dance from eastern India, enacting episodes from epics like the Mahabharata and Ramayana, and local folklore.
  • Durga Puja:
    • Inscribed in 2021 on the Representative List of the Intangible Cultural Heritage of Humanity.
    • It is an annual festival celebrated in September or October, most notably in Kolkata, marking the ten-day worship of the Hindu mother-goddess Durga.
  • Garba dance:
    • Inscribed in 2023 on the Representative List of the Intangible Cultural Heritage of Humanity.
    • It is a ritualistic and devotional dance performed during the Hindu festival of Navaratri, dedicated to the worship of the feminine energy or Shakti.
  • Kumbh Mela:
    • Inscribed in 2017 on the Representative List of the Intangible Cultural Heritage of Humanity.
    • It is the largest peaceful congregation of pilgrims on earth, held every four years by rotation in four different cities in India.

Learn more

  • UNESCO Intangible Cultural Heritage List:
    • Purpose: To ensure better protection of important intangible cultural heritages worldwide and to raise awareness of their significance.
    • Categories:
      • Representative List of the Intangible Cultural Heritage of Humanity: Demonstrates the diversity of cultural heritage and raises awareness about its importance.
      • List of Intangible Cultural Heritage in Need of Urgent Safeguarding: Includes cultural elements that require urgent measures to keep them alive.
      • Register of Good Safeguarding Practices: Showcases successful examples of safeguarding intangible cultural heritage.
  • Criteria for Inscription:
    • Community Involvement: The practice must involve the community and be recognized as a symbol of their identity and continuity.
    • Visibility and Awareness: Inscription should encourage cooperation and dialogue between communities and promote the visibility of intangible cultural heritage.
    • Safeguarding Measures: There must be national and local measures in place to encourage and sustain the practice.
    • Consent: The nomination must be submitted with the participation and consent of the communities of practitioners.
    • Inventory Inclusion: The practice must be included in a national inventory of intangible cultural heritage.
  • Significance:
    • Cultural Diversity: Intangible cultural heritage is a repository of cultural diversity and creative expression.
    • Transmission: It is transmitted across generations through practice, performance, imitation, and observation.
    • Social Cohesion: Many practices foster social equality and strengthen community bonds by being inclusive and participative.

Who was the Provisional President of the Constituent Assembly before Dr. Rajendra Prasad took over?

(a) C. Rajagopalachari
(b) Dr. B.R. Ambedkar
(c) T.T. Krishnamachari
(d) Dr. Sachchidananda Sinha

Correct Answer: (d) Dr. Sachchidananda Sinha

  • Dr. Sachchidananda Sinha was the Provisional President of the Constituent Assembly before Dr. Rajendra Prasad took over as the permanent president.

Explanation

  • C. Rajagopalachari:
    • He was a prominent member of the Constituent Assembly and played a significant role in India’s independence movement.
    • He was elected to the Constituent Assembly from Madras and intervened on issues of religious freedom and citizenship.
    • However, he was not the Provisional President of the Constituent Assembly.
  • Dr. B.R. Ambedkar:
    • He was the Chairman of the Drafting Committee and a key figure in the constitution-making process.
    • Ambedkar is often referred to as the father of the Indian Constitution due to his significant contributions.
    • He was not the Provisional President of the Constituent Assembly.
  • T.T. Krishnamachari:
    • He was a member of the Drafting Committee and served as Finance Minister of India.
    • Krishnamachari was also involved in various other political roles but was not the Provisional President of the Constituent Assembly.
  • Dr. Sachchidananda Sinha:
    • He was the first Provisional President of the Constituent Assembly, serving in this role before Dr. Rajendra Prasad was elected as the permanent president.

Learn more

  • Constituent Assembly of India:
    • The Constituent Assembly was formed to draft the Constitution of India.
    • It was created under the Cabinet Mission Plan of 1946 and had 389 members initially, which later reduced to 299 after partition.
    • The Assembly first met on 9 December 1946, and Dr. Sachchidananda Sinha was the Provisional President during this initial meeting.
    • Dr. Rajendra Prasad was elected as the permanent president on 11 December 1946.
    • The Assembly included prominent leaders like Jawaharlal Nehru, B.R. Ambedkar, and many others who played crucial roles in framing the Constitution.
    • The Constitution was adopted on 26 November 1949 and came into force on 26 January 1950, marking the establishment of the Republic of India.

With reference to the Government of India Act, 1935, consider the following statements:

  1. It provided for the establishment of an All India Federation based on the union of the British Indian Provinces and Princely States.
  2. Defence and Foreign Affairs were kept under the control of the federal legislature.

Which of the statements given above is/are correct?

(a) 1 only
(b) 2 only
(c) Both 1 and 2
(d) Neither 1 nor 2

Correct Answer: (a) 1 only

  • Statement 1: The Government of India Act, 1935 provided for the establishment of an All India Federation based on the union of the British Indian Provinces and Princely States. This federation was intended to include both British India and the princely states, although the latter’s participation was not mandatory.
  • Statement 2Defence and Foreign Affairs were not kept under the control of the federal legislature. Instead, these subjects were part of the reserved subjects controlled by the Governor-General, who administered them with the help of appointed counsellors and not the legislature.

Learn more

  • Government of India Act, 1935:
    • Aim: To make further provisions for the governance of India.
    • Territorial Extent: Applied to territories under direct British control.
    • Enacted by: Parliament of the United Kingdom.
    • Royal Assent: 24th July 1935.
    • Commencement: 1st April 1937.
    • Status: Repealed on 26th January 1950 in India.
  • Background:
    • The Act was a response to the growing demand for constitutional reforms in India.
    • It was based on the Simon Commission Report, recommendations of the Round Table Conferences, the White Paper of 1933, and the Report of the Joint Select Committees.
  • Key Features:
    • All India Federation: Proposed a federation consisting of British Indian provinces and princely states, although it never materialized due to lack of support from the princely states.
    • Division of Powers: Powers were divided between the centre and the provinces through three lists: Federal List, Provincial List, and Concurrent List. Residuary powers were vested in the Viceroy.
    • Provincial Autonomy: Introduced provincial autonomy, abolishing dyarchy at the provincial level. Governors were to act on the advice of ministers responsible to the provincial legislatures.
    • Diarchy at the Centre: Federal subjects were divided into reserved and transferred subjects. Reserved subjects were controlled by the Governor-General, while transferred subjects were administered by the Governor-General with his Council of Ministers.
    • Bicameral Legislature: Introduced bicameralism in six out of eleven provinces, with legislative councils and assemblies.
    • Federal Court: Established a Federal Court in 1937, which functioned until the establishment of the Supreme Court of India in 1950.
    • Franchise: Extended the franchise, allowing about 10-14% of the population to vote.
    • Reorganization: Included the separation of Burma from India and the reorganization of provinces like Sindh from Bombay Presidency.
  • Significance:
    • Marked a significant step towards a responsible government in India.
    • Laid the groundwork for the future constitutional framework of independent India.
    • Despite its limitations, it was a crucial milestone in India’s constitutional development.

Which one of the following is a work attributed to playwright Bhasa?

(a) Kavyalankara
(b) Natyashastra
(c) Madhyama-vyayoga
(d) Mahabhashya

Correct Answer: (c) Madhyama-vyayoga

  • Madhyama-vyayoga is a work attributed to the playwright Bhasa. This play focuses on the name confusion between the priest Keshav Das’s middle son and the middle Pandava prince Bhima, and also features the reunion of Bhima and Ghatotkacha as father and son.

Explanation of Other Options:

  • Kavyalankara: This is a Sanskrit work by Bhamaha, not Bhasa. It is a critical study of poetics and literary devices, focusing on the ornaments of poetry.
  • Natyashastra: This is a Sanskrit treatise on the performing arts attributed to sage Bharata. It covers various aspects of drama, dance, and music, and is not related to Bhasa.
  • Mahabhashya: This is a commentary on selected rules of Sanskrit grammar from Pāṇini’s treatise, attributed to Patañjali. It is not a work of Bhasa.

Learn more

  • Bhasa:
    • Bhasa is one of the earliest known Sanskrit dramatists, often referred to as the “Shakespeare of Sanskrit drama”.
    • His works were rediscovered in 1912 by the Indian scholar Ganapati Shastri, who found 13 plays attributed to him.
    • Bhasa’s plays are known for their rich storytelling, complex characters, and incorporation of mythological and heroic themes from Indian epics like the Mahabharata and Ramayana.
  • Themes and Style:
    • Bhasa’s plays cover a wide range of themes including love, valor, duty, and morality.
    • His characters often embody complex human emotions and dilemmas, making his works timeless and relatable.
    • Bhasa deviated from the accepted dramaturgy of his time by portraying battle scenes and killings on stage, which was unconventional.
  • Influence and Legacy:
    • Bhasa’s influence is evident in the works of later dramatists like Kalidasa and Bhavabhuti, who built upon his literary motifs.
    • His plays continue to be staged and adapted, showcasing their enduring appeal and relevance.
  • Notable Works:
    • Swapnavasavadatta: A tale of love and sacrifice, considered one of his most celebrated works.
    • Madhyama-vyayoga: Focuses on the reunion of Bhima and Ghatotkacha.
    • Karnabharam: Depicts the burden of Karna from the Mahabharata.
    • Urubhanga: Focuses on the broken thigh of Duryodhana from the Mahabharata.

Sanghabhuti, an Indian Buddhist monk, who travelled to China at the end of the fourth century AD, was the author of a commentary on:

(a) Prajnaparamita Sutra
(b) Visuddhimagga
(c) Sarvastivada Vinaya
(d) Lalitavistara

Correct Answer: (c) Sarvastivada Vinaya

Explanation:

  • Sanghabhuti was an Indian Buddhist monk who traveled to China at the end of the fourth century AD.
  • He authored a commentary on the Sarvastivada Vinaya.

Dealing with Options:

  • Prajnaparamita Sutra: This is a body of sutras in Mahayana Buddhism that represents the Perfection of Wisdom. It is not authored by Sanghabhuti.
  • Visuddhimagga: This is a Theravada Buddhist text written by Buddhaghosa in the 5th century. It is a comprehensive manual on Buddhist practice and not related to Sanghabhuti.
  • Sarvastivada Vinaya: This is the correct answer. Sanghabhuti wrote a commentary on this text, which is a set of monastic rules followed by the Sarvastivada school of Buddhism.
  • Lalitavistara: This is a Mahayana Buddhist text that describes the life of the Buddha. It is not authored by Sanghabhuti.

Learn more

  • Sarvastivada Vinaya:
    • The Sarvastivada Vinaya is a set of monastic rules followed by the Sarvastivada school of Buddhism.
    • It was one of the most influential Buddhist monastic groups in North India, especially in Kashmir and Central Asia.
    • The Sarvastivada Vinaya was prominent in China before the Dharmaguptaka Vinaya became more widely adopted.
    • The Vinaya includes detailed rules and regulations for monastic life, covering aspects such as ordinationmonastic discipline, and daily conduct.
    • Sanghabhuti translated many Buddhist scriptures into Chinese, including his commentary on the Sarvastivada Vinaya, which contains detailed annotations about the Vinaya Pitaka of the Sarvastivada School.

Consider the following properties included in the World Heritage List released by UNESCO:

  1. Shantiniketan
  2. Rani-ki-Vav
  3. Sacred Ensembles of the Hoysalas
  4. Mahabodhi Temple Complex at Bodhgaya

How many of the above properties were included in 2023?

(a) Only one
(b) Only two
(c) Only three
(d) All four

Correct Answer: (b) Only two

  • Shantiniketan was included in the UNESCO World Heritage List in 2023. It was recognized for its cultural and educational significance, founded by Rabindranath Tagore in West Bengal, India.
  • Rani-ki-Vav was included in the UNESCO World Heritage List in 2014. This stepwell in Gujarat, India, is known for its architectural and artistic mastery.
  • Sacred Ensembles of the Hoysalas were included in the UNESCO World Heritage List in 2023. These temples in Karnataka, India, are celebrated for their intricate stone carvings and architectural excellence.
  • Mahabodhi Temple Complex at Bodhgaya was included in the UNESCO World Heritage List in 2002. This site in Bihar, India, is significant for being the place where Lord Buddha attained enlightenment.

Learn more

  • Shantiniketan:
    • Founded by Rabindranath Tagore in 1901 in West Bengal, India.
    • Recognized for its unique blend of environmental art, educational reform, and cultural traditions.
    • Home to Visva-Bharati University, a center of excellence in various disciplines.
  • Rani-ki-Vav:
    • Located in Patan, Gujarat, India.
    • Built in the 11th century as a memorial to King Bhima I by his queen Udayamati.
    • Known for its seven levels of stairs and over 1,500 sculptures depicting religious, mythological, and secular themes.
  • Sacred Ensembles of the Hoysalas:
    • Comprises three main temples: Chennakeshava Temple at Belur, Hoysaleswara Temple at Halebidu, and Keshava Temple at Somanathapura.
    • Built in the 12th-13th centuries by the Hoysala kings.
    • Noted for their hyper-real sculptures, stone carvings, and unique architectural style combining various Indian temple traditions.
  • Mahabodhi Temple Complex at Bodhgaya:
    • Located in Bihar, India.
    • Marks the spot where Lord Buddha attained enlightenment around 2,500 years ago.
    • Features a large brick temple built by Emperor Ashoka in the 3rd century BC, with significant additions during the Gupta period.

As per Article 368 of the Constitution of India, the Parliament may amend any provision of the Constitution by way of:

  1. Addition
  2. Variation
  3. Repeal

Select the correct answer using the code given below:
(a) 1 and 2 only
(b) 2 and 3 only
(c) 1 and 3 only
(d) 1, 2 and 3

Correct Answer: (d) 1, 2 and 3

Explanation:

  • Article 368 of the Constitution of India grants the Parliament the power to amend the Constitution by way of addition, variation, or repeal of any provision.
  • This means that the Parliament can:
    • Add new provisions to the Constitution.
    • Vary or modify existing provisions.
    • Repeal or remove provisions from the Constitution.

Learn more:

Article 368 of the Constitution of India

  • Constituent Power: Article 368 grants the Parliament the constituent power to amend the Constitution, which is different from the ordinary legislative process.
  • Procedure for Amendment:
    • Introduction of Bill: An amendment can be initiated by introducing a Bill in either House of Parliament.
    • Special Majority: The Bill must be passed by a majority of the total membership of each House and by a majority of not less than two-thirds of the members present and voting.
    • President’s Assent: After being passed by both Houses, the Bill is presented to the President for assent. Upon receiving the President’s assent, the Constitution stands amended.
  • Special Provisions:
    • Certain amendments require ratification by the Legislatures of not less than one-half of the States. This includes amendments affecting:
      • The election of the President (Articles 54 and 55).
      • The extent of the executive power of the Union and the States (Articles 73 and 162).
      • The Union judiciary and the High Courts in the States (Chapter IV of Part V and Chapter V of Part VI).
      • The distribution of legislative powers between the Union and the States (Chapter I of Part XI and the Seventh Schedule).
      • The representation of States in Parliament.
      • The provisions of Article 368 itself.
  • Judicial Review: The Basic Structure Doctrine established by the Supreme Court in the Kesavananda Bharati case (1973) limits the Parliament’s power to amend the Constitution. Amendments cannot alter the basic structure of the Constitution, which includes fundamental principles like the supremacy of the Constitution, the rule of law, and the separation of powers.

Consider the following countries:

  1. Italy
  2. Japan
  3. Nigeria
  4. South Korea
  5. South Africa

Which of the above countries are frequently mentioned in the media for their low birth rates, or ageing population or declining population?

(a) 1, 2 and 4
(b) 1, 3 and 5
(c) 2 and 4 only
(d) 3 and 5 only

Correct Answer: (a) 1, 2 and 4

Explanation:

  • Italy:
    • Italy is frequently mentioned in the media for its low birth rates and ageing population. The country’s fertility rate has fallen to 1.2 in 2023, and almost one in four residents are above the age of 65.
  • Japan:
    • Japan is well-known for its declining birth rate and ageing population. The fertility rate hit a record low of 1.26 in 2022, and the population is projected to fall significantly by 2070.
  • Nigeria:
    • Nigeria is not frequently mentioned for low birth rates or ageing population. Instead, it has a high birth rate and a young population structure.
  • South Korea:
    • South Korea is frequently highlighted for its record-low fertility rate and ageing population. The fertility rate dropped to 0.72 in 2023, the lowest in the world.
  • South Africa:
    • South Africa is not commonly mentioned in the context of low birth rates or ageing population. It has a relatively young population and higher fertility rates compared to countries like Italy, Japan, and South Korea.

Learn more

  • Italy:
    • Italy’s fertility rate has been declining for years, reaching a historic low of 1.2 in 2023. The country faces a significant ageing population issue, with 24% of its residents aged 65 or older. The population is projected to decrease to 48 million by 2070 if current trends continue.
  • Japan:
    • Japan’s fertility rate has been declining for decades, hitting a record low of 1.26 in 2022. The population is expected to fall from over 125 million to 87 million by 2070. The ageing population, with a high percentage of elderly, poses economic and social challenges.
  • South Korea:
    • South Korea has the world’s lowest fertility rate, which dropped to 0.72 in 2023. The country faces a severe ageing population issue, with projections indicating that the working-age population will be less than the elderly population by 2067. Efforts to reverse the trend have included significant financial incentives and policy changes.

Which of the following statements are correct in respect of a Money Bill in the Parliament?

  1. Article 109 mentions special procedure in respect of Money Bills.
  2. A Money Bill shall not be introduced in the Council of States.
  3. The Rajya Sabha can either approve the Bill or suggest changes but cannot reject it.
  4. Amendments to a Money Bill suggested by the Rajya Sabha have to be accepted by the Lok Sabha.

Select the answer using the code given below:

(a) 1 and 2 only
(b) 2 and 3 only
(c) 1, 2 and 3
(d) 1, 3 and 4

The correct answer is (c) 1, 2 and 3.

Explanation

  • Option 1: Correct. Article 109 of the Indian Constitution outlines the special procedure for Money Bills.
  • Option 2: Correct. A Money Bill cannot be introduced in the Council of States (Rajya Sabha).
  • Option 3: Correct. The Rajya Sabha can only suggest amendments or approve the Bill but cannot reject it.
  • Option 4: Incorrect. The Lok Sabha is not obligated to accept the amendments suggested by the Rajya Sabha.

Learn more

Money Bills in the Indian Parliament

  • Definition: A Money Bill is defined under Article 110 of the Indian Constitution. It deals exclusively with financial matters such as taxation, public expenditure, and government borrowing.
  • Introduction: A Money Bill can only be introduced in the Lok Sabha (House of the People) and not in the Rajya Sabha (Council of States).
  • President’s Recommendation: The introduction of a Money Bill requires the recommendation of the President of India.
  • Role of Lok Sabha: The Lok Sabha has the primary role in the passage of a Money Bill. It undergoes the usual legislative process of debates, discussions, and voting in the Lok Sabha.
  • Role of Rajya Sabha: The Rajya Sabha can only make recommendations on a Money Bill within 14 days of receiving it. The Lok Sabha may accept or reject these recommendations.
  • Final Approval: Once the Lok Sabha passes the Money Bill, it is sent to the President for assent. The President can either give assent or withhold it but cannot return the Bill for reconsideration.
  • SignificanceMoney Bills are crucial for the financial management and governance of the country. They ensure that matters related to taxation and expenditure are given due importance and are subject to thorough scrutiny.

Which of the following is/are correctly matched in terms of equivalent rank in the three services of Indian Defence forces?

ArmyAirforceNavy
1. BrigadierAir CommodoreCommander
2. Major GeneralAir Vice MarshalVice Admiral
3. MajorSquadron LeaderLieutenant Commander
4. Lieutenant ColonelGroup CaptainCaptain

Select the correct answer using the code given below:

(a) 1 and 4
(b) 1 and 3
(c) 2, 3 and 4
(d) 3 only

The correct answer is (d) 3 only.

Explanation

  • Option 1: Brigadier (Army) – Air Commodore (Airforce) – Commander (Navy)
    • Incorrect: The equivalent rank of Brigadier in the Indian Army is Air Commodore in the Airforce and Commodore in the Navy, not Commander.
  • Option 2: Major General (Army) – Air Vice Marshal (Airforce) – Vice Admiral (Navy)
    • Incorrect: The equivalent rank of Major General in the Indian Army is Air Vice Marshal in the Airforce and Rear Admiral in the Navy, not Vice Admiral.
  • Option 3: Major (Army) – Squadron Leader (Airforce) – Lieutenant Commander (Navy)
    • Correct: The equivalent rank of Major in the Indian Army is Squadron Leader in the Airforce and Lieutenant Commander in the Navy.
  • Option 4: Lieutenant Colonel (Army) – Group Captain (Airforce) – Captain (Navy)
    • Incorrect: The equivalent rank of Lieutenant Colonel in the Indian Army is Wing Commander in the Airforce and Commander in the Navy, not Group Captain and Captain respectively.

Learn more

  • Equivalent Ranks in Indian Defence Forces:
    • The Indian Armed Forces consist of the Army, Navy, and Air Force.
    • Each branch has a structured hierarchy of ranks to ensure smooth command and control.
    • Commissioned Officers lead and manage the forces, with ranks corresponding across the three branches for effective communication and cooperation.
  • Commissioned Officer Ranks:
    • Indian Army: Lieutenant, Captain, Major, Lieutenant Colonel, Colonel, Brigadier, Major General, Lieutenant General, General, Field Marshal.
    • Indian Navy: Sub Lieutenant, Lieutenant, Lieutenant Commander, Commander, Captain, Commodore, Rear Admiral, Vice Admiral, Admiral, Admiral of the Fleet.
    • Indian Air Force: Flying Officer, Flight Lieutenant, Squadron Leader, Wing Commander, Group Captain, Air Commodore, Air Vice Marshal, Air Marshal, Air Chief Marshal, Marshal of the Indian Air Force.
  • Junior Commissioned Officers (JCOs) and Non-Commissioned Officers (NCOs):
    • Indian Army: Naib Subedar, Subedar, Subedar Major.
    • Indian Navy: Chief Petty Officer, Master Chief Petty Officer II, Master Chief Petty Officer I.
    • Indian Air Force: Junior Warrant Officer, Warrant Officer.
  • Operational Efficiency:
    • The ranks ensure a clear chain of command and operational efficiency.
    • Understanding equivalent ranks is crucial for joint missions and inter-service communication.

The North Eastern Council (NEC) was established by the North Eastern Council Act, 1971. Subsequent to the amendment of NEC Act in 2002, the Council comprises which of the following members?

  1. Governor of the Constituent State
  2. Chief Minister of the Constituent State
  3. Three Members to be nominated by the President of India
  4. The Home Minister of India

Select the correct answer using the code given below:

(a) 1, 2 and 3 only
(b) 1, 3 and 4 only
(c) 2 and 4 only
(d) 1, 2, 3 and 4

Correct Answer: (a) 1, 2 and 3 only

  • The North Eastern Council (NEC) was established by the North Eastern Council Act, 1971.
  • Subsequent to the amendment of NEC Act in 2002, the Council comprises the following members:
    • Governor of the Constituent State .
    • Chief Minister of the Constituent State .
    • Three Members to be nominated by the President of India .

Explanation

  • Option 1: Governor of the Constituent State
    • The Governors of the constituent states are members of the NEC .
  • Option 2: Chief Minister of the Constituent State
    • The Chief Ministers of the constituent states are also members of the NEC .
  • Option 3: Three Members to be nominated by the President of India
    • The President of India nominates three members to the NEC .
  • Option 4: The Home Minister of India
    • The Home Minister of India is not a member of the NEC. Instead, the Union Home Minister is the ex-officio chairman of the NEC .

Learn more

North Eastern Council (NEC)

  • Establishment: The NEC was established by the North Eastern Council Act, 1971 to promote the socio-economic development of the North Eastern region .
  • Amendment in 2002: The NEC Act was amended in 2002 to redefine its functions as a regional planning body and to include Sikkim as a member state .
  • Composition:
    • Governors and Chief Ministers of the constituent states .
    • Three members nominated by the President of India .
    • The Union Home Minister serves as the ex-officio chairman, and the Minister of State for Development of North Eastern Region (DoNER) is the ex-officio vice-chairman .
  • Functions:
    • Acts as a regional planning body for the North Eastern region .
    • Provides financial assistance for infrastructure projects, including roads and air connectivity .
    • Oversees the implementation of projects and schemes to ensure proper and timely execution .
  • Headquarters: Located in Shillong, the NEC started functioning in 1972 .
  • Funding: Primarily provided by the Central Government, with contributions from state governments .

How many Delimitation Commissions have been constituted by the Government of India till December 2023?

(a) One
(b) Two
(c) Three
(d) Four

Correct Answer: (d) Four

  • Four Delimitation Commissions have been constituted by the Government of India till December 2023.

Explanation:

  • Option (a) One: This is incorrect. The first Delimitation Commission was constituted in 1952, but there have been more since then.
  • Option (b) Two: This is incorrect. The second Delimitation Commission was constituted in 1963, but there have been additional commissions after that.
  • Option (c) Three: This is incorrect. The third Delimitation Commission was constituted in 1973, but there was another one after that.
  • Option (d) Four: This is correct. The four Delimitation Commissions were constituted in 1952, 1963, 1973, and 2002.

Learn more

  • Delimitation: The process of fixing limits or boundaries of territorial constituencies in a country to represent changes in population.
  • Purpose: To ensure equal representation for equal segments of the population and to follow the principle of “One Vote One Value”.
  • Constitutional Basis: Under Article 82, Parliament enacts a Delimitation Act after every Census. Article 170 provides for the division of states into territorial constituencies.
  • Composition: The Delimitation Commission is appointed by the President of India and includes a retired Supreme Court judge, the Chief Election Commissioner, and respective State Election Commissioners.
  • Powers: The Commission’s orders have the force of law and cannot be challenged in any court. The orders are laid before the Lok Sabha and State Legislative Assemblies but cannot be modified.
  • History: Delimitation Commissions have been set up four times: in 1952, 1963, 1973, and 2002. The most recent delimitation was based on the 2001 Census.
  • Challenges: Issues include disparities in constituency sizes and the impact of population control measures on political representation. The Constitution has capped the number of seats in the Lok Sabha and Rajya Sabha to address these issues.
  • Future: The next delimitation is expected after the 2026 Census, following the 84th Amendment to the Constitution.

The Constitution (71st Amendment) Act, 1992 amends the Eighth Schedule to the Constitution to include which of the following languages?

  1. Konkani
  2. Manipuri
  3. Nepali
  4. Maithili

Select the correct answer using the code given below:

(a) 1, 2 and 3
(b) 1, 2 and 4
(c) 1, 3 and 4
(d) 2, 3 and 4

Correct Answer: (a) 1, 2 and 3

  • The Constitution (71st Amendment) Act, 1992 amended the Eighth Schedule to include Konkani, Manipuri, and Nepali languages.
  • Maithili was not included by the 71st Amendment but was added later by the 92nd Amendment Act, 2003.

Learn more

  • 71st Amendment Act, 1992:
    • Purpose: To include Konkani, Manipuri, and Nepali languages in the Eighth Schedule of the Indian Constitution.
    • Significance: Recognized these languages as official languages of India, promoting linguistic inclusivity and cultural diversity.
    • Implementation: Passed by both Lok Sabha and Rajya Sabha on 20 August 1992, received presidential assent on 31 August 1992, and came into force on the same date.
  • Eighth Schedule:
    • Initial Languages: Originally included 14 languages.
    • Amendments:
      • 21st Amendment (1967): Added Sindhi.
      • 71st Amendment (1992): Added Konkani, Manipuri, and Nepali.
      • 92nd Amendment (2003): Added Bodo, Dogri, Maithili, and Santhali.
    • Current Total: Lists 22 languages.
  • Importance of Eighth Schedule:
    • Government Responsibility: The Government of India is responsible for the development of the languages listed in the Eighth Schedule.
    • Cultural Representation: Ensures representation and support for linguistic communities, facilitating educational resources, official documentation, and communication in these languages.
  • Article 344 and 351:
    • Article 344: Provides for the constitution of a commission to make recommendations for the progressive use of Hindi for official purposes.
    • Article 351: Mandates the Union to promote the spread of Hindi and enrich it by assimilating elements from other languages listed in the Eighth Schedule.

Consider the following pairs:

PartyIts Leader
1. Bharatiya Jana SanghDr. Shyama Prasad Mukherjee
2. Socialist PartyC. Rajagopalachari
3. Congress for DemocracyJagjivan Ram
4. Swatantra PartyAcharya Narendra Dev

How many of the above are correctly matched?

(a) Only one
(b) Only two
(c) Only three
(d) All four

Correct Answer: (b) Only two

Explanation:

  • Pair 1: Bharatiya Jana Sangh – Dr. Shyama Prasad Mukherjee
    • Correct: Dr. Shyama Prasad Mukherjee founded the Bharatiya Jana Sangh in 1951 as the political wing of the RSS.
  • Pair 2: Socialist Party – C. Rajagopalachari
    • Incorrect: C. Rajagopalachari was the founder of the Swatantra Party, not the Socialist Party. The Socialist Party was associated with leaders like Jai Prakash Narayan and Ram Manohar Lohia.
  • Pair 3: Congress for Democracy – Jagjivan Ram
    • Correct: Jagjivan Ram was indeed a leader of the Congress for Democracy, a party he founded in 1977.
  • Pair 4: Swatantra Party – Acharya Narendra Dev
    • Incorrect: The Swatantra Party was founded by C. Rajagopalachari in 1959. Acharya Narendra Dev was associated with the Congress Socialist Party.

Learn more

  • Bharatiya Jana Sangh (BJS)
    • Founded in 1951 by Dr. Shyama Prasad Mukherjee.
    • Political wing of the Rashtriya Swayamsevak Sangh (RSS).
    • Advocated for rebuilding India in accordance with Hindu culture.
  • Socialist Party
  • Congress for Democracy
    • Founded by Jagjivan Ram in 1977.
    • Formed as a breakaway faction from the Indian National Congress.
  • Swatantra Party
    • Founded by C. Rajagopalachari in 1959.
    • Advocated for free-market policies and opposed the socialist policies of the Indian National Congress.

Which of the following statements are correct about the Constitution of India?

  1. Powers of the Municipalities are given in Part IX A of the Constitution.
  2. Emergency provisions are given in Part XVIII of the Constitution.
  3. Provisions related to the amendment of the Constitution are given in Part XX of the Constitution.

Select the answer using the code given below:

(a) 1 and 2 only
(b) 2 and 3 only
(c) 1 and 3 only
(d) 1, 2 and 3

Correct Answer: (d) 1, 2 and 3

Explanation:

  • Statement 1:Powers of the Municipalities are given in Part IX A of the Constitution.
    • This statement is correct. Part IXA of the Constitution, added by the 74th Amendment Act, 1992, deals with the Municipalities and includes Articles 243P to 243ZG.
  • Statement 2:Emergency provisions are given in Part XVIII of the Constitution.
    • This statement is correct. Part XVIII of the Constitution contains provisions for emergency situations, including national, state, and financial emergencies, covered under Articles 352 to 360.
  • Statement 3:Provisions related to the amendment of the Constitution are given in Part XX of the Constitution.
    • This statement is correct. Part XX of the Constitution, specifically Article 368, outlines the procedure for amending the Constitution.

Learn more

  • Part IXA: The Municipalities
    • Introduction: Added by the 74th Amendment Act, 1992.
    • Articles: 243P to 243ZG.
    • Types of Municipalities: Nagar Panchayat, Municipal Council, and Municipal Corporation.
    • Functions: Urban planning, regulation of land use, water supply, public health, and more.
    • Finance: State legislatures can authorize municipalities to levy taxes and receive grants.
  • Part XVIII: Emergency Provisions
    • Introduction: Contains provisions for handling emergencies.
    • Articles: 352 to 360.
    • Types of Emergencies: National Emergency (Article 352), State Emergency (Article 356), and Financial Emergency (Article 360).
    • Effects: Alters the distribution of powers between the center and states, impacts fundamental rights, and modifies financial arrangements.
  • Part XX: Amendment of the Constitution
    • Introduction: Governs the process of amending the Constitution.
    • Article: 368.
    • Types of Amendments: By simple majority, by special majority, and by special majority with ratification by half of the state legislatures.
    • Scope: Ensures the Constitution remains adaptable while preserving its fundamental principles.

Which one of the following statements is correct as per the Constitution of India?

(a) Inter-State trade and commerce is a State subject under the State List.
(b) Inter-State migration is a State subject under the State List.
(c) Inter-State quarantine is a Union subject under the Union List.
(d) Corporation tax is a State subject under the State List.

Correct Answer:(c) Inter-State quarantine is a Union subject under the Union List.

Explanation:

  • Option (a): Inter-State trade and commerce is not a State subject under the State List. It falls under the Union List as per the Seventh Schedule of the Indian Constitution, which grants exclusive powers to the Union to legislate on this matter.
  • Option (b): Inter-State migration is not a State subject under the State List. It is also a Union subject under the Union List, which means the central government has the authority to legislate on this matter.
  • Option (c): Inter-State quarantine is indeed a Union subject under the Union List. This means that the central government has the exclusive power to legislate on matters related to inter-state quarantine.
  • Option (d): Corporation tax is not a State subject under the State List. It is a Union subject, meaning the central government has the authority to impose and regulate corporation tax.

Learn more

  • Inter-State Trade and Commerce:
    • Articles 301-307 of the Indian Constitution ensure the freedom of trade, commerce, and intercourse throughout the territory of India.
    • Article 301: Guarantees that trade, commerce, and intercourse throughout India shall be free from restrictions.
    • Article 302: Allows Parliament to impose restrictions on trade, commerce, or intercourse between states in the public interest.
    • Article 303: Prohibits both Parliament and state legislatures from making laws that give preference to one state over another or discriminate between states, except in cases of scarcity of goods.
    • Article 304: Permits state legislatures to impose taxes on goods imported from other states, provided they do not discriminate against goods from other states and are in the public interest.
    • Article 305: Protects existing laws and state monopolies from being affected by Articles 301 and 303, unless directed otherwise by the President.
    • Article 307: Empowers Parliament to appoint an authority to ensure the provisions of Articles 301-304 are carried out effectively.
  • Inter-State Migration:
    • Article 19(1)(e) of the Indian Constitution guarantees the right to reside and settle in any part of the territory of India, subject to reasonable restrictions in the interest of the general public or protection of any scheduled tribe.
    • The Inter-State Migrant Workmen (Regulation of Employment and Conditions of Service) Act, 1979 provides protections for inter-state migrant workers, including licensing of contractors, registration of workers, and guidelines for wages and accommodations.
  • Inter-State Quarantine:
    • Union List: Includes inter-state quarantine, giving the central government the authority to legislate on matters related to quarantine and public health emergencies that affect multiple states.
  • Corporation Tax:
    • Defined under Article 366(6) of the Indian Constitution as a tax on the income of companies, which is exclusively under the jurisdiction of the central government.

Under which of the following Articles of the Constitution of India, has the Supreme Court of India placed the Right to Privacy?

(a) Article 15
(b) Article 16
(c) Article 19
(d) Article 21

Correct Answer: (d) Article 21

Explanation:

  • Article 15: This article prohibits discrimination on grounds of religion, race, caste, sex, or place of birth. It does not address the right to privacy.
  • Article 16: This article ensures equality of opportunity in matters of public employment and prohibits discrimination on similar grounds as Article 15. It does not cover the right to privacy.
  • Article 19: This article guarantees certain freedoms, including freedom of speech and expression, assembly, association, movement, residence, and profession. While it is crucial for personal liberties, it does not specifically address the right to privacy.
  • Article 21: This article guarantees the protection of life and personal liberty. The Supreme Court of India has interpreted the right to privacy as an intrinsic part of the right to life and personal liberty under Article 21.

Learn more:

  • Right to Privacy:
    • Recognized as a fundamental right under Article 21 of the Indian Constitution.
    • The landmark judgment in Justice K.S. Puttaswamy (Retd.) vs Union of India (2017) affirmed this right.
    • The judgment overruled previous decisions in M.P. Sharma vs Satish Chandra and Kharak Singh vs State of Uttar Pradesh, which had denied the existence of a fundamental right to privacy.
    • The right to privacy is essential for the meaningful exercise of other freedoms guaranteed by Part III of the Constitution.
    • The judgment emphasized the need for a robust data protection regime to safeguard individual privacy against state and private actions.
  • Implications:
    • The right to privacy includes the right to be forgotten and the right to be left alone.
    • It has significant implications for data protection laws and surveillance practices in India.
    • The judgment has influenced subsequent rulings on issues like same-sex relationships and decriminalization of adultery, highlighting the broader impact on personal liberties.
  • Government Steps:
    • The Personal Data Protection Bill 2019 aims to establish a comprehensive data protection framework in India.
    • Existing laws like the Information Technology Act, 2000 provide some safeguards against privacy breaches.

What are the duties of the Chief of Defence Staff (CDS) as Head of the Department of Military Affairs?

  1. Permanent Chairman of Chiefs of Staff Committee
  2. Exercise military command over the three Service Chiefs
  3. Principal Military Advisor to Defence Minister on all tri-service matters

Select the correct answer using the code given below:

(a) 1, 2 and 3
(b) 1 and 2 only
(c) 2 and 3 only
(d) 1 and 3 only

Correct Answer: (d) 1 and 3 only

Explanation:

  • Option 1: Permanent Chairman of Chiefs of Staff Committee
    • The Chief of Defence Staff (CDS) serves as the Permanent Chairman of the Chiefs of Staff Committee (COSC). This role involves ensuring coordination and jointness among the three services (Army, Navy, and Air Force) and providing integrated military advice to the Defence Minister.
  • Option 2: Exercise military command over the three Service Chiefs
    • The CDS does not exercise military command over the three Service Chiefs. Instead, the CDS acts as a coordinator and advisor, ensuring jointness and integration among the services without direct command authority over them.
  • Option 3: Principal Military Advisor to Defence Minister on all tri-service matters
    • The CDS is the Principal Military Advisor to the Defence Minister on all tri-service matters. This role includes advising on matters that involve the Army, Navy, and Air Force collectively, ensuring that the Defence Minister receives integrated and comprehensive military advice.

Learn more

Duties of the Chief of Defence Staff (CDS) as Head of the Department of Military Affairs (DMA):

  • Permanent Chairman of the Chiefs of Staff Committee (COSC)
    • The CDS chairs the COSC, which includes the chiefs of the Army, Navy, and Air Force. This role is crucial for fostering jointness and operational synergy among the three services.
  • Principal Military Advisor to the Defence Minister
    • The CDS provides integrated military advice to the Defence Minister on all tri-service matters, ensuring that the Defence Minister receives a unified perspective on military issues.
  • Head of the Department of Military Affairs (DMA)
    • The CDS heads the DMA within the Ministry of Defence, overseeing matters related to the armed forces, including procurement, training, and logistics. This role involves ensuring the optimal utilization of resources and promoting jointness in operations.
  • Advisory Role in the Nuclear Command Authority (NCA)
    • The CDS serves as a military advisor to the NCA, which is responsible for India’s nuclear arsenal. This role involves providing strategic military advice on nuclear matters.
  • Promoting Jointness and Integration
    • The CDS is tasked with promoting jointness in procurement, training, and staffing among the three services. This includes the establishment of joint-service theatre commands to ensure integrated and efficient military operations.
  • Implementation of Defence Reforms
    • The CDS is responsible for implementing various defence reforms aimed at enhancing the combat capabilities of the Indian armed forces. This includes the creation of theatre commands and the rationalization of military infrastructure.
  • Coordination of Defence Planning and Acquisition
    • The CDS plays a key role in coordinating defence planning and acquisition processes, ensuring that the armed forces’ procurement needs are met in a timely and efficient manner.

Operations undertaken by the Army towards upliftment of the local population in remote areas to include addressing of their basic needs is called:

(a) Operation Sankalp
(b) Operation Maitri
(c) Operation Sadbhavana
(d) Operation Madad

Correct Answer: (c) Operation Sadbhavana

Explanation:

  • Operation Sankalp:
    • Initiated by: Indian Navy.
    • Purpose: To ensure the safety and security of Indian vessels in the Persian Gulf and the Gulf of Oman.
    • Background: Launched in response to attacks on merchant ships in the Gulf of Oman in June 2019.
    • Deployment: Involves warships and aircraft to monitor and respond to maritime threats.
  • Operation Maitri:
    • Initiated by: Indian Armed Forces.
    • PurposeRescue and relief operation in Nepal following the 2015 Nepal earthquake.
    • Scope: Included evacuation of Indian and foreign citizens, medical aid, and infrastructure rebuilding.
    • Deployment: Involved medical teams, engineer task forces, and aviation assets.
  • Operation Sadbhavana:
    • Initiated by: Indian Army.
    • PurposeWelfare activities in Jammu and Kashmir and Ladakh to address the needs of people affected by terrorism.
    • Activities: Running Army Goodwill Schools, infrastructure development projects, and educational tours.
    • Objectives: National integration, women empowerment, employment generation, and development activities.
  • Operation Madad:
    • Initiated by: Indian Navy.
    • PurposeRescue and relief operations during natural disasters within India.
    • Notable Operations: 2004 Indian Ocean tsunami and 2018 Kerala floods.
    • Deployment: Involves rescue teams, diving teams, and aerial sorties for relief and rescue.

Learn more

  • Operation Sadbhavana:
    • Objective: To uplift the local population in remote areas of Jammu and Kashmir and Ladakh.
    • Key Activities:
      • Education: Running Army Goodwill Schools to improve educational standards.
      • Infrastructure Development: Projects include construction of community halls, water supply schemes, and road repairs.
      • Women Empowerment: Training programs in vocational skills like Pashmina shawl weaving and Yak cheese making.
      • Healthcare: Medical camps, provision of medical equipment, and upgradation of medical infrastructure.
      • Community Development: Initiatives like solar lighting, arboriculture, and establishment of bore wells.
    • Impact: Enhances national integration, empowers women, generates employment, and contributes to nation-building.

The longest border between any two countries in the world is between:

(a) Canada and the United States of America
(b) Chile and Argentina
(c) China and India
(d) Kazakhstan and Russian Federation

Correct Answer:
(a) Canada and the United States of America

Explanation:

  • The longest border between any two countries in the world is between Canada and the United States of America. This border stretches for 8,893 kilometers.

Dealing with All Options:

  • Option (a): Canada and the United States of America
    • The Canada–United States border is the longest international border in the world, measuring 8,893 kilometers.
  • Option (b): Chile and Argentina
    • The Argentina–Chile border is the longest international border in South America and the third longest in the world, measuring 5,308 kilometers.
  • Option (c): China and India
    • The China–India border, also known as the Line of Actual Control, measures 3,380 kilometers and is not among the longest borders in the world.
  • Option (d): Kazakhstan and Russian Federation
    • The Kazakhstan–Russia border is the second longest international border in the world, measuring 7,644 kilometers.

Learn more

  • Canada–United States Border:
    • Length: 8,893 kilometers.
    • Characteristics: It includes the border between Canada and the contiguous United States and the border between Canada and Alaska.
    • Management: The International Boundary Commission and the International Joint Commission handle boundary marking and water issues, respectively.
  • Kazakhstan–Russia Border:
    • Length: 7,644 kilometers.
    • Characteristics: It is the longest continuous international border in the world.
    • Geography: Runs from the Caspian Sea to the tripoint with China, traversing the Eurasian Steppe and Altai Mountains.
  • Argentina–Chile Border:
    • Length: 5,308 kilometers.
    • Characteristics: Runs along the Andes and on the islands of Tierra del Fuego.
    • Disputes: There are some border disputes, particularly around the Southern Patagonian Ice Field.
  • China–India Border:
    • Length: 3,380 kilometers.
    • Characteristics: Known as the Line of Actual Control, it is a notional demarcation line separating Indian-controlled territory from Chinese-controlled territory.
    • Disputes: The border is a major point of contention between the two countries, with several skirmishes and ongoing disputes.

Which of the following statements about the Ethics Committee in the Lok Sabha are correct?

  1. Initially it was an ad-hoc Committee.
  2. Only a Member of the Lok Sabha can make a complaint relating to unethical conduct of a member of the Lok Sabha.
  3. This Committee cannot take up any matter which is sub-judice.

Select the answer using the code given below:

(a) 1 and 2 only
(b) 2 and 3 only
(c) 1 and 3 only
(d) 1, 2 and 3

Correct Answer: (c) 1 and 3 only

Explanation

  • Statement 1: Initially, it was an ad-hoc Committee.
    • This statement is correct. The Ethics Committee in the Lok Sabha was initially constituted as an ad-hoc committee in 2000 by the late Speaker G M C Balayogi and became a permanent part of the House only in 2015.
  • Statement 2: Only a Member of the Lok Sabha can make a complaint relating to unethical conduct of a member of the Lok Sabha.
    • This statement is incorrect. Any person can complain against a Member through another Lok Sabha MP, along with evidence of the alleged misconduct and an affidavit stating that the complaint is not “false, frivolous, or vexatious”.
  • Statement 3: This Committee cannot take up any matter which is sub-judice.
    • This statement is correct. The Ethics Committee does not entertain complaints based only on media reports or on matters that are sub-judice.

Learn more

Lok Sabha’s Ethics Committee

  • Constitution and History:
    • The idea of ethics panels for the two Houses was first mooted at a Presiding Officers’ Conference held in Delhi in 1996.
    • The Rajya Sabha constituted its Ethics Committee on March 4, 1997, inaugurated in May 1997.
    • In the Lok Sabha, a study group of the House Committee of Privileges recommended the constitution of an Ethics Committee in 1997, but it was not taken up until the 13th Lok Sabha.
    • The late Speaker G M C Balayogi constituted an ad-hoc Ethics Committee in 2000, which became a permanent part of the House in 2015.
  • Functions:
    • The Ethics Committee examines every complaint relating to unethical conduct of a Member of Lok Sabha referred to it by the Speaker and makes recommendations as it deems fit.
    • It oversees the moral and ethical conduct of members and examines cases of misconduct referred to it.
  • Procedure for Complaints:
    • Any person can complain against a Member through another Lok Sabha MP, along with evidence of the alleged misconduct and an affidavit stating that the complaint is not “false, frivolous, or vexatious”.
    • The Speaker can refer any complaint against an MP to the Committee.
    • The Committee does not entertain complaints based only on media reports or on matters that are sub-judice.
    • The Committee makes a prima facie inquiry before deciding to examine a complaint and makes its recommendations after evaluating the complaint.
  • Potential Outcomes:
    • If the Ethics Committee finds merit in the complaint, it can recommend punishments such as suspension of the MP for a specified period.
    • The House, which includes all MPs, will ultimately decide whether to accept the committee’s recommendation and determine the nature and extent of the punishment, if any.
    • If the accused were to be expelled or face a potentially adverse decision, they could challenge it in a court of law on limited grounds such as unconstitutionality, gross illegality, or denial of natural justice.
  • Notable Cases:
    • In 2005, the two Houses adopted motions to expel 10 Lok Sabha MPs and one Rajya Sabha MP accused of agreeing to ask questions in Parliament for money, based on the Bansal Committee’s report.

Consider the following statements regarding ‘Nari Shakti Vandan Adhiniyam’:

  1. Provisions will come into effect from the 18th Lok Sabha.
  2. This will be in force for 15 years after becoming an Act.
  3. There are provisions for the reservation of seats for Scheduled Castes Women within the quota reserved for the Scheduled Castes.

Which of the statements given above are correct?

(a) 1, 2 and 3
(b) 1 and 2 only
(c) 2 and 3 only
(d) 1 and 3 only

Correct Answer: (c) 2 and 3 only

Explanation:

  • Statement 1: The provisions of the Nari Shakti Vandan Adhiniyam will not come into effect from the 18th Lok Sabha. The bill specifies that the reservation will be implemented only after a delimitation exercise is completed, which is scheduled to be held after the first census post-2026. Therefore, the earliest implementation could be in the 2029 general election.
  • Statement 2: The bill will be in force for 15 years after becoming an Act. This is explicitly mentioned in the bill.
  • Statement 3: There are provisions for the reservation of seats for Scheduled Castes (SC) women within the quota reserved for the SCs. One-third of the seats reserved for SCs and STs will be allocated to women.

Learn more

Nari Shakti Vandan Adhiniyam

  • Introduction:
    • The Nari Shakti Vandan Adhiniyam, also known as the Women’s Reservation Bill 2023, aims to reserve one-third of the seats in the Lok Sabha, State legislative assemblies, and the Delhi assembly for women.
  • Key Features:
    • Reservation Period: The reservation will be in force for 15 years after the bill becomes an Act.
    • Implementation: The provisions will come into effect only after a delimitation exercise based on the first census conducted after the bill’s commencement. This means the earliest implementation could be in the 2029 general election.
    • Sub-Quota for SC/ST: One-third of the seats reserved for Scheduled Castes (SCs) and Scheduled Tribes (STs) will be allocated to women.
    • Rotation of Seats: The seats reserved for women will be rotated after each delimitation exercise.
    • Exclusion of Rajya Sabha and Legislative Councils: The bill does not provide for women’s reservation in the Rajya Sabha and State Legislative Councils.
  • Historical Context:
    • The bill has been introduced multiple times in the past but failed to pass due to lack of consensus. It was first introduced as the 81st Constitutional Amendment Bill in 1996.
    • The current bill, known as the 128th Constitutional Amendment Bill, was introduced in the Lok Sabha on 19 September 2023 and passed by both houses of Parliament in September 2023.
  • Current Status:
    • The bill has been signed by the President and the gazette notification has been published, making it a law. However, its implementation is contingent upon the completion of the delimitation exercise post-2026.
  • Impact:
    • Once implemented, the number of women members in the Lok Sabha is expected to rise significantly, from the current 82 to 181.
    • The bill aims to enhance women’s participation in policy-making at both state and national levels, thereby strengthening democracy.

Which of the following statements about ‘Exercise Mitra Shakti-2023’ are correct?

  1. This was a joint military exercise between India and Bangladesh.
  2. It commenced in Aundh (Pune).
  3. Joint response during counter-terrorism operations was a goal of this operation.
  4. Indian Air Force was a part of this exercise.

Select the answer using the code given below:

(a) 1, 2 and 3
(b) 1, 2 and 4
(c) 1, 3 and 4
(d) 2, 3 and 4

Correct Answer: (d) 2, 3 and 4

Explanation:

  • Statement 1: This statement is incorrectExercise Mitra Shakti-2023 was a joint military exercise between India and Sri Lanka, not Bangladesh.
  • Statement 2: This statement is correct. The exercise commenced in Aundh (Pune).
  • Statement 3: This statement is correct. One of the goals of the exercise was to jointly rehearse responses during counter-terrorism operations.
  • Statement 4: This statement is correct. The Indian Air Force was a part of this exercise, with 15 personnel participating.

Learn more

  • Exercise Mitra Shakti: This is an annual joint military exercise conducted between India and Sri Lanka. It aims to enhance bilateral defense cooperation and interoperability between the armed forces of the two nations.
  • Commencement and Location: The ninth edition of the exercise, Exercise Mitra Shakti-2023, commenced on November 16, 2023, and was held in Aundh (Pune).
  • Participants: The Indian contingent included 120 personnel from the Maratha Light Infantry Regiment and 15 personnel from the Indian Air Force. The Sri Lankan side was represented by personnel from the 53 Infantry Division and five personnel from the Sri Lankan Air Force.
  • Objectives: The primary goal was to jointly rehearse sub-conventional operations under Chapter VII of the United Nations Charter, focusing on counter-terrorist operations. The exercise included tactical actions such as raid, search and destroy missions, heliborne operations, and more.
  • Modern Techniques: The exercise incorporated the use of drones, counter-unmanned aerial systems, and helicopters. Drills to secure helipads and undertake casualty evacuation during counter-terrorist operations were also part of the exercise.
  • Knowledge Exchange: Both sides exchanged views and practices on a wide spectrum of combat skills, facilitating mutual learning and enhancing defense cooperation. This exercise also aimed to foster strong bilateral relations between India and Sri Lanka.

A Writ of Prohibition is an order issued by the Supreme Court or High Courts to:

(a) a government officer prohibiting him from taking a particular action.
(b) the Parliament/Legislative Assembly to pass a law on Prohibition.
(c) the lower court prohibiting continuation of proceedings in a case.
(d) the Government prohibiting it from following an unconstitutional policy.

Correct Answer: (c) the lower court prohibiting continuation of proceedings in a case.

Explanation

  • (a) a government officer prohibiting him from taking a particular action:
    • This option is incorrect. The writ of prohibition is not issued to government officers but to lower courts or quasi-judicial bodies to prevent them from exceeding their jurisdiction or acting unlawfully.
  • (b) the Parliament/Legislative Assembly to pass a law on Prohibition:
    • This option is incorrect. The writ of prohibition is not directed at legislative bodies to pass laws. It is specifically aimed at judicial or quasi-judicial authorities to stop them from proceeding with cases beyond their jurisdiction.
  • (c) the lower court prohibiting continuation of proceedings in a case:
    • This option is correct. The writ of prohibition is issued by a higher court to a lower court or tribunal to prevent it from continuing proceedings in a case where it lacks jurisdiction or is acting contrary to the principles of natural justice.
  • (d) the Government prohibiting it from following an unconstitutional policy:
    • This option is incorrect. The writ of prohibition is not used to prohibit government policies. It is specifically used to control the jurisdictional overreach of lower courts and quasi-judicial bodies.

Learn more

  • Definition and Purpose:
    • The writ of prohibition is a judicial order issued by a higher court to a lower court or tribunal, directing it to cease proceedings in a case that falls outside its jurisdiction or violates the principles of natural justice.
  • Jurisdiction:
    • It is issued by the Supreme Court under Article 32 and by the High Courts under Article 226 of the Indian Constitution.
  • Scope:
    • The writ can only be issued against judicial and quasi-judicial authorities and not against administrative authorities, legislative bodies, or private individuals.
  • Historical Context:
    • The writ of prohibition has its roots in English common law and was historically used to control the jurisdiction of ecclesiastical courts by common law courts.
  • Grounds for Issuance:
    • It is typically sought when there is a lack of jurisdictionexcess of jurisdiction, or a violation of natural justice such as denial of a fair hearing or bias.
  • Comparison with Certiorari:
    • While the writ of prohibition is used to stop ongoing proceedings, the writ of certiorari is used to quash a decision already made by a lower court or tribunal.
  • Case Laws:
    • Notable cases include Brij Khandelwal v. India (1975)S. Govind Menon v. Union of India (1967), and Hari Vishnu v. Syed Ahmed Ishaque (1955), which illustrate the application and scope of the writ of prohibition in India.

Consider the following statements:

  1. It is the Governor of the State who recognizes and declares any community of that State as a Scheduled Tribe.
  2. A community declared as a Scheduled Tribe in a State need not be so in another State.

Which of the statements given above is/are correct?

(a) 1 only
(b) 2 only
(c) Both 1 and 2
(d) Neither 1 nor 2

Correct Answer: (b) 2 only

Explanation:

  • Statement 1: It is not the Governor of the State who recognizes and declares any community of that State as a Scheduled Tribe. According to Article 342 of the Constitution of India, it is the President who may, with respect to any State or Union Territory, and where it is a state, after consultation with the Governor thereof, by public notification, specify the tribes or tribal communities or parts of or groups within tribes or tribal communities which shall, for the purposes of this Constitution, be deemed to be Scheduled Tribes in relation to that State or Union Territory. Therefore, this statement is incorrect.
  • Statement 2: A community declared as a Scheduled Tribe in a State need not be so in another State. This is correct as the list of Scheduled Tribes is State/UT specific and a community declared as a Scheduled Tribe in one State need not be recognized as such in another State.

Learn more

  • Scheduled Tribes (STs): These are indigenous communities recognized by the government for special protection and assistance due to their social, educational, and economic backwardness. The term ‘Scheduled Tribes’ first appeared in the Constitution of India under Article 366(25), which refers to tribes or tribal communities as specified under Article 342.
  • Article 342: This article prescribes the procedure for the specification of Scheduled Tribes. The President may, with respect to any State or Union Territory, and where it is a state, after consultation with the Governor thereof, by public notification, specify the tribes or tribal communities or parts of or groups within tribes or tribal communities which shall, for the purposes of this Constitution, be deemed to be Scheduled Tribes in relation to that State or Union Territory. Parliament may by law include in or exclude from the list of Scheduled Tribes specified in a notification issued under clause (1) any tribe or tribal community or part of or group within any tribe or tribal community.
  • State/UT Specific Lists: The list of Scheduled Tribes is specific to each State or Union Territory. A community recognized as a Scheduled Tribe in one State may not be recognized as such in another State. This ensures that the recognition of tribal communities is tailored to the specific social and cultural contexts of each State or Union Territory.
  • Criteria for Specification: The criteria for specifying a community as a Scheduled Tribe include indications of primitive traits, distinctive culture, geographical isolation, shyness of contact with the community at large, and backwardness. These criteria are not explicitly mentioned in the Constitution but have become well-established through various reports and commissions.
  • Particularly Vulnerable Tribal Groups (PVTGs): These are a subset of Scheduled Tribes that are characterized by pre-agriculture level of technology, stagnant or declining population, low literacy, and a subsistence level of economy. They receive special attention and benefits under various government schemes.
  • Population Statistics: According to the 2011 Census, the tribal population in India is approximately 10.45 crore, constituting 8.6% of the total population. The literacy rate among Scheduled Tribes improved from 47.1% in 2001 to 59% in 2011.

With reference to Union Budget, consider the following statements:

  1. The Union Finance Minister on behalf of the Prime Minister lays the Annual Financial Statement before both the Houses of Parliament.
  2. At the Union level, no demand for a grant can be made except on the recommendation of the President of India.

Which of the statements given above is/are correct?

(a) 1 only
(b) 2 only
(c) Both 1 and 2
(d) Neither 1 nor 2

Correct Answer: (b) 2 only

Explanation:

  • Statement 1: The Union Finance Minister presents the Annual Financial Statement before both Houses of Parliament, but not on behalf of the Prime Minister. This is a key duty of the Finance Minister, who heads the Ministry of Finance and is responsible for the fiscal policy of the government.
  • Statement 2: At the Union level, no demand for a grant can be made except on the recommendation of the President of India. This is mandated by Article 113 of the Indian Constitution, which states that no demand for grants can be presented in the Lok Sabha without the President’s prior approval.

Learn more

  • Union Budget: The Union Budget, also known as the Annual Financial Statement, is a comprehensive statement of the government’s estimated receipts and expenditures for a financial year. It is presented by the Finance Minister in Parliament.
  • Article 112: This article of the Indian Constitution mandates that the President shall cause to be laid before both Houses of Parliament a statement of the estimated receipts and expenditure of the Government of India for that year.
  • Presentation Date: Since 2017, the Union Budget is presented on the 1st of February each year, earlier it was presented on the last working day of February.
  • Budget Speech: The Finance Minister delivers the budget speech in the Lok Sabha (Lower House) first. The budget can be presented in two or more parts, and no discussion takes place on the day of presentation.
  • Demand for Grants: These are proposals seeking withdrawal of money from the Consolidated Fund of India, presented in the Lok Sabha. Each ministry prepares its demand for grants, which includes both charged and voted expenditures.
  • Approval Process: The Lok Sabha has the power to approve or reject the demands for grants. No demand for a grant can be made without the President’s recommendation, ensuring a check on the financial proposals.
  • Budget Cycle: The budget preparation begins in the third quarter of the financial year and involves multiple stages, including estimates of expenditures and revenues, narrowing of deficit, and presentation and approval of the budget.

Who of the following is the author of the books “The India Way” and “Why Bharat Matters”?

(a) Bhupender Yadav
(b) Nalin Mehta
(c) Shashi Tharoor
(d) Subrahmanyam Jaishankar

Correct Answer: (d) Subrahmanyam Jaishankar

Explanation:

  • Subrahmanyam Jaishankar is the author of the books “The India Way” and “Why Bharat Matters”.

Dealing with Other Options:

  • Bhupender Yadav: He is not the author of these books. He is known for his book “The Rise of the BJP: The Making of the World’s Largest Political Party”.
  • Nalin Mehta: He is not the author of these books. He is known for his book “The New BJP: Modi and the Making of the World’s Largest Political Party”.
  • Shashi Tharoor: He is not the author of these books. He is known for his books such as “An Era of Darkness” and “Why I Am A Hindu”.

Learn more

  • Subrahmanyam Jaishankar:
    • Career: Jaishankar has had a distinguished career in diplomacy, serving as India’s Ambassador to the United States, China, and the Czech Republic, and as High Commissioner to Singapore.
    • Minister of External Affairs: He has been serving as the Minister of External Affairs of India since May 30, 2019.
    • Books:
      • “The India Way: Strategies for an Uncertain World”: This book provides insights into India’s foreign policy and strategic approach to navigating global challenges. It emphasizes strategic autonomyhistorical contextglobal challenges, and diplomatic flexibility.
      • “Why Bharat Matters”: This book delves into why India, or Bharat, is a crucial player in today’s interconnected world. It highlights economic potentialcultural influencegeopolitical significance, and vision for the future.
  • Bhupender Yadav:
    • Book: “The Rise of the BJP: The Making of the World’s Largest Political Party”.
    • Focus: The book discusses the history and rise of the Bharatiya Janata Party (BJP) in India.
  • Nalin Mehta:
    • Book: “The New BJP: Modi and the Making of the World’s Largest Political Party”.
    • Focus: The book provides a non-partisan account of the rise of the BJP in India.
  • Shashi Tharoor:
    • Books: “An Era of Darkness”, “Why I Am A Hindu”, “The Paradoxical Prime Minister”.
    • Focus: Tharoor’s works often explore India’s history, culture, and politics, with a critical view of colonialism and contemporary political issues.

Consider the following pairs:

CountryReason for being in the news
1. ArgentinaWorst economic crisis
2. SudanWar between the country’s regular army and paramilitary forces
3. TurkeyRescinded its membership of NATO

How many of the pairs given above are correctly matched?

(a) Only one pair
(b) Only two pairs
(c) All three pairs
(d) None of the pairs

Correct Answer: (b) Only two pairs

Explanation:

  • Pair 1: Argentina – Worst economic crisis
    • Correct: Argentina is indeed experiencing one of the worst economic crises, characterized by soaring inflation, high poverty rates, and significant fiscal deficits. The country has been grappling with severe economic instability, with inflation rates reaching alarming levels and economic activity plunging.
  • Pair 2: Sudan – War between the country’s regular army and paramilitary forces
    • Correct: Sudan is embroiled in a devastating war between the Sudanese Armed Forces (SAF) and the Rapid Support Forces (RSF). This conflict has led to significant loss of life, displacement, and a severe humanitarian crisis.
  • Pair 3: Turkey – Rescinded its membership of NATO
    • Incorrect: Turkey has not rescinded its membership of NATO. Although there have been discussions and political tensions, Turkey remains a member of NATO and has recently approved Sweden’s membership bid.

Learn more

  • Argentina’s Economic Crisis:
    • Inflation: Argentina has one of the highest inflation rates in the world, with annual inflation reaching 287% in March 2024.
    • Fiscal Deficits: Persistent fiscal deficits have plagued the country, with the government defaulting on its sovereign debt multiple times.
    • Economic Reforms: President Javier Milei’s administration has implemented aggressive fiscal and monetary reforms to stabilize the economy, including reducing public spending and aiming for a fiscal surplus.
  • Sudan’s War:
    • Conflict Origins: The war in Sudan began in April 2023 due to a power struggle between the SAF and the RSF, following a military coup in 2021.
    • Humanitarian Crisis: The conflict has resulted in nearly 16,000 deaths, massive displacement, and severe food insecurity, with millions of people affected.
    • International Response: Despite the severity of the crisis, international attention and aid have been limited, exacerbating the humanitarian situation.
  • Turkey and NATO:
    • Membership Status: Turkey remains a member of NATO and has not rescinded its membership. The country has been involved in negotiations and has recently approved Sweden’s bid to join NATO.
    • Political Tensions: Turkey’s relations with NATO have been strained due to various geopolitical issues, but it continues to be an active member of the alliance.

Consider the following statements:

Statement-I:
Sumed pipeline is a strategic route for Persian Gulf oil and natural gas shipments to Europe.

Statement-II:
Sumed pipeline connects the Red Sea with the Mediterranean Sea.

Which one of the following is correct in respect of the above statements?

(a) Both Statement-I and Statement-II are correct and Statement-II explains Statement-I
(b) Both Statement-I and Statement-II are correct, but Statement-II does not explain Statement-I
(c) Statement-I is correct, but Statement-II is incorrect
(d) Statement-I is incorrect, but Statement-II is correct

The correct answer is (a) Both Statement-I and Statement-II are correct and Statement-II explains Statement-I.

Explanation

  • (a) Both Statement-I and Statement-II are correct and Statement-II explains Statement-I:
    • Statement-I: The Sumed pipeline is indeed a strategic route for Persian Gulf oil and natural gas shipments to Europe. It serves as a major route for transporting crude oil from the Middle East to Europe and beyond, bypassing the limitations of the Suez Canal for fully loaded supertankers.
    • Statement-II: The Sumed pipeline connects the Red Sea with the Mediterranean Sea. It runs from the Ain Sokhna terminal in the Gulf of Suez to the Sidi Kerir port on the Mediterranean Sea.
    • Explanation: Statement-II explains Statement-I because the Sumed pipeline’s connection between the Red Sea and the Mediterranean Sea allows it to serve as a strategic route for oil shipments from the Persian Gulf to Europe, bypassing the Suez Canal’s limitations.
  • (b) Both Statement-I and Statement-II are correct, but Statement-II does not explain Statement-I:
    • This option is incorrect because Statement-II directly explains why the Sumed pipeline is a strategic route for oil shipments to Europe.
  • (c) Statement-I is correct, but Statement-II is incorrect:
    • This option is incorrect because both statements are correct. The Sumed pipeline does connect the Red Sea with the Mediterranean Sea.
  • (d) Statement-I is incorrect, but Statement-II is correct:
    • This option is incorrect because Statement-I is correct. The Sumed pipeline is a strategic route for oil shipments from the Persian Gulf to Europe.

Learn more

  • Sumed Pipeline:
    • Strategic Importance: The Sumed pipeline is crucial for transporting crude oil from the Middle East to Europe, bypassing the Suez Canal’s limitations for fully loaded supertankers.
    • Route: It connects the Red Sea (Ain Sokhna terminal) with the Mediterranean Sea (Sidi Kerir port).
    • Capacity: The pipeline has a capacity of 2.5 million barrels per day and consists of two parallel lines of 42 inches in diameter.
    • Ownership: It is operated by the Arab Petroleum Pipeline Company, a joint venture involving EGPC, Saudi Aramco, IPIC, and other stakeholders.
    • Economic and Environmental Benefits: The pipeline reduces transportation time and costs, generates revenue for Egypt, and is considered safer and more environmentally friendly compared to maritime shipping.

Consider the following statements:

  1. The Red Sea receives very little precipitation in any form.
  2. No water enters the Red Sea from rivers.

Which of the statements given above is/are correct?

(a) 1 only
(b) 2 only
(c) Both 1 and 2
(d) Neither 1 nor 2

The correct answer is (c) Both 1 and 2.

  • Statement 1: The Red Sea receives very little precipitation in any form.
    • This statement is correct. The Red Sea region receives extremely low rainfall, averaging about 60 mm (2.36 inches) per year, with most of it occurring as short showers often accompanied by thunderstorms and dust storms.
  • Statement 2: No water enters the Red Sea from rivers.
    • This statement is also correct. The Red Sea is surrounded by desert or semi-desert areas and has no major freshwater inflow from rivers.

Learn more

  • Geographical Location:
    • The Red Sea is a semi-enclosed tropical basin located between northeastern Africa and the Arabian Peninsula. It extends from the Mediterranean Sea in the northwest to the Indian Ocean in the southeast, connected via the Suez Canal and the Strait of Bab-el-Mandeb, respectively.
  • Climate:
    • The climate of the Red Sea is characterized by very high surface temperatures and high salinity. It experiences two monsoon seasons: a northeasterly monsoon and a southwesterly monsoon. The average surface water temperature ranges from 26 °C (79 °F) in the north to 30 °C (86 °F) in the south during summer, with minimal variation in winter.
  • Hydrology:
    • The Red Sea has a unique hydrological system with no river inflow and high evaporation rates, leading to high salinity levels. The water mass exchanges with the Arabian Sea and the Indian Ocean via the Gulf of Aden, which helps mitigate the high salinity caused by evaporation.
  • Salinity:
    • The Red Sea is one of the saltiest bodies of water in the world, with salinity levels ranging from 36 parts per thousand in the southern part to 41 parts per thousand in the northern part around the Gulf of Suez. This high salinity is due to high evaporation rates and the lack of significant freshwater inflow.
  • Biodiversity:
    • The Red Sea supports a diverse range of marine life, including coral reefs, sea turtles, dugongs, dolphins, and many endemic fish species. The unique habitats are recognized and protected by initiatives such as the Ras Mohammed National Park in Egypt.
  • Economic Aspects:
    • The Red Sea region is rich in mineral resources, including petroleum deposits, evaporite deposits, sulfur, phosphates, and heavy-metal deposits. It is also a crucial trade route connecting Europe to Asia through the Suez Canal and is known for its recreational diving sites.
  • Formation:
    • The Red Sea is a relatively young sea, formed by the seafloor spreading that began about 55 million years ago. The basin continues to widen at a rate of 1-2 cm per year, making it one of the youngest oceanic zones on Earth.

According to the Environmental Protection Agency (EPA), which one of the following is the largest source of sulphur dioxide emissions?

(a) Locomotives using fossil fuels
(b) Ships using fossil fuels
(c) Extraction of metals from ores
(d) Power plants using fossil fuels

The correct answer is:(d) Power plants using fossil fuels

  • Power plants using fossil fuels are the largest source of sulfur dioxide (SO₂) emissions. According to the Environmental Protection Agency (EPA), the majority of SO₂ emissions come from the combustion of fossil fuels at power plants, accounting for 73% of total emissions.

Explanation

  • (a) Locomotives using fossil fuels: While locomotives do contribute to SO₂ emissions, they are not the largest source. They are considered smaller sources compared to power plants and other industrial facilities.
  • (b) Ships using fossil fuels: Ships are significant contributors to SO₂ emissions, especially those using high-sulfur fuels. However, they are not the largest source. Recent regulations have aimed to reduce sulfur content in marine fuels, which has decreased their contribution.
  • (c) Extraction of metals from ores: The extraction and processing of metals, such as smelting, do release SO₂. However, this source is smaller compared to emissions from power plants.
  • (d) Power plants using fossil fuels: This is the largest source of SO₂ emissions. Power plants burning coal and oil release substantial amounts of SO₂ into the atmosphere, making them the primary contributors.

Learn more

  • Sulfur Dioxide (SO₂) Emissions:
    • Definition: SO₂ is a colorless, reactive gas with a pungent odor, produced primarily from the combustion of sulfur-containing fuels like coal and oil.
    • Health Effects: Short-term exposure can cause respiratory problems, including difficulty breathing, throat irritation, and aggravation of asthma. Long-term exposure can lead to respiratory illnesses and cardiovascular diseases.
    • Environmental Impact: SO₂ contributes to the formation of acid rain, which can harm ecosystems, damage buildings, and reduce visibility by forming sulfate particles that cause haze.
    • Regulations: Various regulations, such as the Clean Air Act in the U.S., aim to limit SO₂ emissions from power plants and other industrial sources. Technologies like scrubbers are used to remove SO₂ from emissions before they are released into the atmosphere.
    • Reduction Measures: Transitioning to cleaner energy sources, such as natural gas and renewables, and implementing stricter emission controls can significantly reduce SO₂ emissions.

Consider the following statements:

Statement-I:
There is instability and worsening security situation in the Sahel region.

Statement-II:
There have been military takeovers/coups d’état in several countries of the Sahel region in the recent past.

Which one of the following is correct in respect of the above statements?

(a) Both Statement-I and Statement-II are correct and Statement-II explains Statement-I
(b) Both Statement-I and Statement-II are correct, but Statement-II does not explain Statement-I
(c) Statement-I is correct, but Statement-II is incorrect
(d) Statement-I is incorrect, but Statement-II is correct

Correct Answer: (a) Both Statement-I and Statement-II are correct and Statement-II explains Statement-I

Explanation

  • Option (a): Both Statement-I and Statement-II are correct and Statement-II explains Statement-I
    • Statement-I: There is instability and worsening security situation in the Sahel region.
      • This is correct as the Sahel region has been experiencing severe security and humanitarian crises, exacerbated by violent extremist organizations and weak governance.
    • Statement-II: There have been military takeovers/coups d’état in several countries of the Sahel region in the recent past.
      • This is also correct as there have been multiple coups in countries like Mali, Burkina Faso, and Niger since 2020.
    • Explanation: The military takeovers have contributed to the instability and worsening security situation by disrupting governance and enabling extremist groups to exploit the resulting power vacuums.
  • Option (b): Both Statement-I and Statement-II are correct, but Statement-II does not explain Statement-I
    • While both statements are correct, the military coups have directly contributed to the instability and worsening security situation, making this option incorrect.
  • Option (c): Statement-I is correct, but Statement-II is incorrect
    • This option is incorrect because Statement-II is correct; there have indeed been military coups in the Sahel region.
  • Option (d): Statement-I is incorrect, but Statement-II is correct
    • This option is incorrect because Statement-I is correct; the Sahel region is experiencing instability and a worsening security situation.

Learn more

  • Instability and Worsening Security Situation:
    • The Sahel region faces severe and complex security and humanitarian crises, exacerbated by violent extremist organizations like Jama’at Nusrat al-Islam wal Muslimeen (JNIM) and Islamic State in the Greater Sahara (ISGS).
    • The region has seen a dramatic increase in violence, with significant fatalities and displacement of populations.
  • Military Takeovers/Coups d’état:
    • Since 2020, there have been multiple successful coups in the Sahel region, including in Mali, Burkina Faso, and Niger.
    • These coups have often been justified by the perpetrators as necessary to address corruption, economic mismanagement, and security failures of the previous regimes.
  • Impact of Coups on Security:
    • The coups have led to the withdrawal of international counterterrorism support, creating a vacuum that extremist groups have exploited.
    • The military regimes have adopted militarized approaches to counter-insurgency, which have often escalated violence rather than containing it.
  • International and Regional Dynamics:
    • The coups have strained relations with Western countries and organizations like ECOWAS, leading to sanctions and further isolation of the junta-led governments.
    • The involvement of external actors like Russia and the Wagner Group has further complicated the security landscape, as these actors have supported the military regimes in exchange for influence in the region.

Consider the following statements:

Statement-I:
India does not import apples from the United States of America.

Statement-II:
In India, the law prohibits the import of Genetically Modified food without the approval of the competent authority.

Which one of the following is correct in respect of the above statements?

(a) Both Statement-I and Statement-II are correct and Statement-II explains Statement-I
(b) Both Statement-I and Statement-II are correct, but Statement-II does not explain Statement-I
(c) Statement-I is correct, but Statement-II is incorrect
(d) Statement-I is incorrect, but Statement-II is correct

Correct Answer: (d) Statement-I is incorrect, but Statement-II is correct

Explanation

  • Option (a): Both Statement-I and Statement-II are correct and Statement-II explains Statement-I
    • Incorrect: Statement-I is incorrect because India does import apples from the United States. The import of US apples surged 40 times in three months after India removed the retaliatory import duty in September 2023. Statement-II is correct, but it does not explain Statement-I.
  • Option (b): Both Statement-I and Statement-II are correct, but Statement-II does not explain Statement-I
    • Incorrect: Statement-I is incorrect as India does import apples from the United States. Statement-II is correct, but since Statement-I is incorrect, this option is invalid.
  • Option (c): Statement-I is correct, but Statement-II is incorrect
    • Incorrect: Statement-I is incorrect because India imports apples from the United States. Statement-II is correct as Indian law prohibits the import of genetically modified food without the approval of the competent authority.
  • Option (d): Statement-I is incorrect, but Statement-II is correct
    • Correct: Statement-I is incorrect because India imports apples from the United States. Statement-II is correct as Indian law prohibits the import of genetically modified food without the approval of the competent authority.

Learn More

  • India’s Import of US Apples:
    • India imports apples from the United States. The import of US apples surged 40 times in three months after India removed the retaliatory import duty in September 2023.
    • The import of US apples had dropped significantly due to a 20% retaliatory tariff imposed in 2019, but the removal of this tariff has led to a significant increase in imports.
  • Regulation of Genetically Modified (GM) Foods in India:
    • Indian law prohibits the import of genetically modified food without the approval of the competent authority, which includes the Genetic Engineering Appraisal Committee (GEAC) and the Food Safety and Standards Authority of India (FSSAI).
    • The Environment Protection Act of 1986 and the Food Safety and Standards Act of 2006 govern the approval process for GM foods.
    • Despite the prohibition, there have been instances of GM food products being imported into India without proper approvals, highlighting regulatory challenges.
  • Impact of Retaliatory Tariffs:
    • The 20% retaliatory tariff imposed by India in 2019 on US apples was in response to the US increasing tariffs on Indian steel and aluminum products.
    • This tariff significantly reduced the import of US apples, but its removal has led to a resurgence in imports.
  • Market Dynamics:
    • The removal of the additional duty on US apples has allowed them to compete on a level playing field with apples from other countries like Turkey, Italy, Chile, and New Zealand.
    • The import of apples from other countries increased significantly during the period when the additional duty on US apples was in place.

With reference to the Speaker of the Lok Sabha, consider the following statements:

While any resolution for the removal of the Speaker of the Lok Sabha is under consideration:

  1. He/She shall not preside.
  2. He/She shall not have the right to speak.
  3. He/She shall not be entitled to vote on the resolution in the first instance.

Which of the statements given above is/are correct?

(a) 1 only
(b) 1 and 2 only
(c) 2 and 3 only
(d) 1, 2 and 3

Correct Answer: (a) 1 only

Explanation:

  • Statement 1: He/She shall not preside.
    • This statement is correct. When a resolution for the removal of the Speaker is under consideration, the Speaker shall not preside over the sitting of the House. Instead, the Deputy Speaker or another member appointed by the House will preside over the proceedings.
  • Statement 2: He/She shall not have the right to speak.
    • This statement is incorrect. The Speaker may be present during the consideration of the resolution and has the right to speak in the House. However, the Speaker does not preside over the session.
  • Statement 3: He/She shall not be entitled to vote on the resolution in the first instance.
    • This statement is incorrect. The Speaker can vote on the resolution in the first instance as a member of the House. The restriction applies to presiding over the session, not to voting.

Learn more

Speaker of the Lok Sabha: Roles and Responsibilities

  • Presiding Officer: The Speaker is the presiding officer of the Lok Sabha, responsible for maintaining order and decorum in the House.
  • Election: The Speaker is elected by the members of the Lok Sabha from among themselves, usually from the ruling party, after informal consultations with other parties.
  • Term: The Speaker’s term is coterminous with the Lok Sabha, typically five years, but they continue in office until a new Speaker is elected.
  • Powers:
    • Administrative and Discretionary Powers: The Speaker has significant administrative and discretionary powers, including deciding the agenda, interpreting rules, and ensuring discipline among members.
    • Casting Vote: In case of a tie, the Speaker has a casting vote to break the deadlock.
    • Certification of Money Bills: The Speaker certifies whether a bill is a money bill, which is crucial for legislative procedures.
    • Joint Sessions: The Speaker presides over joint sessions of both Houses of Parliament.
    • Disqualification Powers: The Speaker can disqualify members on grounds of defection under the Tenth Schedule of the Constitution.
  • Removal:
    • Resolution: The Speaker can be removed by a resolution passed by a majority of all the then members of the House, with a 14-day notice period.
    • Presence During Resolution: The Speaker may be present during the consideration of the resolution but cannot preside over the session.

With reference to the Indian Parliament, consider the following statements:

  1. A bill pending in the Lok Sabha lapses on its dissolution.
  2. A bill passed by the Lok Sabha and pending in the Rajya Sabha lapses on the dissolution of the Lok Sabha.
  3. A bill in regard to which the President of India notified his/her intention to summon the Houses to a joint sitting lapses on the dissolution of the Lok Sabha.

Which of the statements given above is/are correct?

(a) 1 only
(b) 1 and 2
(c) 2 and 3
(d) 3 only

Correct Answer: (b) 1 and 2

Explanation:

  • Statement 1: A bill pending in the Lok Sabha lapses on its dissolution. This is correct. When the Lok Sabha is dissolved, all business including bills, motions, resolutions, notices, petitions pending before it or its committees lapse.
  • Statement 2: A bill passed by the Lok Sabha and pending in the Rajya Sabha lapses on the dissolution of the Lok Sabha. This is also correct. Bills that originate and are passed in the Lok Sabha but are pending in the Rajya Sabha are considered lapsed upon the dissolution of the Lok Sabha.
  • Statement 3: A bill in regard to which the President of India notified his/her intention to summon the Houses to a joint sitting lapses on the dissolution of the Lok Sabha. This is incorrect. If the President has notified the holding of a joint sitting before the dissolution of Lok Sabha, the bill does not lapse.

Learn more

  • Lapsing of Bills in Indian Parliament:
    • Dissolution of Lok Sabha: When the Lok Sabha is dissolved, all business including bills, motions, resolutions, notices, petitions pending before it or its committees lapse. This means that any bill pending in the Lok Sabha or passed by the Lok Sabha but pending in the Rajya Sabha lapses.
    • Bills in Rajya Sabha: A bill pending in the Rajya Sabha but not passed by the Lok Sabha does not lapse upon the dissolution of the Lok Sabha.
    • Joint Sitting Notification: If the President has notified the holding of a joint sitting before the dissolution of Lok Sabha, the bill does not lapse.
    • Bills Passed by Both Houses: Bills that have been passed by both Houses but are pending assent from the President do not lapse.
    • Returned Bills: Bills passed by both Houses but returned by the President for reconsideration do not lapse.
    • Prorogation: Prorogation does not affect the bills or any other business pending before the House. All pending notices (other than those for introducing bills) lapse on prorogation and fresh notices have to be given for the next session.

With reference to the Parliament of India, consider the following statements:

  1. Prorogation of a House by the President of India does not require the advice of the Council of Ministers.
  2. Prorogation of a House is generally done after the House is adjourned sine die but there is no bar to the President of India prorogating the House which is in session.
  3. Dissolution of the Lok Sabha is done by the President of India who, save in exceptional circumstances, does so on the advice of the Council of Ministers.

Which of the statements given above is/are correct?

(a) 1 only
(b) 1 and 2
(c) 2 and 3
(d) 3 only

Correct Answer: (c) 2 and 3

  • Statement 1: Incorrect. The prorogation of a House by the President of India is generally done on the advice of the Council of Ministers.
  • Statement 2: Correct. Prorogation of a House is generally done after the House is adjourned sine die, but there is no bar to the President of India proroguing the House which is in session.
  • Statement 3: Correct. Dissolution of the Lok Sabha is done by the President of India, who, save in exceptional circumstances, does so on the advice of the Council of Ministers.

Learn more

  • Prorogation:
    • Definition: Prorogation means the termination of a session of the House by an order made by the President under Article 85(2)(a) of the Constitution.
    • Process: Usually, within a few days after the House is adjourned sine die by the presiding officer, the President issues a notification for the prorogation of the session.
    • Effects: Prorogation terminates both the sitting and session of the House. All pending notices (other than those for introducing bills) lapse on prorogation, and fresh notices have to be given for the next session.
  • Dissolution:
    • Definition: Dissolution ends the very life of the existing House, and a new House is constituted after general elections are held.
    • Process: The President can dissolve the Lok Sabha on the advice of the Prime Minister and the Council of Ministers. This is a power that the President has under Article 85 of the Indian Constitution.
    • Effects: When the Lok Sabha is dissolved, all business including bills, motions, resolutions, notices, petitions, and so on pending before it or its committees lapse.
  • Adjournment:
    • Definition: An adjournment suspends the work in a sitting for a specified time, which may be hours, days, or weeks. The time of reassembly is specified.
    • Adjournment Sine Die: This means terminating a sitting of Parliament for an indefinite period. The power of adjournment sine die lies with the presiding officer of the House.
  • Summoning:
    • Definition: Summoning is the process of calling all members of the Parliament to meet. It is the duty of the Indian President to summon each House of the Parliament from time to time.
    • Frequency: The maximum gap between two sessions of Parliament cannot be more than six months. In other words, the Parliament should meet at least twice a year.

Consider the following statements:

Statement-I:
The European Parliament approved The Net-Zero Industry Act recently.

Statement-II:
The European Union intends to achieve carbon neutrality by 2040 and therefore aims to develop all of its own clean technology by that time.

Which one of the following is correct in respect of the above statements?

(a) Both Statement-I and Statement-II are correct and Statement-II explains Statement-I
(b) Both Statement-I and Statement-II are correct, but Statement-II does not explain Statement-I
(c) Statement-I is correct, but Statement-II is incorrect
(d) Statement-I is incorrect, but Statement-II is correct

Correct answer is (c) Statement-I is correct, but Statement-II is incorrect

Explanation:

  • Statement-I is correct: The European Parliament recently approved the Net-Zero Industry Act. This act aims to bolster EU production in technologies needed for decarbonisation and was formally approved by the European Parliament on April 25, 2024.
  • Statement-II is incorrect: The European Union aims to achieve carbon neutrality by 2050, not 2040. This objective is part of the European Green Deal and is a legally binding target under the European Climate Law.

Learn more

  • Net-Zero Industry Act:
    • Objective: To bolster EU production in technologies needed for decarbonisation.
    • Target: Produce 40% of annual deployment needs in net-zero technologies by 2030 and capture 15% of the global market value for these technologies.
    • Technologies Supported: Renewable technologies, nuclear, industrial decarbonisation, grid, energy storage technologies, and biotech.
    • Permitting Process: Simplified with maximum timelines for project authorisation.
    • Net-Zero Acceleration Valleys: Initiatives to speed up the permitting process by delegating parts of the evidence collection for environmental assessments to member states.
    • Sustainability and Resilience Criteria: Applied in public procurement procedures and auctions to deploy renewable energy, with a minimum of 30% of the volume auctioned annually in the member state or a maximum of 6GW auctioned annually per country.
  • EU’s Climate Neutrality Goal:
    • Target Year: 2050.
    • Legal Framework: European Climate Law.
    • Alignment: Part of the European Green Deal and consistent with the Paris Agreement.
    • National Strategies: Member States have developed national long-term strategies to achieve the greenhouse gas emissions reductions needed to meet their commitments under the Paris Agreement and the EU’s climate neutrality objective.

Consider the following statements:

Statement-I:
Recently, Venezuela has achieved a rapid recovery from its economic crisis and succeeded in preventing its people from fleeing/emigrating to other countries.

Statement-II:
Venezuela has the world’s largest oil reserves.

Which one of the following is correct in respect of the above statements?

(a) Both Statement-I and Statement-II are correct and Statement-II explains Statement-I
(b) Both Statement-I and Statement-II are correct, but Statement-II does not explain Statement-I
(c) Statement-I is correct, but Statement-II is incorrect
(d) Statement-I is incorrect, but Statement-II is correct

Correct Answer: (d) Statement-I is incorrect, but Statement-II is correct

Explanation

  • Option (a): Both Statement-I and Statement-II are correct and Statement-II explains Statement-I
    • Incorrect: While Venezuela does have the world’s largest oil reserves, it has not achieved a rapid recovery from its economic crisis, nor has it succeeded in preventing its people from fleeing to other countries. The country continues to face significant economic challenges and a large-scale migration crisis.
  • Option (b): Both Statement-I and Statement-II are correct, but Statement-II does not explain Statement-I
    • Incorrect: Statement-I is incorrect. Venezuela has not achieved a rapid economic recovery, and the migration crisis persists. Therefore, this option is invalid.
  • Option (c): Statement-I is correct, but Statement-II is incorrect
    • Incorrect: Statement-I is incorrect as Venezuela has not achieved a rapid economic recovery, and the migration crisis continues. Statement-II is correct as Venezuela has the world’s largest oil reserves.
  • Option (d): Statement-I is incorrect, but Statement-II is correct
    • Correct: Statement-I is incorrect because Venezuela is still experiencing significant economic challenges and a migration crisis. Statement-II is correct as Venezuela has the world’s largest oil reserves.

Learn more

  • Economic Crisis:
    • Persistent Challenges: Despite some economic improvements, Venezuela continues to face significant economic challenges, including high inflation and food insecurity.
    • Migration Crisis: Over 7.7 million Venezuelans have fled the country due to economic hardship and political instability, making it one of the largest migration crises in the world.
  • Oil Reserves:
    • Largest in the World: Venezuela has the largest proven oil reserves in the world, totaling approximately 303 billion barrels.
    • Economic Dependency: The country’s economy is heavily dependent on oil exports, which have been affected by political instability and international sanctions.
  • Inflation and Food Insecurity:
    • High Inflation: Although inflation rates have decelerated, they remain high, impacting the population’s access to food and other essential goods.
    • Food Prices: Food prices have stabilized but remain significantly higher than in previous years, making it difficult for many Venezuelans to afford basic necessities.
  • Humanitarian Crisis:
    • Health and Services: The humanitarian crisis includes severe shortages of basic goods, healthcare, and essential services, exacerbating the living conditions for many Venezuelans.
    • International Aid: Efforts by international organizations and neighboring countries continue to provide humanitarian assistance, but the needs remain vast and underfunded.
  • Political Instability:
    • Government Control: The political landscape is marked by the control of key institutions by the ruling party, leading to a weakened system of checks and balances and ongoing political repression.
    • Sanctions and International Relations: International sanctions, particularly from the United States, have significantly impacted Venezuela’s economy, although some easing of sanctions has occurred in recent years.

With reference to the Digital India Land Records Modernisation Programme, consider the following statements:

  1. To implement the scheme, the Central Government provides 100% funding.
  2. Under the Scheme, Cadastral Maps are digitised.
  3. An initiative has been undertaken to transliterate the Records of Rights from local language to any of the languages recognized by the Constitution of India.

Which of the statements given above are correct?

(a) 1 and 2 only
(b) 2 and 3 only
(c) 1 and 3 only
(d) 1, 2 and 3

Correct Answer: (d) 1, 2 and 3

Explanation:

  • Statement 1: To implement the scheme, the Central Government provides 100% funding.
  • Statement 2: Under the Scheme, Cadastral Maps are digitised.
    • This statement is correct. The DILRMP includes the digitization of Cadastral Maps as part of its objectives.
  • Statement 3: An initiative has been undertaken to transliterate the Records of Rights from local language to any of the languages recognized by the Constitution of India.
    • This statement is correct. The government has initiated the transliteration of Records of Rights into any of the 22 Schedule VIII languages of the Constitution.

Learn more

  • Objective of DILRMP:
    • The Digital India Land Records Modernisation Programme (DILRMP) aims to develop a modern, comprehensive, and transparent land record management system.
    • It seeks to create an Integrated Land Information Management System to improve real-time information on land, optimize land resource use, benefit landowners and prospectors, assist in policy and planning, reduce land disputes, check fraudulent transactions, and enable information sharing with various organizations.
  • Achievements:
    • Computerization of Land Records: 95.08% of Record of Rights (RoR) have been completed.
    • Digitization of Cadastral Maps: 68.02% of Cadastral Maps have been digitized.
    • Computerization of Registration: 94.95% of Sub-Registrar Offices (SROs) have been computerized.
    • Integration of SROs with Land Records: 87.48% of SROs have been integrated with land records.
  • Linkage with e-Courts:
    • The linkage of e-Courts with land records and registration databases aims to provide authentic first-hand information to courts, resulting in the speedy disposal of cases and reduction in land disputes. Pilot tests have been successfully conducted in Haryana, Maharashtra, and Uttar Pradesh.
  • Transliteration Initiative:
    • To address linguistic barriers, the government, with technical support from C-DAC Pune, has undertaken an initiative to transliterate Records of Rights into any of the 22 Schedule VIII languages. Pilot tests are underway in eight states.
  • SVAMITVA Scheme:
    • The SVAMITVA Scheme aims to provide ‘Record of Rights’ to village household owners by mapping land parcels using drone technology. It is implemented by the Ministry of Panchayati Raj in collaboration with state departments and the Survey of India.

With reference to the ‘Pradhan Mantri Surakshit Matritva Abhiyan’, consider the following statements:

  1. This scheme guarantees a minimum package of antenatal care services to women in their second and third trimesters of pregnancy and six months post-delivery health care service in any government health facility.
  2. Under this scheme, private sector health care providers of certain specialities can volunteer to provide services at nearby government health facilities.

Which of the statements given above is/are correct?

(a) 1 only
(b) 2 only
(c) Both 1 and 2
(d) Neither 1 nor 2

Correct Answer: (b) 2 only

Explanation:

  • Statement 1: This statement is incorrect. The Pradhan Mantri Surakshit Matritva Abhiyan (PMSMA) guarantees a minimum package of antenatal care services to women in their second and third trimesters of pregnancy but does not include six months post-delivery health care services in any government health facility.
  • Statement 2: This statement is correct. Under this scheme, private sector health care providers of certain specialities can volunteer to provide services at nearby government health facilities.

Learn more

  • Objective: The Pradhan Mantri Surakshit Matritva Abhiyan (PMSMA) aims to provide assured, comprehensive, and quality antenatal care to all pregnant women on the 9th of every month.
  • Target Group: The scheme targets pregnant women in their second and third trimesters to ensure they receive at least one checkup during these critical periods.
  • Services Provided:
    • Antenatal Care (ANC): Includes diagnostics, screening, and management of conditions like hypertension and diabetes.
    • Counselling: Pregnant women receive counselling on the safety and risks associated with pregnancies.
    • High-Risk Pregnancy Management: Identification and management of high-risk pregnancies to promote safe motherhood.
  • Implementation:
    • Fixed-Day Service: Services are provided on the 9th of every month at designated government health facilities.
    • Private Sector Involvement: The scheme encourages private practitioners to volunteer and provide services at government health facilities.
  • Accessibility:
    • Mobile/Web-Based Application: Helps pregnant women find the nearest PMSMA facility.
    • Registration: Pregnant women are registered in a separate PMSMA register during their first visit.
  • Launch and Coverage: The scheme was launched by the Ministry of Health and Family Welfare (MoHFW) and is part of the Reproductive Maternal Neonatal Child and Adolescent Health (RMNCH+A) Strategy.

With reference to the Pradhan Mantri Shram Yogi Maan-dhan (PM-SYM) Yojana, consider the following statements:

  1. The entry age group for enrolment in the scheme is 21 to 40 years.
  2. Age specific contribution shall be made by the beneficiary.
  3. Each subscriber under the scheme shall receive a minimum pension of ₹ 3,000 per month after attaining the age of 60 years.
  4. Family pension is applicable to the spouse and unmarried daughters.

Which of the statements given above is/are correct?

(a) 1, 3 and 4
(b) 2 and 3 only
(c) 2 only
(d) 1, 2 and 4

Correct Answer: (b) 2 and 3 only

Explanation:

  • Statement 1: The entry age group for enrolment in the scheme is 18 to 40 years, not 21 to 40 years.
  • Statement 2: Age-specific contribution shall be made by the beneficiary. This is correct as the contribution amount varies with the age of entry into the scheme.
  • Statement 3: Each subscriber under the scheme shall receive a minimum pension of ₹ 3,000 per month after attaining the age of 60 years. This is correct.
  • Statement 4: Family pension is applicable only to the spouse, not to unmarried daughters.

Learn more

Pradhan Mantri Shram Yogi Maan-dhan (PM-SYM) Yojana

  • Objective: To provide old age protection and social security to unorganized workers.
  • Eligibility:
    • Age group: 18 to 40 years.
    • Monthly income: ₹ 15,000 or less.
    • Exclusions: Should not be covered under any statutory social security schemes like NPS, ESIC, EPFO, and should not be an income tax payer.
  • Contribution:
    • Age-specific contributions: Beneficiaries contribute a fixed amount based on their age at entry, with a matching contribution from the Central Government.
    • Example: A 29-year-old would contribute ₹ 100 per month, matched by the government.
  • Benefits:
    • Pension: Minimum assured pension of ₹ 3,000 per month after attaining the age of 60 years.
    • Family pension: 50% of the pension amount to the spouse after the subscriber’s death.
  • Enrollment Process:
    • Documents required: Aadhaar card, savings bank account/Jan-Dhan account number.
    • Enrollment centers: Common Service Centres (CSCs), LIC offices, EPFO/ESIC offices, and Labour offices.
  • Contribution Mode: Monthly contributions are auto-debited from the subscriber’s bank account.
  • Grievance Redressal: A dedicated call center and web portal/app for complaints and queries.

If you like this post, please share your feedback in the comments section below so that we will upload more posts like this.

Subscribe
Notify of
guest
0 Comments
Inline Feedbacks
View all comments
X
Home Courses Plans Account